*NURSING > STUDY GUIDE > Chamberlain College of Nursing:NR 566 Week 5 final study guide Latest,100% CORRECT (All)

Chamberlain College of Nursing:NR 566 Week 5 final study guide Latest,100% CORRECT

Document Content and Description Below

Chamberlain College of Nursing:NR 566 Week 5 final study guide Latest Chapter 18: Drugs Affecting the Hematopoietic System • Know the pharmacodynamics, pharmacotherapeutics clinical use, drug in... teractions and adverse drug reactions for: Anticoagulants • Pharmacodynamics • Oral anticoagulants such as warfarin (Coumadin) inhibit the hepatic synthesis of several clotting factors, including factor X. • Heparin inhibits the activity of several activated clotting factors by accelerating the activity of antithrombin III. • LMWH enoxaparin (Lovenox) potentiates the activity of antithrombin III and inactivates factors Xa and IIa (thrombin). • Dabigatran (Pradaxa) is a direct thrombin inhibitor. • Thrombin is required for the conversion of fibrinogen to fibrin in the clotting cascade, thus dabigatran's inhibition of thrombin prevents thrombi from forming. • Fondaparinux (Arixtra) is a selective inhibitor of antithrombin III and a factor Xa inhibitor. • Rivaroxaban (Xarelto) an anticoagulant, is a highly selective factor Xa inhibitor that inhibits thrombin formation and the development of thrombi. • Apixaban (Eliquis) is a selective inhibitor of factor Xa. • Aspirin antagonizes the cyclooxygenase pathway and interferes with platelet aggregation. • NSAIDs have this same action. • NSAIDs are not used as antiplatelet drugs, but this explains why concurrent use with anticoagulants is contraindicated • Ticlopidine (Ticlid) and clopidogrel (Plavix) reduce platelet aggregation by inhibiting the ADP pathway of platelets. • Unlike aspirin, they have no effect on prostaglandin metabolism. • Ticagrelor (Brilinta) reversibly interacts with the platelet P2Y12 ADP-receptor to prevent platelet activation. • Vorapaxar (Zontivity) is a protease-activated receptor-1 (PAR-1) antagonist, inhibiting thrombin-induced and thrombin receptor agonist peptide-induced platelet aggregation • Pharmacotherapeutics: • Precautions and Contraindications • All anticoagulants are contraindicated for patients who are hypersensitive to the drug or actively bleeding or who have hemophilia, thrombocytopenia, severe HTN, intracranial hemorrhage, infective endocarditis, active tuberculosis, or ulcerative lesions of the GI tract. • Heparins are contraindicated in advanced hepatic or renal disease. • They may be used in patients who are actively bleeding to treat DIC • Heparin is Pregnancy Category C: stillbirth, prematurity • Some heparin preparations contain benzyl alcohol: known to cause “gasping syndrome”: • fatal toxicity in neonates • Hyperkalemia may develop • Use for patient with DM or renal insufficiency requires care and frequent monitoring of aPTT • Has been associated with fatal medication errors r/t different strengths of preparations • JCo: anticoagulant therapy is a National patient Safety Goal: plan in place at each facility to reduce patient harm • LMWHs are contraindicated for patients with allergies to pork, sulfites, or benzyl alcohol; uncontrolled bleeding; and in patients who have antiplatelet antibodies • Renal impairment: cautious use • Body weight less than 50 kg associated with increased r/f bleeding: • enoxaparin dose adjustment • Cautious use: untreated HTN, retinopathy (HTN or DM caused), severe liver disease, recent Hx of ulcer, or malignancy • Not used for thromboprophylaxis in patients with mechanical heart values: especially pregnant (r/f heart value thrombosis) • Enoxaparin: Preg Cat B, tinzaparin: teratogenicity and fetal death, fondaparinux: Preg B • First line drug for women who require antithrombotic therapy during pregnancy: LMWH • Pharmacokinetics of LMWH is altered during pregnancy • Warfarin • Hepatic dysfunction potentiates response through impaired synthesis of coagulation factors • Use with caution: Hypermetabolic states produced by fever or hyperthyroidism increase responsiveness to warfarin: • r/t increased catabolism of vit K dependent coagulation factors • Increased r/f bleeding in older adults • Caution use based on balance between potential for decreased r/f thromboembolism and the risk for bleeding especially in those with dementia or severe cognitive impairment: Hx of three falls in the previous year or recurrent injurious falls, uncontrolled HTN, or non-adherent or unreliable • Warfarin is Pregnancy Category X: Crosses placenta and can cause hemorrhagic disorders in the fetus and serious birth defects • Safe during lactation • Rivaroxaban (Xarelto): Black-Box Warning: premature discontinuation of anticoagulants including rivaroxaban may lead to thrombotic events. • An increased risk of stroke is seen in patients with atrial fibrillation when transitioning to warfarin • Rivaroxaban is Pregnancy Category C and is not recommended for use in pregnant women. • Apixaban (Eliquis): Black Box warning premature discontinuation leading to thrombotic events • Although there are no well-controlled studies: Pregnancy Category B • Hypersensitivity to aspirin and cross-sensitivity with NSAIDs may occur, contraindicating the drug • Aspirin hypersensitivity is more prevalent in patients with asthma, nasal polyps, or chronic urticaria. • Reye syndrome has been associated with its use in children and teenagers who have influenza or chickenpox. • Reversible hepatotoxicity has occurred • Use with caution in liver damage, preexisting hypoprothrombinemia, or vit K deficiency • Preg Cat C and Cat D in third trimester • Avoid during lactation • Clopidogrel and ticlopidine: severe hepatic disease (r/f bleeding d/o), do not use in these patients • Not recommended for patients with GI d/o • Preg Category B • Ticlopidine: clearance increased with age, older adults increased sensitivity to this drug (closely monitor or ADRs) • Older adults: increased levels of clopidogrel: no dosage adjustments • In older adults clopidogrel is a safer drug • Ticlopidine: elevations in cholesterol and TC within 1 month of therapy: factor in HLD • Ticagrelor: Black-Box Warning to not use in a patient with active pathological bleeding or history of intracranial hemorrhage. • Ticagrelor should be discontinued 5 days prior to any surgery. • Dabigatran: Black-Box Warning concerning discontinuation increasing risk of thrombotic events. • There is no reversal agent available for dabigatran • Vorapaxar: Black-Box Warning to not use in patients with a history of stroke, TIA, intracranial hemorrhage, or active pathological bleeding. • Vorapaxar is Pregnancy Category B, with no congenital malformations found in animal studies • ADRs • All anticoagulants: excessive bleeding • Risk is higher early in therapy, with wide fluctuations in aPTT or INR, and older adults (especially women over 60) • Heparins: cause thrombocytopenia and anemia • More likely with bovine than with porcine • Early thrombocytopenia 2 to 3 days after starting therapy and delayed forms 7 to 12 days • Platelet below 100,000: d/c heparin • Enoxaparin antidote: protamine sulfate 1 mg for each mg of enoxaparin dalteparin and tinzaparin: 1 mg for each 100 anti-Xa IUs of dalteparin • Slow IV injection • Fondaparinux: no known antidote • Heparin has a short half-life: Tx is usually d/c of the drug • Protamine sulfate antidote for heparin OD • Warfarin: toxicity and OD Tx withholding one or more doses • Or 1 to 10 mg of vitamin K is the antidote for minor bleeding • 5 to 50 may be used for frank bleeding • Hemorrhagic skin necrosis in women and cyanotic toes in men have been reported • r/t transient inhibition in proteins S and C • Allergic reactions: symmetrical, maculopapular, erythematous lesions • Some are isolated and some confluent: face, neck, and torso • May not appear until 8 to 10 days • Ribaroxaban: bleeding is major ADR • No reversal agent • Back pain, upper ABD pain, osteoarthritis, dyspepsia, and fatigue • Apixaban: bleeding • No reversal agent • ASA: gastric erosions: increased r/f serious upper GI bleeding • More likely when used in combo with other anticoags such as warfarin • Salicylism (tinnitus) : serum levels above 200 mcg/mL • Toxicity: tinnitus, HA, hyperventilation, agitation, confusion, lethargy, diarrhea, and sweating • Ticagrelor: bleeding • Dyspnea: sometimes improves with use or must be d/c • HA, cough, dizziness, and nausea • Ticlopidine: reversible neutropenia at 3 weeks to 3 months after initiation of therapy • Severe neutropenia (<450) or thrombocytopenia (<80,000) d/c drug • Clopidogrel: ADRs similar to ASA, safe and effective • Dabigatran: bleeding • GI ADRs: dyspepsia and gastritis-like symptoms • Angioedema and thrombocytopenia • Vorapaxar: bleeding • Anemia, depression, and rashes or eruptions • Drug Interactions • Cephalosporins and PCNs given parenterally associated with coagulopathies, increased r/f bleeding when given with Heparin • Second and third gen cephalosporins and high doses of PCNs: increased r/f bleeding with Warfarin • Drugs that affect platelet functioning or cause hypoprothrombinemia (ASA, NSAIDS, dipyridamole, quinidine, and valproic acid: increased r/f bleeding • Heparin and LMWHs: similar drug interactions: also interact with antiplatelets including NSAIDS and dextran • Foods can effect anticoags • Warfarin: many drug interactions and increased INR monitoring within 4 to 7 days when other drugs are started or stopped even ABX • Vit K: antagonistic interaction with warfarin: decreases effectiveness • Inhibitors of CYP 1A2, CYP 2C9, or CYP 3A4 increase effect of warfarin • Inhibitors decrease the effectiveness of warfarin • Interacts with any other drug that increased bleeding: anticoags, antiplatelets, NSAIDS and SSRIs • ABX and antifungals may affect INR • Herbal Productions may increase bleeding: ginkgo biloba and garlic • Decrease effectiveness: St John’s wort, ginseng, and coenzyme Q10 • May influence metabolism of warfarin: St John’s wort, echinacea, ginkgo, goldenseal, AND GRAPEFRUIT JUICE • Dabigatran: rifampin (decreased levels of dabigatran) • Dronedarone increased dabigatran levels by 70 to 150% • CCB verapamil: increases dabigatran • Amiodarone increases dabigatran • Clopidogrel increases dabigatran • Ticagrelor: metabolized by CYP 3A4: avoid strong inhibitors ketoconazole, itraconazole, voriconazole, clarithromycin, nefazodone, ritonavir, saquinavir, nelfinavir, indinavir, atazanavir, and telithromycin • Avoid inducers: rifampin, dexamethasone, phenytoin carbamazepine, and phenobarbital • Vorapaxar: avoid with CYP3A inhibitors • Avoid use of inducers (rifampin, carbamazepine, St John’s Wort, and phenytoin • Clinical Use and Dosing Prevention and Treatment of Thromboembolism • Warfarin: drug of choice for prevention of venous thrombosis, systemic thrombosis, and pulmonary embolism o For prevention: give dose sufficient for INR between 2 and 3 o Beginning dose is 10 mg daily for the first 2 days with a recheck of INR in two to three doses o ICSI guidelines: recommend starting warfarin at 5 mg/day with a range of 2.5 mg to 7.5 mg/day o Further dosing is based on INR o Lower initiation doses for: ▪ Older than 75 ▪ Multiple comorbid conditions ▪ Poor nutrition (low albumin) ▪ Elevated INR when off warfarin ▪ Elevated LFTS ▪ Changing thyroid status o If INR is greater than 2 after the first 3 doses, consider decreased the dose by one half o If INR raises rapidly: search for reasons: drug interactions, poor nutrition, infection, or systemic disease process • Acute PE, DVT, or acute systemic embolization: admit to hospital for IV anticoagulation o Heparin, LMWH, or SC fondaparinux and then placed on oral anticoags o Treat while waiting on Dx tests o Acute PE: heparin, LMWH, or fondaparinux for at least 5 days until INR is greater than or equal to 2 for 24 hrs o ACCP recommends staring warfarin on the first day of Tx in conjunction with heparin, LMWH, or fondaparinux or SX heparin o PE due to transient (reversible) risk factor is warfarin for 3 months • Superficial Vein Thrombosis: 45 days of prophylaxis with fondaparinux or LMWH o Lower limb of at least 5 cm in length o ACCP recommends fondaparinux 2.5 mg/day over prophylactic LMWH • Deep Vein Thrombosis: Divided the Tx of acute DVT into acute treatment of DVT of the leg and Tx of the upper extremity DVT o Leg or UE: short term Tx with SC LMWHs, SC fondaparinux, or IV or SC heparin for at least 5 days and until the INR is 2 for 24 hr o LMWH and SC fondaparinux: preferred over heparin o Warfarin: initiated on the first Tx day (like PE) • Idiopathic DVT: maintained on warfarin with target of 2.5 (range 2.0 to 3.0) for 3 months • Indwelling Cline: at r/f DVT o Tx is the same as for leg DVT o After Cath removed: 3 months of anticoags (Warfarin) • All other patients requiring anticoags therapy can be started on Warfarin as OPs o 10 mg/day or 5 mg/day, o Unless patient wt. is less than 110 lbs., over 75, or at ARF bleeding ▪ Start these patients on 2.5 mg/day o Steady state is achieved in 5 to 7 days: dosage adjustments are made ATT based on INR • Goal of therapy is an INR of 2 to 3 for all Tx durations • For patients with DVT or PE and CA: LMWH for 3 to 6 months o Referral for management • Air travel time longer than 8 hrs: increases r/f thrombus formation o Wear loose clothing, avoid tight clothing around the waist or lower extremities, good hydration, and frequent calf muscle exercises o At high risk for DVT: wear compression stockings 15 to 30 mm Hg during air travel • Immobility (bedrest): ambulation as tolerated o Use of compression stocking with 30 to 40 mm Hg for 2 years after DVT episode Prevention of Embolic Stroke in AFib • Low risk for stroke: ASA 75 mg to 325 mg daily • Intermediate risk for stroke: oral anticoags o Anticoags or a combination of ASA and clopidogrel • High risk for stroke: oral anticoags o ACCP recommends: 150 mg of dabigatran BID rather than warfarin • For patients with paroxysmal AFib at high risk for stroke: warfarin target INR between 2 to 3 • For AFib and mitral stenosis: follow recommendation of those at high risk o If valve replacement: follow recommendations for those with heart valves regardless of AFib • Apixaban: to reduce risk of stroke and systemic embolism in non-valvular AFib o 5 mg BID o For those older than 80, less than 60 kg, or crt >1.5 dose at 2.5 mg BID Mitral Valve Disease • Rheumatic MVD but no AFib and left atrial diameter of less than 55 mm: no therapy • Rheumatic MVD with left atrial diameter of greater than 55 mm or left atrial thrombus or AFib or previous embolism: warfarin target INR of 2.5 • Therapy is continued indefinitely in all cases Antithrombotic Therapy for Ischemic Stroke • Acute ischemic stroke or TIA: early (within 48 hours) ASA therapy at 160 to 325 mg • ACCP has three options for long term therapy after noncardioembolic ischemic stroke or TIA • 75 to 100 mg ASA/day • Combo of ASA (25 mg) and ER dipyridamole or clopidogrel • Patients with Hx of ischemic stroke or TIA with AFib: 150 mg dabigatran BID over warfarin Recurrent Embolism or Prosthetic Heart Valves • Warfarin: drug of choice o Therapy initiated and maintained the same as for prevention of venous thrombosis o Except that the target INR is valve dependent • For patients with additional risk factors (AFib, MI, left atrial enlargement, endocardial damage, and low EF) o 75 to 100 mg/day of ASA should be added to Warfarin protocol o The same is true for patients with caged ball or caged disc valves • Bioprosthetic valves in SR: 75 to 100 mg/day of ASA o Therapy is continued indefinitely for mechanical heart valves • For systemic embolization that recurs after 6 months of therapy: therapy is continues for an additional 12 months • Pregnant patients with MHV: require anticoags o Use one of three treatment regiments ▪ LMWH BID ▪ SC heparin every 12 hours ▪ Or SX heparin or LMWH until the 13th week of pregnancy then warfarin until close to delivery with LMWH or heparin is resumed o Warfarin is a known teratogen: may cause fetal hemorrhage o Women with MHV at high risk of thromboembolism: ASA 75 to 100 mg/day added to anticoags regimen o Women on warfarin planning on pregnancy: frequent pregnancy tests and substitution of heparin or LMWH when pregnancy is achieved o Anticoagulation specialist and perinatologist Prevention of MI • Several protocols are effective in prevention of MI in patients with and w/o CAD o Over 50 w/o symptomatic CVD: ASA 75 to 100 mg/day o Established CAD (1 year post ACS, with prior revascularization, coronary stenoses greater than 50% by angiogram, and/or evidence for cardiac ischemia on Dx testing): ASA 75 to 100 mg/day or clopidogrel (75 mg/day) o Patients with ACS who have undergone PCI with or w/o stent placement: dual therapy: 90 mg ticagrelor BID plus ASA 75 to 100 mg/day ▪ ACCP: recommends use of ticagrelor over clopidogrel in PCI • Patient who have experienced an MI: anticlot therapy o STEMI: fibrinolytics then ongoing therapy after hospital d/c • Post-AMI TX: • Anterior AMI w/ r/f LV thrombus who do not get a stent: warfarin (INR target 2 to 3) and ASA 75-100 mg/day • Patients with an acute MI who have a bare metal stent: triple therapy o Warfarin (INR target 2-3), ASA low dose, and 75 mg daily clopidogrel for a month after stent placement o Then warfarin and one antiplatelet drug for months 2 and 3 o After 3 months: d/c warfarin and replaced with dual antiplatelet therapy (90 mg ticagrelor BID plus ASA 75 to 100 mg/day for up to 12 months • Patients with DM: ASA 75 to 162 mg/day • Vorapaxar: approved for reduction of thrombotic CV events in patients with Hx of MI or with PAD o Patient takes one 2.08 mg tablet daily in addition to ASA or clopidogrel Prevention of Post-Operative Thromboembolism • J Co requires that all hospital have a formal active strategy to prevent venous thrombosis • LMWH, low dose heparin, or fondaparinux o Moderate-risk major general surgery, higher risk patients who are having a major procedure for CA, major vascular surgery, major gynecological surgery, major urological procedures, bariatric surgery, thoracic surgery, and many orthopedic surgeries • Length of Tx: determined by the type of surgery, ranging from a single dose to 10 days for most orthopedic procedures, to 28 days in high risk patients, patients undergoing major gynecological surgery, and patients having major surgery for CA • Hip or knee replacement: Apixaban 2.5 mg BID Perioperative Therapy of Patients on Warfarin or Antiplatelet Therapy • Stop Warfarin 5 days before surgery and resume 12 to 24 hours after surgery • For prevention of thromboembolism (MHV, AFib): may require parenteral anticoags perioperatively o Decision to bridge with heparin is determined by balancing r/f bleeding due to the surgical procedure and clotting risk due to the underlying d/o • Low bleeding risk: skin biopsies, cataract surgery, and most dental procedures can remain on warfarin o Dental procedures: tranexamic acid mouthwash can be used w/o interrupting anticoags therapy • Low risk thromboembolic risk (AFib w/o prior stroke or remote Hx of venous thrombosis): warfarin stopped 5 days prior to surgery and resumed that evening of surgery • High thromboembolic risk (PHV): bridging with LMWH or unfractionated heparin may be indicated • Antiplatelets: do not d/c before surgery • Rational Drug Selection • Cost can be significant • PO anticoags are preferred: no special equipment or skills to administer, less expensive • Brands are not interchangeable • Monitoring • Dosage adjustments of warfarin is by INR o Daily INRs initially to guard against excessive anticoagulation until therapeutic range for 2 days o Testing interval is then lengthened to 2 or 3 times weekly for 1 or 2 weeks o Self-testing is increasing in popularity: competency using the equipment o Computer assisted, validated, decision, support tools for warfarin dose regulation: more effective than traditional dosing at maintaining therapeutic INR values o Evidence based dosing tools, patient Edu, follow up, and optimal patient communication: best patient outcomes o CYP 2C9 or VKORC1 gene variants: may require 2 to 4 weeks longer to reach target INR • Monitoring heparin adjustments: aPTT blood tests o Goal of therapy is 1.5 to 2.5 times the control o Platelet and Hct every 2 or 3 days o Thrombocytopenis: tends to occur about 4th day and resolves despite continued heparin therapy o Day 8: thrombocytopenia severe enough to require d/c med o After this time period: periodic testing of platelet and Hct and occult blood in stool o Low doses of SC heparin (5,000 U BID) do not require monitoring r/t does not prolong the aPTT • LMWH: same periodic monitoring of platelet and Hct: likelihood of thrombocytopenia is much less o Test for monitoring: anti factor Xa assay o aPTT may be prolonged: dopes not reliably reflect activity o Routine Xa not recommended: except in pregnant patients • ASA dose for antiplatelet therapy is low to moderate o Serum salicylate level is approximately 100 mcg/mL • Clopidogrel: safety profile similar to ASA, no routine monitoring • Ticlopidine: Severe neutropenia and thrombocytopenia have occurred o CBC, WBC with differential counts every 2 weeks, starting from the second week to the end of the third month of therapy o More frequent monitoring is necessary for patients whose absolute neutrophil counts consistently decline or are 30% lower than baseline o After the first 3 months: CBCs as needed for s/s of infection • Also know: Use of anticoagulants in pregnancy: • Enoxaparin is Pregnancy Category B. • Teratogenicity and fetal death have been reported as well for tinzaparin, although a clear cause-and-effect relationship was not established. • Fondaparinux is also listed as Pregnancy Category B but without adequate or well-controlled studies in pregnancy. • LMWHs do not cross the placenta and do not cause teratogenicity or fetal bleeding • The American College of Chest Physicians recommends LMWH as the first-line drug for women who require antithrombotic therapy during pregnancy: enoxaparin (Lovenox), dalteparin (Fragmin), and tinzaparin (Innohep) • The pharmacokinetics of LMWHs are altered during pregnancy. • LMWH passes in small amounts into breast milk but has low oral bioavailability and may be safely used during breastfeeding Thromboprophylaxis • Prevention and treatment of thromboembolism: o Warfarin is drug of choice for the prevention of venous thrombosis, systemic thrombosis, and pulmonary embolism • Prevention of embolic stroke in AFib: o Low risk for stroke: ASA 75 mg to 325 mg daily o Intermediate risk for stroke: oral anticoags ▪ Anticoags or a combination of ASA and clopidogrel o High risk for stroke: oral anticoags ▪ ACCP recommends: 150 mg of dabigatran BID rather than warfarin o For patients with paroxysmal AFib at high risk for stroke: warfarin target INR between 2 to 3 • Prevention of MI o Several protocols are effective in prevention of MI in patients with and w/o CAD ▪ Over 50 w/o symptomatic CVD: ASA 75 to 100 mg/day ▪ Established CAD (1 year post ACS, with prior revascularization, coronary stenoses greater than 50% by angiogram, and/or evidence for cardiac ischemia on Dx testing): ASA 75 to 100 mg/day or clopidogrel (75 mg/day) ▪ Patients with ACS who have undergone PCI with or w/o stent placement: dual therapy: 90 mg ticagrelor BID plus ASA 75 to 100 mg/day • ACCP: recommends use of ticagrelor over clopidogrel in PCI o Patient who have experienced an MI: anticlot therapy ▪ STEMI: fibrinolytics then ongoing therapy after hospital d/c o Post-AMI TX: o Anterior AMI w/ r/f LV thrombus who do not get a stent: warfarin (INR target 2 to 3) and ASA 75- 100 mg/day o Patients with an acute MI who have a bare metal stent: triple therapy ▪ Warfarin (INR target 2-3), ASA low dose, and 75 mg daily clopidogrel for a month after stent placement ▪ Then warfarin and one antiplatelet drug for months 2 and 3 ▪ After 3 months: d/c warfarin and replaced with dual antiplatelet therapy (90 mg ticagrelor BID plus ASA 75 to 100 mg/day for up to 12 months o Patients with DM: ASA 75 to 162 mg/day o Vorapaxar: approved for reduction of thrombotic CV events in patients with Hx of MI or with PAD ▪ Patient takes one 2.08 mg tablet daily in addition to ASA or clopidogrel • Prevention of Post-Operative Thromboembolism o J Co requires that all hospital have a formal active strategy to prevent venous thrombosis o LMWH, low dose heparin, or fondaparinux ▪ Moderate-risk major general surgery, higher risk patients who are having a major procedure for CA, major vascular surgery, major gynecological surgery, major urological procedures, bariatric surgery, thoracic surgery, and many orthopedic surgeries o Length of Tx: determined by the type of surgery, ranging from a single dose to 10 days for most orthopedic procedures, to 28 days in high risk patients, patients undergoing major gynecological surgery, and patients having major surgery for CA o Hip or knee replacement: Apixaban 2.5 mg BID • The ACCP recommends the use of LMWH, low-dose heparin, or fondaparinux as thromboprophylaxis in the following surgical procedures: moderate-risk major general surgery, higher-risk patients who are having a major procedure for cancer, major vascular surgery, major gynecological surgery, major urological procedures, bariatric surgery, thoracic surgery, and many orthopedic surgeries • LMWH should not be used for thromboprophylaxis in patients with mechanical prosthetic heart valves, especially pregnant women, as prosthetic heart valve thrombosis may occur. Population needed warfarin starting dose adjustment: Older than 75 years • Multiple comorbid conditions • Poor nutrition (low albumin) • Elevated INR when off warfarin • Elevated liver function tests • Changing thyroid status INR target goals o Prevention of DVT, PE, or systemic embolism: 2-3 o Prevention of embolic stoke in patient with AFib: 2-3 o Patients with St. Jude Medical bileaflet PHV: 2-3 o Patients with tilting disk valves and bileaflet PHV ion aortic position: 2.5-3.5 o Patients with CarboMedics bileaflet valve or Medtronic Hall tilting disk PHV: 2-3 o Patients with MHV and high risk (atrial thrombus, AFib, hypercoagulable state, and low EF): 2.5-3.5 o Patients with caged ball or caged disk PHV: 2.5 to 3.5 o Patients with bioprosthetic valve in mitral position: 2-3 o Patients with bioprosthetic vale in aortic position: 2-3 Anticoagulant preferred for patients at high risk for stroke with atrial fibrillation: • High risk for stroke: oral anticoags o ACCP recommends: 150 mg of dabigatran BID rather than warfarin • For patients with paroxysmal AFib at high risk for stroke: warfarin target INR between 2 to 3 o than 80, less than 60 kg, or crt >1.5 dose at 2.5 mg BID Drugs which decrease anticoagulant medication effectiveness: o Heparin: NTG decreases effect of heparin o Warfarin: oral contraceptives, barbiturates, carbamazepine, chlordiazepoxide, cholestyramine, dicloxacillin, griseofulvin, nafcillin, rifampin, flu vaccine, foods high in vitamin K, large amounts of avocado, soy milk, ginseng o Clopidogrel: phenytoin, tolbutamide, tamoxifen, torsemide, Fluvastatin, NSAIDS Chapter 19: Drugs Affecting the Immune System • Know the pharmacodynamics, pharmacotherapeutics clinical use, drug interactions and adverse drug reactions for: Immunizations Immunizations • Frequently encountered in primary care: Prevent the spread of infectious diseases • Single best technique for preventing infectious disease Attenuated Vaccines Influenza Live, Attenuated Influenza Vaccine • Pharmacodynamics o LAIV: included strains dependent upon predicted circulating influenza strains • Pharmacotherapeutics o Contraindicated: o Egg or egg product hypersensitivity, asthma, reactive airway disease, or other chronic d/o of the pulmonary or CV systems o Persons with underlying medical conditions: DM, renal dysfunction, and hemoglobinopathies o Known or suspected immunodeficiency diseases or immunosuppressive therapy ▪ Patients who are immune compromised or with HIV should not be vaccinated with LAIV o Do not administer to those with Guillain-Barre syndrome o Preg Cat C: should not be vaccinated with LAIV o Contraindicated: children under 2 y/o r/t increased incidence of RAD and asthma o Contraindicated in children or adolescents receiving ASA or other salicylates r/t Reye syndrome • ADRs o Usually well tolerated: mild and transient ADRs o Adults: reported resp symptoms: runny nose or nasal congestion, HA, sore throat, cough, and muscle aches o No serious ADRs have been reported in children older than 2 or adults • Drug Interactions o Wait 48 hrs after stopping antivirals to receiving LAIV o May be concurrently administered with MMR and varicella o May be given with any of the inactivated vaccines ▪ May be given with inactivated or live vaccines on the same day if not separate vaccines by 4 weeks • Clinical Use and Dosing o Children and adults aged 9 through 49: 0.2 mL LAIV o Children 5 to 8: w/o prior flu immunization: two doses (0.2 mL each) separated by 4 weeks o Children 8 years and younger who have previously received two doses in the same year need only one day annually o Intranasally: half of drug in each nostril • Monitoring and Patient Education o Lab monitoring not necessary o Live virus: stay away from close contact with immunocompromised persons for 7 days after admin o Report serious or moderate reactions: difficulty breathing, wheezing, hives, swelling, unusual weakness, and temp 38.9C or higher Measles, Mumps, and Rubella Vaccine (MMR) ▪ Pharmacodynamics o Stimulates the immune system to produce antibodies by inducing a subclinical infection with attenuated virus particles o Subclinical infection is not contagious o The antibodies are capable of virus neutralization by complement activation, induction of cell mediated immunity , and opsonization o M-R-Vax II: commonly used combo vaccine also ProQuad ▪ Pharmacotherapeutics o The virus has been detected 1 to 4 weeks in the pharynx or nose after vaccine administration: does not appear to cause virus transmission o Relatively few true contraindications o Previous anaphylactic reaction to the MMR, neomycin, or gelatin ▪ Hx of contact dermatitis to neomycin is not a contraindication o Not recommended for immunocompromised patients ▪ Do not give to patients who are severely immunocompromised r/t CA, leukemia, or lymphomas or on immunosuppressive drug therapy (high dose steroids or radiation) o May be given to close contacts of immune suppressed patients (health care workers) o Deferred if a patient has moderate or severe febrile illness ▪ Minor illnesses with or w/o fever (diarrhea, URI, or OM) are not contraindications o Patients who receive blood products should wait 3 months before getting the MMR o MMR should be given at least 2 weeks before immune globulin (IG) ▪ Those who receive IG should wait 3 to 11 months o Do not give to pregnant women or women who may become pregnant within 3 months ▪ r/t congenital rubella syndrome in the infant o Safely given to children of all given ▪ May not be immunogenic in infants under 12 months ▪ If given under age 12 months, revaccinate at 12 to 15 months, and a third dose at 4 to 6 y/o ▪ ADRs o Children: fever 7 to 12 days after vaccination, lasts 1 to 2 days otherwise asymptomatic o May cause transient maculopapular rash 7 to 10 days o May cause febrile seizure: highest risk at 6 to 14 days o Thrombocytopenia: rare and may occur within 2 month, generally benign and transient o Injection site pain, redness and swelling, muscle or joint aches, and dizziness or light headedness ▪ Drug Interactions o Do not give to patients receiving immunosuppressants (high dose corticosteroid, interferon, and antineoplastic drugs) r/t insufficient response to immunization o MMR may be inactivated by IG: give 14 to 30 days before or 3 months after o Delay PPD for 4 to 6 weeks after MMR: temporary decrease in tuberculin skin sensitivity o MMR and varicella: compatible if done the same day (different needles, separate sites) ▪ If not given the same day: separate by 1 month o MMRV (ProQuad): measles, mumps, rubella, and varicella in children 12 months to 12 y/o ▪ First dose separate MMR and varicella vaccines r/t risk of febrile seizures with MMRV o MMR can be given with LAIV on the same day, if not 4 weeks ▪ Clinical Use and Dosing o Routinely given SC at 12 to 15 months with repeat dose at age 4 to 6 y/o o Second dose: may be given as soon as 4 weeks after the first dose o Indicated ruing an epidemic or before international travel o CDC recommends second dose as MMRV o Adults: born in 1957 or later who are at least 18 should receive at least one dose of MMR if no serological proof of immunity or documentation of a dose given on or after the patient’s first birthday o High risk groups: students entering college, military, and international travelers, healthcare workers: two doses o Adults born before 1957 are considered immune but documentation of immunity is important ▪ Monitoring and Patient Education o Labs not needed o Rubella titer: to determine immunity o All patients or the parents/guardians: required by law to receive Vaccine Information Statements that are developed by the CDC o Provide adequate information to the patient before immunization should be made o Fever up to 103 7 to 12 days after admin o Patient may experience rash, malaise, or sore throat Measles, Mumps, Rubella, and Varicella Vaccine o Pharmacodynamics o Similar to MMR o MMRV (ProQuad): live attenuated virus, produces a subclinical infection o Creates active immunity to measles, mumps, rubella, and varicella o Pharmacotherapeutics o Same as for MMR: Hx of anaphylactic reaction to neomycin or gelatin or any other component of the vaccine ▪ Immediate Tx for anaphylaxis reaction should be available when administering MMRV o Contraindication: primary or acquired immunodeficiency (HIV/ADIS and patients with blood dyscrasias, leukemia, lymphomas, or other malignant neoplasms affecting the bone marrow or lymphatic system o Preg Cat C: do not administer, avoid pregnancy for at least 3 months o Untreated TB or febrile illness with temp greater than 101.3, deferred until well o Use with caution in patient with Hx of cerebral injury, seizures, or other conditions in which physiological stress due to fever should be avoided r/t increased risk of fever and febrile seizures with MMRV o Avoid for 3 months after blood transfusion or IG admin o ADRs o Similar to MMR o More likely to develop fever and rash o Injection site pain, erythema, and swelling o Drug Interactions o Same as MMR: immunosuppressants, IG, tuberculin skin tests, and salicylates o Clinical Use and Dosing o Given SC at 12 to 15 months o First dose may be given any time before age 12 o Three months should elapse before second dose of MMRV o MMRV: can be given 1 month after MMR o Patient Education o Talk with parents about increases risk of fever and febrile seizures with the combo vaccine Oral Poliovirus Vaccine (No longer used in the US but used in other countries) o Pharmacodynamics o Stimulates immune system to produce anti-poliovirus antibodies: Sabin poliovirus types 1, 2, and 3 o Live attenuated virus enters small intestine: replicates in villous epithelial cells o Specialized epithelial cells transport the viral antigens to the B cells and macrophages: produce antibodies in 1 to 2 weeks o Live attenuated poliovirus lingers in GI for 4 to 6 weeks o Two doses of PPV are needed for intestinal immunity ▪ Three doses for lifelong immunity o Pharmacotherapeutics o Contraindication: anaphylactic reaction to any previous dose of OPV o Do not use with neomycin or streptomycin hypersensitivity o Delay with moderate or severe febrile illness or severe resp infection o Contraindicated: viral GI infection, ongoing diarrhea, or vomiting o Immunocompromised patients may develop poliomyelitis from live poliovirus o Contraindicated: CA, leukemia, lymphoma, radiation therapy, immunodeficiency (HIV or AIDS) or drugs that effect immunity ▪ Use IVP in immune compromised patients and household members are immune compromised o Pregnancy Cat: C o ADRs o Low incidence of paralytic poliomyelitis (those who receive it and household contacts) o Most likely to occur after the first dose o Household members are exposed r/t virus shedding in feces o Drug Interactions o Do not administer to patients receiving immunosuppressants, r/t insufficient response to immunization ▪ Use IPV in these patients o MMR, MMRV, hep B, DTP, DTaP, influenza, and HIB may be given with OPV o Avoid concurrent administration with cholera vaccine, parenteral typhoid vaccine, or plague vaccine r/t increased ADRs o IG may be given with OPV o Clinical Use and Dosing o IPV is first choice for immunization o OPV: use only for special circumstances: mass vaccination campaigns to control outbreaks or in unvaccinated children who will be traveling in less than 4 weeks to endemic or epidemic areas Rotavirus Vaccine o Pharmacodynamics o Leading cause of gastroenteritis in infants and young children worldwide o RotaTeq live oral vaccine: contains five strains (Gi, G2, G3 or G4, P7) o Rotarix: G1, G3, G4, and G9 o Both vaccines are live vaccines that replicate in the small intestine and induce active immunity o Pharmacotherapeutics o Do not administer: ▪ Infants who are or may be potentially immunocompromised (including infants with blood dyscrasias, leukemia, lymphoma, or other malignant neoplasms, infants on immunosuppressive therapy) ▪ Infants with primary and acquired immunodeficiency states (HIV/AIDS) ▪ Infants who have received blood transfusion or blood products in the past 42 days o Delay vaccine with febrile illness except when withholding the vaccine creates greater risk to the patient o Minor URI with low grade fever (under 100.5) not a reason to withhold o Do not administer with acute gastroenteritis until it improves o Rotavirus may shed in stools: prudent to use caution if in contact with those with malignancies or otherwise immune compromised and contacts on immunosuppressive therapy o Hx of intussusception is a contraindication r/t increased risk of intussusception following the first dose o ADRs o ARF intussusception o RotaTeq: GI ADRs: vomiting, diarrhea o Rotarix: slight incidence of vomiting in the week after admin o Drug Interactions o Can be administered concurrently with DTaP, IPV, HIB, HBV, and pneumococcal conjugate vaccine o Clinical Use and Dosing o RotaTeq: series of three ready to use oral liquid doses ▪ 6 to 12 weeks first dose, second and third doses in 4 to 10 week intervals o Rotarix: two dose schedule of 1 mL/dose given at 2 to 4 months o May be given between 6 to 24 weeks, give the two doses 4 weeks apart o No considerations for blood products or IG Varicella Virus Vaccine o Pharmacodynamics o Varivax: live vaccine, produces IgG antibody humoral immune response to VZV o Also have cell mediated immune response: activation of CD41 helper T cells and CD 81 T lymphocytes o Efficacy is measured by protection against disease and protection against severe disease ▪ One dose: 94% over 10 years ▪ Two doses: 98% effective against any disease and 96% effective against household exposure o Vaccination appears to prevent serious illness even in those who do not seroconvert ▪ Post vaccination cases are mild, and recovery is quicker o Pharmacotherapeutics o Contraindicated: neomycin or gelatin hypersensitivity o Delay vaccine if moderate or severe illness present with or w/o fever o Immune compromised individuals: ARF development of varicella from use of live virus o Contraindicated: CA, leukemia, lymphoma, radiation therapy, and immunodeficiency, symptomatic HIV o Contraindicated: drugs that affect the immune system ▪ Those on low dose systemic steroids (asthma) if less than 2 mg/kg or less than 20 mg/day of prednisone and not otherwise immune compromised o May be given to a patient if an immune compromised person in the household o If a rash develops: avoid contact with immune compromised people for the duration of the rash o Avoid in pregnancy: Category C o Avoid pregnancy for 1 to 3 months o ADRs o Children and adults: fever, injection site reaction, and a vesicular rash o Adults and adolescents: two injections and have similar ADRs ▪ Fever, injection site reaction, generalized vesicular rash o Drug Interactions o Do not give to those taking immunosuppressants: r/t insufficient immunization o Avoid use during IG therapy, do not use for 3 to 11 months o Check for need to revaccinate if taken close together o MMR and varicella are compatible: given the same day, separate by 1 month if not o Can be given simultaneously with DTaP, DT, Td, HIB, IPV, OPV, PCV13, or HBV o Avoid ASA for 6 weeks after vaccination r/t Reye syndrome o Clinical Use and Dosing o Vaccinate at 12 to 15 months: all healthy children ▪ Second dose at 4 to 6 y/o (before kindergarten) ▪ Separate first and second dose by at least 3 months o Healthy adolescents age 13 and older w/o Hx of infection who have not been vaccinated: two doses 4 to 8 weeks apart o If a child is accidentally administered zoster vaccine (Zostavax) instead of varicella vaccine (Varivax): should be counted as a valid vaccine and reported to VAERS whether ADRs occur ▪ Child will still require two doses of varicella vaccine o Adults: w/o immunity should receive the vaccine ▪ Adults at high risk of exposure: live with children, live or work in an environment in which transmission is likely (teachers, healthcare workers, day care workers) or could occur (college dorm, correctional institution, military) or immune compromised household member ▪ Nonpregnant women of childbearing age, and international travelers o Adults should receive two doses: 4 to 8 weeks apart o Patient Education o ARF fever, injection site reaction, or vesicular rash Zoster Vaccine o Pharmacodynamics o Shingles is a localized and painful cutaneous eruption caused by reactivation of VZV o Latent in neuronal cell bodies and when reactivated causes shingles o Zoster vaccine (Zostavax): live attenuated VZV o Reduces risk of shingles by 50% and 66% effective in preventing post herpetic neuralgia o Reduces severity of zoster o Approved for adults aged 50 years and older o Most effective in patients aged 60 to 69 and least effective in those over age 80 o Pharmacotherapeutics o Contraindicated: Hx of anaphylactic reaction to neomycin, gelatin, or vaccine components o Do not give to those with primary immunodeficiency states o Do not give to those with leukemia, lymphoma, CA of the bone marrow or lymphatic system or AIDS r/t development of disseminated disease o Do not take while actively receiving high dose corticosteroids (prednisone), wait a month after stopping o Not approved for women of childbearing age and should not be used in pregnancy o Do not give to those with active TB or acute illness, or with fever o ADRs o Injection site pain/tenderness, erythema, swelling, and pruritis; HA o Drug Interactions o Stop antiviral drugs (acyclovir, famciclovir, and valacyclovir) 24 hours before vaccine admin o Do not use antiviral for at least 14 days after vaccination o Can be given before, after, or at the same time as blood products or antibody containing products o Stop long term/high dose prednisone for 1 month before vaccination o Short term or low dose may be vaccinated o Methotrexate low doses, azathioprine, or 6 mercaptopurine for Tx of RA, psoriasis, polymyositis, sarcoidosis, or IBD may be vaccinated with zoster vaccine o Clinical Use and Dosing o A single dose of zoster vaccine is recommended for all adults aged 60 years or older o SC tissue of deltoid o Can be given with Hx of herpes zoster o Can be given with Td, Tdap, and pneumococcal o Separate zoster vaccine and other live virus vaccines by 4 weeks o Zoster vaccine is not recommended for those who received the varicella vaccine o Patients with chronic medical conditions (CRF, DM, RA, and chronic pulmonary disease) may be vaccinated o Defer in severe illness or with fever o Can be given in minor acute illness w/o fever (URI) o Immunosuppressed: greater morbidity and mortality from herpes zoster o Give at least 14 days prior to immune suppressive therapy o Avoid for at least 24 months after stem cell transplant o Avoid antiviral meds for 14 days after vaccination o If a child is accidentally given zoster vaccine: valid vaccine, report to VAERS o If an adult is given varicella vaccine: not a valid dose and the zoster vaccine is given the same day o If not the same day 28 days later Oral Typhoid Vaccine Pharmacodynamics o Used to increase resistance to enteric fever caused by Salmonella typhi o Spread by ingesting water contaminated by feces from infected persons o Risk is greatest in travelers to South Asian, East and Southeast Asia, Africa, Caribbean, and Central and South America o OTV (Vivotif Berna): live attenuated vaccine Ty21a o Ingested and works in the small intestine to synthesize a lipopolysaccharide that evokes a protective immune response o 50 to 80% effective in preventing typhoid fever o Efficacy of protective immunity: dependent upon the size of the bacterial inoculum consumed Pharmacotherapeutics o Contraindication: hypersensitivity to typhoid vaccine o Do not give to immune compromised patients o Do not given during acute febrile illness or acute GI illness o Preg Cat C: if vaccine is needed use inactivated vaccine o Not recommended for children younger than 6 y/o: use inactivated vaccine in young children ADRs o Infrequent and transient and resolve with intervention o ABD pain, diarrhea, vomiting, fever, HA, and rash have been reported Drug Interactions o Mefloquine (Lariam) (antimalaria drug): can inhibit the growth of the live Ty21a strain: given either 24 hours before or after mefloquine o Immunosuppressants: insufficient response to the vaccine o OTV: do not given while receiving sulfonamides and ABX r/t prevention of sufficient degree of multiplication to induce a protective immune response Clinical Use and Dosing o Used for primary immunization against S. typhi in the following o Travelers to areas of increased risk o Household contact with a documented typhoid fever carrier o Lab workers o In patients over 6 y/o: dose is one capsule on alternative days for a total of four doses o Take 1 hour before meals (empty stomach) with a glass of cold water o Swallow whole o All four doses must be taken at least 1 week prior to travel Yellow Fever Vaccine Pharmacodynamics o Viral illness spread by some species of mosquitoes in Central and South America and in tropical regions of Africa o Endemic in sub Saharan African and tropical South America o Produces a hemorrhagic fever and is fatal in 20 to 50% of severe cases o All travelers should use personal protective measures to avoid mosquito bites o Vaccination is recommended for travel to endemic areas o Refer to current CDC guidelines for endemic areas because they can change o Certification of vaccination may be required to travel into some areas o Yellow fever vaccine (YF-Vax): live attenuated virus that is prepared by culturing the 17D strain virus in a living chick embryo Pharmacotherapeutics o Avoid in patient with Hx of egg hypersensitivity or sensitivity to chicken protein or gelatin o Contraindicated in immune compromised o Use with caution in HIV patients o Contraindicated: thymus d/o that affects immune function o Defer vaccination for 8 weeks following blood or plasma transfusion o Pregnancy Category C: use only if exposure risk is high o Contraindicated: infants younger than 6 months r/t increased r/f vaccine associated neurological disease o Use with caution in infants aged 6 to 8 months o Rare cases of encephalitis have occurred in infants at this age o Use with caution: aged 60 years and older r/t increased r/f serious ADRs after vaccination ADRs o Fever or malaise (7 to 14 days after vax) o Myalgia or HA o Anaphylaxis has occurred: keep epi on hand when given the vaccine o Very rare reaction: yellow fever vaccine associated viscerotropic disease: fever and multiple organ failure o Higher among those 60 and older o Case fatality ration is 53% o Yellow fever vaccine associated neurological disease: conglomerate of clinical syndromes including meningoencephalitis, GBS, encephalomyelitis, bulbar palsy, and Bell’s palsy o Adverse neuro outcomes are usually seen among infants as encephalitis but may occur at any age Drug Interactions o Concurrent vaccination with HAV, HBV, meningococcal vaccine, typhoid fever vaccine, DTaP, Tdap and measles vaccines is ok o Immunosuppressants: insufficient response to the yellow fever vaccine Clinical Use and Dosing o Immunization against yellow fever is recommended for all people over age 9 months who are living in or traveling to endemic areas o Dose is a single 0.5 mL dose given SC o Should be repeated every 10 years Bacillus Calmette-Guerin Vaccine Pharmacodynamics o BCG lowers the risk of serious complications of primary TB in children o Not widely used in the US but used in countries where TB is endemic o BCG stimulates natural infection with My tuberculosis and results in a T cell medicated immune reaction and immunity against TB but with variable degrees of protection o The protective effect of BCG used in children against military and meningeal TB is about 80% o It is less effective in adults Pharmacotherapeutics o Do not use in Active TB o PPD skin testing should be performed on all patients over 2 months of age o Contraindicated in CA, leukemia, lymphoma, radiation therapy, and immunodeficiency o Patients with symptomatic or asymptomatic HIV should not receive BCG o Drugs that affect the immune system: contraindicated o Take precautions to avoid accidental exposure to BCG solutions during preparation and administration because this is a live attenuated virus o Pregnancy Cat C: CDC does not recommend use ADRs o Normal reaction: skin lesions that appear within 10 to 14 days after the multiple puncture disc application o The lesions consist of small red papules at the site of admin o Papules reach max diameter (3 mm) after 4 to 6 weeks and then scale away and slowly subside o Lymphadenopathy: may occur in a regional lymph node and resolves spontaneously o Osteomyelitis: rare o BCG induced osteomyelitis: affects the epiphyses of the long bones and can occur from 4 months to 2 years after administration o Lupoid like skin reactions: Tx with INH for 3 months o Disseminated BCG infection and death are very rare and usually occurs in patients with impaired immune systems Drug Interactions o Anti-TB agents (RIF, INH, streptomycin) and immunosuppressives: interfere with development of an appropriate immune response o BCG: will cause PPD skin tests to give false positive readings for up to 10 years, after 10 years: a positive PPD usually indicates infection with M. Tuberculosis Clinical Use and Dosing o In the US: given only in very special circumstances: unavoidable risk of exposure to M. tuberculosis and failure of other methods of prevention and control of TB o Should be considered for infants and children who reside in settings in which the likelihood of TB transmission and subsequent infection is high provided no other measures can be implemented (removing children from the source of infection) o May be considered for health care workers who are employed in settings in which the likelihood of transmission and subsequent infection with TB strains resistant to INH and RIF is high o BCG: given to healthy infants from birth to 2 months w/o TB skin testing o After that, BCG is given only to children with negative Mantoux skin tests o Must be administered exactly as manufacturer suggests o Vaccine is dropped onto clean, dry skin over the deltoid muscle and spread over the area to be punctured, using the edge of the multipuncture disc o The prongs of the disc are coated with the virus by lightly dipping them into the spread vaccine o The prongs of the disc are pressing into the skin and held for 5 to 10 seconds o After the disc is removed: vaccine is respread to fill all the puncture areas o Additional vaccine may be applied to ensure a wet vaccine site o Keep the area dry for 24 hours, no dressing is required o The person administering: take precautions against coming into contact with the live virus o Educate not to touch the site r/t live virus o Keep clean until local reaction has resolved Inactivated Vaccines Diphtheria, Tetanus, and Pertussis Vaccine Pharmacodynamics o Various combinations of diphtheria, tetanus, and pertussis vaccines: basic pharmacodynamics are the same o Diphtheria toxoid: induces the production of antibodies against the exotoxin excreted by Corynebacterium diphtheria o Complete immunization (four doses, then boosters every 10 years) reduces incidence by 95% o Immunized persons who develop diphtheria have milder illness o Infection with C. diphtheria does not confer immunity: those previously infected should vax o Tetanus toxoid: contains antigens that induce the production of antibodies against the exotoxin excreted by Clostridium tetani o Duration of immunity: 10 years o Natural immunity to C. tetani does not occur in the US and patients with previous C. tetani infection should be vaccinated o Pertussis vaccine: inactivated pertussis antigens o Acellular pertussis vaccine contains one or more immunogens derived from Bordetella pertussis and unlike whole cell vaccine contains little or no endotoxin o Antibodies produced against B. pertussis Pharmacotherapeutics o In the US: DTaP be used for primary immunization of infants and children o DTaP contraindication: immediate anaphylactic reaction with a pervious dose o Encephalopathy w/in 7 days of a previous dose, unexplained by another cause: do not use ▪ In this case DT should be substituted for DTaP o Patients with unstable, progressive, neurological problems: defer until neuro status clarified o Precautions: previous temp of 105 or higher within 48 hours after a dose, Hx of continuous crying (more than 3 hours) within 48 hours of a dose, convulsions within 3 days of a previous dose, and collapse or shock like state (hypotonic-hyporesponsive episode) within 48 hours of a dose o Precautions: seizures 3 days or less after a previous dose of DTaP, persistent or inconsolable crying lasting more than 3 hours within 48 hours of a pervious dose, GBS less than 6 weeks after a previous dose of tetanus toxoid containing vaccine, or moderate or severe acute illness with or w/o fever o DTaP: may be given to immunocompromised patients or patients on immune suppressive drugs: immune response may be less than optimal o HIV infected patients may be immunized o Patients with minor acute or febrile illness may be immunized ▪ Delay in cases of moderate or severe illness, with or w/o fever o Infants born prematurely should begin series based on their DOB: first vaccine at 2 months o Pregnant women, including pregnant adolescents, receive a booster of Tdap with each pregnancy ideally between 27 and 36 weeks gestation: passive pertussis immunity, gives 2 to 3 months of protection o Td and Tdap are Pregnancy Cat C: but have been used extensively worldwide in pregnant women o DT contraindications: patients older than age 7, who should be given Td or Tdap, as appropriate o DT and Td: hypersensitivity to any component of the vaccine and moderate to severe illness, with or w/o fever: do not postpone for minor illnesses o Tdap contraindications: GBS 6 weeks or sooner after previous use of tetanus toxoid containing vaccine, previous arthus reaction after receiving tetanus or diphtheria vaccine, or a progressive neurological d/o o Tdap and Td should be deferred with moderate or severe illness, with or w/o fever ADRs o Injection site reactions of mild to moderate pain, erythema, swelling, and induration for a few days o Transient low grad fever, chills, malaise, generalized aches and pain, and HA may occur o Fever was common after DPT and less common after DTaP o Drowsiness, fretfulness, and GI upset may occur o Tdap: most frequent ADR pain at injection site o Seizures may occur and are more likely in children with a Hx of seizures: less common now that DPT is no longer used in the US o Seizures may be r/t fever: antipyretic prophylaxis after DTaP admin every 4 to 6 hrs Drug Interactions o Decreased immunological response: radiation, antineoplastic agents, or immune suppressive agents o DTaP, DT, Tdap, or Td may be coadministered with HBV, HIB, meningococcal, influenza, hep A, and pneumococcal vaccines o Do not co administer: cholera vaccine, typhoid vaccine, or plague vaccine r/t accentuated ADRs Clinical Use and Dosing o DTaP: routinely given at 2, 4, 6, and 15 to 18 months and 4 to 6 years old o Tdap: recommended at age 11 to 12 y/o o Patients should receive a booster dose of Td or Tdap Q 10 years o “Cocooning”: Parents, siblings, grandparents, childcare providers, and health care personnel who care for infants younger than 12 months receive Tdap vaccine to protect the infant against pertussis Haemophilus B Conjugate Vaccine Pharmacodynamics o HIB conjugate vaccine: consist of the HIB capsular polysaccharide covalently linked to another antigen to increase immunogenicity o Stimulates the immune system to produce antibodies that destroy the capsule: making the organism vulnerable to antibody and cell mediated immunity o Unconjugated capsule polysaccharide vaccines cause B cell stimulation only o Conjugation: T cell stimulation occurs as well o HIB conjugate vaccine comes singly and in combo with other vaccines (HBV/HIB, DTaP/HIB, DTaP/IPV/HIB, meningococcal/HIB Pharmacotherapeutics o Anaphylactic reaction to the vaccine or any component is contraindicated o Moderate to severe illness with or w/o fever may be reason to delay vaccination o Minor illness is not a reason to delay admin o HIB should only be given to children under the age of 6 ADRs o Pain, redness, and swelling at the injection site Drug Interactions o No known interactions Clinical Use and Dosing o Dosing depends on the vaccine used o The first dose can be given at 6 weeks but no earlier Inactivated Poliovirus Vaccine Pharmacodynamics o Parenteral noninfectious suspension of three types of inactivated poliovirus o All US poliovirus vaccines as inactivated polio: reduce risk of vaccine associated polio Pharmacotherapeutics o Hx of immediate hypersensitivity reaction after receiving IPV is a contraindication o Neomycin, streptomycin, or polymyxin B hypersensitivity should not receive the vaccine o IPV is preferred over OPV in immune suppressed patients however a protective immune response is not guaranteed o IPV can be administered to patients with HIV o IPV may be given in pregnancy: Category C o Can be used in infants as young as 6 weeks old ADRs o Injection site reaction o Systemic reactions are infrequent Drug Interactions o Immune response to IPV may be diminished by immunosuppressant medications o Revaccinate 3 months after d/c immunosuppressants o Can be coadministered with all other childhood vaccines Clinical Use and Dosing o Primary series of IPV is four doses given at 2, 4, and 6 to 18 months and 4 to 6 years o Four weeks between doses one and two and 2 to 3 o Minimal interval between dose three and four is 6 months Hepatitis B Virus Vaccine Pharmacodynamics o HBV is produced by recombinant DNA from common baker’s yeast that is genetically modified to synthesize HbsAg o Active immunization with HBV stimulates the immune system to produce anti-hepatitis B surface antigen antibodies Pharmacotherapeutics o Contraindication: hypersensitivity to yeast or other components of the vaccine o Moderate or severe illness with or w/o fever is a contraindication to HBV o Patients with renal disease requiring hemodialysis or patients with immunosuppression may require larger doses to achieve adequate serum levels of anti HBs o Pregnancy Cat C: CDC has stated that pregnancy is not a contraindication ADRs o Injection site reactions o Systemic complaints: fatigue, weakness, malaise, fever, HA, NVD, and pharyngitis o Serum sickness has occurred days to weeks after admin o Alopecia: a very rare side effect Drug Interactions o Immunosuppressants and antineoplastic agents may require larger doses or additional doses Clinical Use and Dosing o Vaccination with HBV is recommended for all ages particularly those at high risk of contracting hep B o IV drug users, infants born to mothers who are HbsAg positive, hemodialysis patients, sexually active people with multiple partners, incarcerated people, international travelers, household contacts of hep B carriers, and sexual contacts of hep B carriers o Patients getting tattoos, share razors, toothbrushes, or body piercing jewelry are also at risk o Healthcare workers, day care workers, and other people exposed to body fluids are at risk o Patients with DM are at increased risk and it is recommended they receive the HBV series o Universal vaccination of all infants as a comprehensive strategy to control hep B o Current childhood immunization: three dose HBV series to newborns or at age 11 to 12 o The series can be started at any age o Schedule for vaccinating infants: first dose at birth or before age 2 months, second dose at age 1 to 4 months, dose 3 at 6 to 18 months o The series can be completed in as little as four months o Series is never restarted: no matter how long since the previous dose Hepatitis A Virus Vaccine Pharmacodynamics • HAV vaccine is used to confer immunity to hep A in people at risk of contracting the disease • Stimulation of specific antibodies takes place w/o producing disease symptoms • Antibody titers after HAV are lower than those resulting from Hep A infection • Antibody titer of 20 mIU/mL is protective • Two products available: both have a two dose schedule Pharmacotherapeutics • Contraindication: Hx of severe reaction to HAV • Contraindication: Moderate or severe illness, w/ or w/o fever • Can be given to immunosuppressed but may have lower antibody titers • Pregnancy Category C: CDC has stated that HAV may be given in pregnancy if indicated and poses no risk to the fetus • Safety and effectiveness in children under 12 months have not been established ADRs • Soreness at the injection site, HA, malaise Drug Interactions • Decreased immunological response while taking immunosuppressants or antineoplastic agents Clinical Use and Dosing • Pre-exposure protection from hep A in adults and children • Recommended that all children begin HAV series at 12 months • Those at increased risk: o Traveling or working in endemic regions, over 12 months (S. America, Africa, Greenland, and Asia) o Men who have sex with men o Illegal drug users o Occupational risk for infection (research lab) o People with clotting factor d/os o People with chronic liver disease o Household members and close personal contact of internationally adopted children from endemic countries (start first dose as soon as the adoption planning process begins: at least 2 weeks before arrival • Two products available: HAVrix and VAQTA also combo products (HAV and HBV) • HAVrix: Two dose schedule 6 to 12 months apart • VAQTA: Adults 6 months apart, children 6 to 18 months apart • IM into deltoid: adults and children 3 y/o and older o Vastus lateralis: 12 months to 36 months old o Injection into gluteal region results in suboptimal response • Those with impaired immunity may require additional doses to obtain adequate response o Post immunization testing for those that are immune compromised, not for healthy adults r/t high seroconversion rates in both children and adults Human Papillomavirus Vaccine Pharmacodynamics • HPV causes cervical CA: HPV-16 and HPV 18 cause neoplasia • Two vaccines: o Gardisil (quadrivalent HPV recombinant vaccine for types 6, 11, 16, and 18) o Gardisil 9 (types 16, 18, 31, 33, 45, 52, and 58 Pharmacotherapeutics • Contraindication: severe allergic reactions • Contraindication: yeast allergy • Both vaccines: syncope, observe for 15 mins after admin put in supine or Trendelenburg is symptomatic • Preg Cat B: not recommended for use in pregnancy ADRs • Pain, redness, and swelling at injection site • Fatigue, HA, myalgias, and arthralgias • Serious reaction: syncopal episodes: reports of falling w/ injury Drug Interactions • No known interactions • Immunosuppressants may reduce immune response to the vaccine Clinical Use and Dosing • Dosing varies with the drug and patient gender • Gardasil: girls and women ages 9 to 26 y/o to prevent cervical, vulvar, and vaginal CA caused by HPV types 16, 18, and genital warts caused by HPV types 6 and 11 • Gardasil 9: girls and women ages 9 to 26 y/o to prevent cervical, vulvar, and vaginal CA caused by HPV types 16, 18, 33, 45, 52, and 58; genital warts caused by HPV 6 and 11. • Vax is recommended for all girls starting at age 11 to 12 y/o but may be started as young as 9 y/o • Dosing schedule: three doses of 0.5 mL IM 0, 2, and 6 months • Gardasil: approved for prevention of genital warts types 6 and 11 in males • Gardasil 9: approved for anal warts caused by 16, 18, 31, 33, 45, 52 and 58 and genital warts caused by 6 and 11 • In males 0.5 mL at 0, 2, and 6 months • Cervarix: approved to prevent cervical CA caused by types 16 and 18 in females aged 10 to 25 y/o, 0, 1, and 6 months Influenza Vaccine Pharmacodynamics • Influenza vaccine (Afluria, Fluarix, Flucelvax, FluLaval, Fluvirin, Fluzone, Fluzone Intradermal, Fluzone High-dose) • Multivalent vaccine that contains three different viral subtypes • Available as a trivalent or quadrivalent (Fluarix, Quadrivalent, Flulaval Quadrivalent, Fluzone Quadrivalent) • Flucelvax (trivalent): first cell based flu vaccine • FluBlok (trivalent recombinant hemagluten (HA) flu vaccine containing purified HA proteins: uses cell based technology • The WHO tells the FDA which strains to include for the year, flu virus consistently changes • Stimulates production of antibodies specific to the disease strain Pharmacotherapeutics • Contraindication: anaphylactic reaction to the flu vaccine, eggs, or egg products • CDC developed an algorithm to be used in those who report egg allergy o Recommendation: admin inactivated flu vaccine to patients who can eat lightly cooked eggs w/o a reaction o For those 18 to 49 who experience hives from eating eggs: admin recombinant flu vaccine (Flucelvax, FluBlok) or inactivated flu vaccine and observe for at least 30 mins o Patients with severe hypersensitivity to eggs: only admin the recombinant flu vaccine: produced using non egg technology • Thimerosal: used as a preservative, hypersensitivity contraindicates flu vaccine • Some vaccines contain sulfites: hypersensitivity contraindicates, choose a vaccine that does not contain sulfites • Patient w/ active neuro d/o should defer vaccine until stabilized • Any patient w/ Hx of GBS less than 6 weeks after a previous flu vaccine should not receive • HIV: can be immunized but may have lower vaccine induced antibody levels • Defer with acute febrile illness until symptoms subside • Preg Cat C: according to CDC can be safely administered to pregnant women • Safety not established in children younger than 6 months • Admin error: dosing schedules for children and adults for injectable flu vaccine are different • If an adult is given a peds dose the adult should receive an addition peds dose the same day. If not discovered until later, a full adult dose is given o If a child is given an adult dose: no action needed ADRs • Usually mild and more common in children • Local injection site reaction • Low grade fever, malaise, and myalgia • Hypersensitivity reaction: urticaria, angioedema, bronchospasm, and/or anaphylactic shock o Most likely result of hypersensitivity to residual egg protein Drug Interactions • Immunosuppressants or antineoplastic agents: decreased immune response • Theophylline, phenytoin, and warfarin: inhibited clearance, not a contraindication Clinical Use and Dosing • Admin annually to all persons aged 6 months and older (including pregnant women) • Optimal time is October through mid-November • Travelers to endemic flu zone: vax 2 to 4 weeks prior to travel • Dosing schedules vary by age and past immunization (children) Pneumococcal Vaccine Pharmacodynamics • Two types of pneumococcal vaccine: Pneumovax 23 and Prevnar • Polyvalent pneumococcal polysaccharide vaccine (PPV) contains 23 highly purified capsular polysaccharides from St. pneumoniae o These are the 23 most prevalent or invasive pneumococcal types o PPV stimulates immune system to produce pneumococcus capsule specific antibodies o These antibodies destroy the capsule: pneumococcus vulnerable to antibody and cell mediated immunity • Limited immunogenicity in children younger than 2 • Infants and children: 7-valent conjugate vaccine (Prevnar) o Targets the 7 most common strains in infants • 13-valent conjugate (Prevnar 13, PCV13): adults age 50 and older and age 19 and older with immunocompromising condition, functional or anatomic asplenia, cerebrospinal fluid leak, or cochlear implants Pharmacotherapeutics • Previous anaphylactic reaction contraindicates use • Moderate to severe illness, w/ or w/o fever, defer until patient improved • Should be given 10 to 14 days before elective splenectomy, organ transplant, immunosuppressive therapy, or chemo • Patients with Hodgkin’s disease and immune suppressed: suboptimal antibody response • PPV: use cautiously in patients w/ idiopathic thrombocytopenic purpura: associated w/ relapse • PPV: Preg cat C, avoid in first trimester • PCV13: Preg cat B: no adequate studies • PPV 23-valent: no recommended in children under 2 y/o ADRs • PPV: Local injection site reactions erythema, induration, and soreness for up to 48 hrs o Low grade fever (high grade fever is rare) and arthralgia • Prevnar: generally mild injection site reaction, irritability, drowsiness, and decreased appetite o Children: fever • PCV13: infants had redness, swelling and tenderness at injection site o Fever, irritability, and increased sleep after the first dose (same rate as infants vaxed w/ PCV7) Drug Interactions • No known drug interactions have occurred Clinical Use and Dosing • Dosing of PPV is based on age and medical condition o PPV: all adults aged 65 or older. Revax if original was 6 years ago or more and the patient was under age 65 o People aged 2 to 65 y/o w/ chronic illness and increased risk of morbidity or mortality PPV (chronic cardiac or pulmonary disease, chronic liver disease or alcoholism, DM, and CSF leaks) Revax is not recommended in these populations o Immunocompetent patients aged 2 to 65 y/o with asplenia, revax in 5 years or more after the first dose if the patient is under 10 y/o revaccinate in 5 years o Immunocompromised, CRF, hematological malignancy, Hodgkin’s disease, lymphoma, or multiple myeloma, immunosuppressive therapy and patients who have received an organ or bone marrow transplant. Revax in 5 years o Adult smoker between 19 and 64 (one dose) o Asthma 19 to 64 y/o (one dose) o One dose at 12 and 24 months following bone marrow transplant o A dose of {PPV is given 2 or more months after last dose of PCV13 to children age 2 years and older • PCV13 dosing o Infants and toddlers: four doses at 2, 4, 6, and 12-15 months of age o Children up to age 59 months who are not vaccinated with PCV13 at the routine infant vaccine times should be doses following the catch up schedule o Children aged 6 to 18 with asplenia, HIV infection, cochlear implant, CSF leak, or other immune compromising condition w/o previous PCV 13 should receiving one dose of PCV13 followed by PPV 8 weeks later o Immunocompromised adults age 19 and older who have not received pneumococcal vaccine should receive a dose of PCV13 followed by a dose of PPV 8 weeks later o Immune compromised adults age 19 and older who have received PPV should receive a dose of PCV 13 at least one year after last PPV dose o All adults age 65 or older: if not previously vaccinated dose of PCV13 followed by PPV in 6 to 12 months o Persons age 65 and older who have previously received PPV should be given a dose of PCV13 at least a year after the last PPV dose Meningococcal Vaccine Pharmacodynamics • Meningococcal polysaccharide vaccine (MPSV) for groups A, C, Y, and W135 (Menomune A/C/Y/W135) o Approved in the US for meningococcemia and meningitis caused by Neisseria meningitidis serogroups A,C,Y, and W135 • MCV4 (Menactra, Menveo): tetravalent meningococcal conjugate vaccine, provides protection against serogroups A C Y and W 135 • Hib-MenCY (MenHibrix): Neisseria meningitidis serogroups C and Y and H. influenza type b • Trumembra: used in 10 to 25 y/o, serogroup B meningococcal vaccine • Bexero: meningococcal serotype B: 10 to 25 y/os • In the US: N. meningitidis serogroup C Y and W 73% of infections aged 11 y/o or older o and serogroup B for 60% of cases in children 0 to 59 months old • Group A is rare in the US, most common in Africa and Asia, there have been 2 recent group A outbreaks in the US on college campuses • All the meningococcal vaccines induce the formation of bactericidal antibodies to meningococcal antibodies • Limitations of MVP (Menomune) group C polysaccharide is poorly immunogenic among child less than 2 y/o, does not confer long lasting immunity, and it does not cause a sustainable reduction of nasopharyngeal carriage of N meningitidis o Conjugation of polysaccharide (as in MCV4) to a protein carrier changes the immune response from T cell independent to T cell dependent: leads to stronger response to the vaccine and reduction of the asymptomatic carrier state Pharmacotherapeutics • Previous anaphylactic reaction to any component: contraindication • Moderate to severe illness w/ or w/o fever: defer until patient improved • Hx of GBS: precaution for MCV4: ACIP removed GBS as a precaution r/t benefits outweigh risk • Immunosuppressed patients: immune response may not be obtained • Menomune A/C/Y/W135 should not be given to pregnant women o Not recommended for children under age 2 • MCV4 (Menactra): approved for use in children 9 months and older and in adults: o not recommended in adults older than 55 y/o • MCV4 (Menveo): not recommended for children younger than 2 y/o or adults older than 55 y/o • HibMenCy: approved for children aged 6 weeks to 18 months o not approved in older children or adults • MPSV (Menomune): approved for adults older than 55 y/o • Both serotype B vaccines (Bexsero, Trumenba): Preg Cat C o Neither approved for children younger than 10 y/o ADRs • Mild: pain and tenderness at injection site for 1 to 2 days Drug Interactions • No known drug interactions Clinical Use and Dosing • The ACIP recommends routine vaccination of young adolescents (11 to 12 y/o) with MCV4 at the preadolescent health care visitor or on entering high school (age 15) o One booster MCV4 at 16 to 18 y/o to adolescents who received the first dose before age 16 o A catch up dose can be administered to those aged 19 to 21 y/o who have not received a dose after their 16th birthday • HibMenCY: infants aged 2 to 18 months with persistent complement deficiencies or asplenia o May also be used for infants 2 to 18 months during an outbreak of meningitis covered by the vaccine • ACIP: these high risk groups should be vaccinated o Patients with deficiencies in late complement components (C3, C5, to C9) o Persons with functional or actual asplenia o Research, industrial, and clinical laboratory personnel who routinely are exposed to N. meningitidis in solutions that may be aerosolized o Travelers to and residents of hyperendemic areas such as sub Saharan African. Epidemics have occurred recently in Saudi Arabia, Kenya, Tanzania, Burundi, and Mongolia o College freshmen living in dorms o Military recruits • HibMenCY: given as a primary series IM at age 2, 4, 6, and 12-15 months. • MCV4: IM single dose with a second dose in 12 weeks in children aged 9 to 23 months and at 8 to 12 weeks in patients aged 2 to 55 y/o • Menomune for all ages is a single SC dose • In patients aged 9 months to 10 years, MCV4 (Menactra) is the preferred vaccine • Adults over age 55 y/o: MPSV • Revaccination: o indicated for persons at high risk for infection (travel or living in epidemic areas) who were previously vaccinated with MPSV or MCV4 o Children vaccinated between age 2 months and 6 y/o should be revaccinated with Menactra in 3 years if they remain at high risk o If previous dose given at age 7 or older: either brand of MCV4 may be given after 5 years o Revaccination with MCV4 of high risk older children and adults is recommended every 5 years after the first dose of MPSV or MCV4 • A booster dose of MCV4 is recommended at age 16 years in children who received their first dose of MCV4 at age 11 to 12 y/o o Adolescents who receive their first dose of MCV4 at age 13 to 15: booster dose at age 16 to 18 y/o o Minimum interval between doses of MCV4 is 8 weeks • Serotype B vaccine: given to 10 to 18 y/o during an outbreak or to those at increased r/f meningococcal disease o Bexero: two does 1 month apart o Trumenba: three doses 1, 2, and 6 months Lyme Disease Vaccine Pharmacodynamics • Vector born illness caused by ticks infested with Borrelia burgdorferi • Recombinant Lyme disease vaccine (LYMErix, Immulyme): imparts immunity against B. burgdorferi by stimulating production of antibodies to the lipoprotein OspA o OspA: lipoprotein of the B. burgdorferi spirochete • MOA: antibody killing of the spirochete in the tick o Transmission of OspA antibody occurs while the tick is feeding on the blood of an immunized host, the antibodies kill the spirochete before transmission occurs • Production was d/c r/t ADRs of the vaccine o Although an FDA investigation did not find the vaccine to be dangerous • Long term immunity for patients previously vaccinated is unknown • New vaccine against B. burgdorferi is in clinical trials in Austria and Germany Typhoid Vaccine The best protection against typhoid fever is food and water precautions to prevent contracting S. Typhi Pharmacodynamics • Typhoid vaccines (Typhoid Vaccine, Typhim Vi) • Used to increase resistance to enteric fever caused by S. typhi o Efficacy of protective immunity depends on the size of the bacterial inoculum consumed • Two parenteral typhoid vaccines available o Heat and phenol inactivated vaccine (Typhoid Vaccine) o Purified Vi polysaccharide (Typhim Vi): used in the United States Pharmacotherapeutics • Hypersensitivity to typhoid vaccine is contraindication • Do not admin to patients w/ acute febrile illness • Preg Cat C: unknown if harmful to the fetus o If vaccination is necessary: inactivated vaccine is recommended • Typhim Vi and Typherix (Canada): not recommended for children under ag e2 ADRs • Local injection site reactions: erythema, induration, and soreness that begin w/in 24 hours and last 1-2 days • Systemic symptoms: low grade fever, HA, and myalgias o High fever is rare Drug Interactions • Plague vaccine should not be given at the same time as typhoid vaccine r/t avoidance of accentuated ARDs • Immunosuppressants: insufficient response to the vaccine Clinical Use and Dosing • Typhoid vaccine is used for primary immunization against S. typhi infection in the following o Travelers to areas where a risk of exposure to S. typhi is recognized o Persons w/ household contact w/ a documented typhoid fever carrier o Lab workers with frequent contact with s. typhi • Typhim Vi and Typherix in children older than age 2 years and adults is 0.5 mL SC o A booster dose should be given every 2 years to patients at continued risk of contracting typhoid Cholera Vaccine • No used in the US • Immunization for travelers not recommended r/t brief and incomplete immunity that the vaccine offers Japanese Encephalitis Virus Vaccine Pharmacodynamics • Japanese encephalitis (JE): most common form of viral encephalitis in Asian and is spread by mosquitoes • Usually severe resulting in death in 20-30% of cases, miscarriage in pregnant women, and serious neuro outcomes in 30 to 50% of those infected • US: inactivated Vero cell culture derived vaccine (JE-VC) or IXIARO Pharmacotherapeutics • Previous anaphylactic reaction to any component of the vaccine, including thimerosal, is a contraindication • Pregnancy is a contraindication to JE-VC use • IXIARO approved for children age 2 to 16 y/o ADRs • IXIARO: same ADRs as placebo or comparison vaccine (PCV7 or HAV) • Injection site tenderness and fever • Report any significant ADRs to VAERS • ADRs typically appear up to 10 days after immunization • Advise patients not to travel outside of the US for 10 days in case of ADRs Drug Interactions • No known drug interactions Clinical Use and Dosing • Prevention is best way to avoid JE o Wear long sleeved shirts and long pants, use repellents, apply permethrin to clothing • ACIP recommends JE-VC for those who plan on residing for more than a month in areas in which JE is endemic or epidemic o Probability of JE infection and illness increases with the duration of stay in rural endemic areas • Obtain information about locations from CDC Yellow Book • JE-VC: not recommended for all travelers to Asia o Those spending a month or longer in endemic areas during transmission season (rural areas especially) o Also, for those who will be spending extensive time outdoors during their travel • JE-VC in adults: two doses 28 days apart • Children aged 2 months to 2 years two doses • Complete at least 1 week before potential exposure to JEV Plague Vaccine Pharmacodynamics • Plague vaccine, a whole cell vaccine consisting of a suspension of inactivated plague bacilli (Yersinia pestis), no longer available in the US • Travelers who may be exposed to plague should carry prophylactic antibiotics (doxycycline, or trimethoprim- sulfamethoxazole for children under 8 y/o) and use them according to the CDC guidelines for plague Rabies Vaccine Pharmacodynamics • Rabies vaccine (Imovax, RabAvert): preparation of inactivated rabies virus that induces active immunity • Two products available differ only in the cell culture used to develop the vaccine o Imovax uses human diploid cell culture o and RabAvert uses purified chick embryo cell culture Pharmacotherapeutics • Previous anaphylactic reaction to any component of the vaccine, including neomycin, is a contraindication • Moderate to severe illness, w/ or w/o fever is reason to defer until patient improved • Immunosuppressed: expected immune response may not be obtained • Pregnancy Cat C: not a contraindication to post exposure vaccination of pregnant women • Safety in children under 6 y/o not established ADRs • Local reactions: pain, erythema, and swelling of the injection site • Systemic reactions: HA, nausea, ABD pain, muscle aches, and dizziness • Three cases of a neuro illness resembling GBS have been reported • Serum sickness like reaction who received booster dose of Imovax Drug Interactions • Long term therapy with chloroquine (Aralen) can interfere w/ the active antibody response • Immunosuppressants or antineoplastic agents: decreased immunological response to the vaccine • Rabies IG (RIG): can partially suppress the antibody response to rabies vaccine • Follow CEC recommendations for simultaneous administration exactly: give no more than the recommended dose of RIG Clinical Use and Dosing • Rabies vaccine can be given for primary or pre exposure vaccination or as part of a post exposure prophylaxis • Pre exposure vaccination is recommended to high risk groups (vets, animal handlers, and certain lab workers) o Three 1 mL IM injections of vaccine in the deltoid o Doses given on day 0, day 7, and either day 21 or 28 o Booster dose of 1 mL every 2 years to those considered at frequent risk of their serum antibody titer is less than 1:5 o High risk groups: vets, animal control officers, wildlife officers, and staff where rabies is enzootic o Ver high risk: research lab workers or vaccine production facilities: a serum rabies antibody test should be done every 6 months and vaccine administered if levels are less than 1:5 • Post-exposure prophylaxis is recommended if the patient has a bite from a rabid animal that penetrates the skin o Post exposure vaccine admin should always be accompanied by use of RIG o Always includes admin of both passive antibody and vaccine with the exception of those who have previously received complete vaccination o ACIP recommendation: four doses of rabies vaccine ▪ 1 mL given IM on days 0, 3, 7, and 14 ▪ And RIG given on day 0 o For those who have previously been vaccinated two doses of rabies vaccine are given on days 0 and 3, with no RIG needed Differentiate between the types of vaccines (live, attenuated, etc.) • Vaccines are divided into two different types: Attenuated or Inactivated • Attenuated (“modified-live”) o Attenuated vaccines include MMR; oral polio (OPV); varicella virus (Varivax, ProQuad); yellow fever (YF-Vax); live, attenuated virus influenza vaccine (Flumist); rotavirus (Rotarix, RotaTeq); varicella zoster (Zostavax); and bacilli Calmette–Guérin (BCG) • Inactivated (“killed”) infective agents o Inactivated vaccines include diphtheria, tetanus, and pertussis (DTP, DTaP, DT, Td, Tdap), H. influenzae type B (HIB), hepatitis A and B, influenza (Fluzone), meningococcal (Menactra, Menomune, Trumenba, Bexsero), inactivated polio vaccine (IPV), human papillomavirus vaccine (Gardasil, Cervarix), pneumococcal polysaccharide (PPV23), and pneumococcal conjugate vaccine (PCV13). Cholera, Japanese encephalitis virus, and plague vaccines are other inactivated vaccines. Typhoid vaccine is available in an inactivated form and an oral live, attenuated form. Immunization types and recommended administration schedule https://www.cdc.gov/vaccines/schedules/index.html Contraindications for each immunization: See above sections PPD administration • Diagnostic biological agent commonly used in primary care • Used to screen asymptomatic individuals for infection w/ M tuberculosis • PPD is administered intradermally to asymptomatic individuals • Once a person has become sensitized to mycobacterial antigens, a hypersensitivity reaction occurs to the administration of the intradermal PPD • In sensitive people, the reaction includes induration and erythema at the site of administration. o A positive reaction to PPD indicates that the person at some time has had a TB infection. o A positive test does not indicate an active infection but rather that further testing is indicated. • PPD testing is safe in pregnancy, during lactation, and in children of all ages, including infants. • The test consists of injecting 5 TU of PPD intradermally. o A small white bleb should appear at the injection site if it is done correctly. o Reactions are read in 48 to 72 hours after administration. o For patients who may be highly sensitized, a test dose of 1 TU is used. • Determining the results of the skin test is based on the likelihood of infection and the risk of active TB if infection has occurred. o If the patient is HIV-positive or has fibrotic lesions on chest x-ray, a reaction of 5 mm or more induration is considered positive. o A reaction of 10 mm or more induration is considered positive in other at-risk patients, including infants and children. o In patients who are not in any high-risk category or high-risk environment, a result of 15 mm or more induration is considered positive. Post-exposure prophylaxis for rabies • Post-exposure prophylaxis is recommended if the patient has a bite from a rabid animal that penetrates the skin o Post exposure vaccine admin should always be accompanied by use of RIG o Always includes admin of both passive antibody and vaccine except for those who have previously received complete vaccination o ACIP recommendation: four doses of rabies vaccine ▪ 1 mL given IM on days 0, 3, 7, and 14 ▪ And RIG given on day 0 o For those who have previously been vaccinated two doses of rabies vaccine are given on days 0 and 3, with no RIG needed Chapter 27: Anemia Differentiate between the types & etiologies of the anemias • Anemia is a sign of disease rather than a disease itself • Diagnosis and Tx of the various forms of anemia entails interpretation of blood studies and peripheral smears to correctly Dx the type of anemia • Underlying pathophysiology in all types of anemia is a decrease in the oxygen-carrying capacity of the blood • Anemias are classified by erythrocyte size and hemoglobin (Hgb) content. • Size is referred to by the terms microcytic (small), macrocytic (large), and normocytic. • Hgb content is referred to by the terms hypochromic (low Hgb) or normochromic. • Regardless of the disease process, the decreased oxygen transport to tissues carries the same results. • A reduction in the number of circulating red blood cells (RBCs) affects the consistency and volume of blood. • Less viscous blood flows faster and more turbulently and may cause ventricular dysfunction, cardiac dilation, and heart valve insufficiency Iron Deficiency Anemia: • IDA decreases oxygen carrying capacity r/t low hemoglobin concentration that is due to reduced RBC production (lack of adequate iron intake, poor absorption of iron by the body, or lead poisoning) or acute or chronic blood loss • This produces a microcytic-hypochromic anemia o Iron is essential for production of Hgb, myoglobin, and other enzymes. o IDA results in problems with oxygen transport that affect the energy metabolism of every cell • Commonly seen in infants, particularly premature infants, in children during rapid growth, and in pregnant and lactating women • After gastrectomy and with malabsorption d/os (particularly small bowel) • Most common cause in adults is blood loss: menstruation, occult blood loss from GI bleeding, and CA o Gastric and duodenal ulcers, diverticula, hemorrhoids, and ulcerative colitis are common sources of this bleeding o GI bleeding is such a common cause of IDA that it is recommended all men and postmenopausal women with IDA and all premenopausal women whose IDA cannot be explained by heavy menses should be evaluated for occult GI bleed • Individuals with RF (especially on dialysis) are at high risk r/t kidneys not secreting enough erythropoietin o Erythropoietin and iron can both be lost in dialysis • IDA develops slowly in three overlapping stages (develops slowly) • Stage 1: the body’s iron stores are depleted, and the patient has iron deficiency w/o anemia o Erythropoiesis proceeds normally with the Hgb content of RBS remaining normal o Few or any symptoms, vague (fatigue) o Dx by measuring serum ferritin (ferritin less than 30 ng/mL indicative of iron deficiency) • Stage 2: Iron deficiency with mild anemia, iron transportation to bone marrow is diminished and iron deficient erythropoiesis takes place o Lab values begin to change o Hgb 9-12, ferritin of less than 20, and iron le\vel less than 60 o Usually no symptoms or vague symptoms of fatigue or HA o Those with lower Hgb may have more definitive symptoms (weakness, SOB) • Stage 3: severe iron deficiency with severe anemia o The small, hemoglobin-deficient cells enter circulation in sufficient numbers and replace normal erythrocytes that have reached maturity and been removed from circulation o RBC lifespan: 120 days o Associated with depleted iron store and diminished Hgb production o Hgb levels are now 6 to 7, ferritin less than 10, and iron less than 40 o Earlobes, palms, and conjunctivae become pale o Nails are brittle, thin, coarsely ridged, and spoon shaped r/t impaired oxygen transport o Tongue becomes red, sore, and painful o Dry, sore skin at the \corners of the mouth and difficulty swallowing r/t decreased salivation o Confusion, memory loss, and disorientation secondary to poor O2 transport to cerebral tissue o Especially problematic in the older adult population o Severe Ida with Hgb below 7: postural hypotension, dizziness, weakness, gastritis, irritability, numbness, and lethargy Folic acid deficiency anemia • (FOA) decreases oxygen-carrying capacity because of a low Hgb concentration • Exogenous folate is required for nucleoprotein synthesis and maintenance of normal erythropoiesis o Folic acid stimulates production of erythrocytes, WBCs, and platelets • Macrocytic-normochromic anemia within 3 months of the start of an inadequate diet, because folate stores are rapidly depleted • Primary function of folate coenzymes is the synthesis of purines and pyrimidines (bases for DNA and RNA) o These coenzymes also involved with the synthesis of thymidylate (precursor of DNA) o Symptoms: become apparent when the synthesis of thymidylate is critically impaired o Apoptosis of RBCs in the late stages of differentiation occurs • Folate is a critical component of the diet of pregnant women and folate supplementation is recommended o r/t neural tube defects and heart disease • Folate is absorbed in the upper small intestine then circulated to the liver for storage • At risk groups: alcoholics (alcohol interferes w/ folate absorption and metabolism), patients w/ malabsorptive d/os and pregnant women o Inadequate dietary intake of green veggies or excessive boiling of veggies while cooking o Impaired absorption r/t ileal disease or phenytoin use, increased demand during pregnancy, hyperthyroidism, hemolytic anemia, or malignancy: impaired utilization for patients taking methotrexate, triamterene, and trimethoprim • Patients commonly c/o glossitis, stomatitis, nausea and anorexia, and diarrhea • Systolic ejection murmur may be heart • Positive Romberg’s sign and increased or decreased deep tendon reflexes • Mild confusion, depression, apathy, and intellectual loss Pernicious anemia (PA) • Also has a low Hgb concentration • Vitamin B12 is necessary for maturation and DNA synthesis in RBCs • When the cause of vitamin B12 deficiency is autoimmune and linked to heredity, it is PA • Associated with other autoimmune conditions, (particularly endocrine) Hashimoto's thyroiditis, type 1 DM, Addison's disease, and Graves’ disease. • Macrocytic-normochromic anemia that develops slowly, often over years, and it is frequently severe before it is diagnosed. • B12 is critical to 2 essential enzyme systems: o Cofactor in metabolism of methyl malonyl CoA: leads to abnormal fatty acids ▪ Responsible for the neurological manifestations of B12 deficiency o Other system involves folate metabolism: final steps in folate metabolism cannot occur ▪ Leads to megaloblastic anemia found in B12 deficiency • Causes: Vit B 12 deficiency: poor intake, impaired absorption, increased demand, or faulty utilization o Poor intake is rare: except in strict vegetarians (no eggs or dairy products) o Poor intake: vegetarians (vegans), Crohn’s disease or others with resection of the bowel (gastric bypass and pregnant women who are strict vegetarians) • Impaired absorption: lack of intrinsic factor excreted from gastric parietal cells o Absorption: impaired by disease of the ileum, bacterial overgrowth from statis (severe constipation), and altered digestive enzymes: gastrectomy • Faulty utilization: rare genetic defects • Symptoms: vague early symptoms (infections, mood swings, and GI, Cardiac, or kidney problems) o Classic symptoms not seen until Hgb approaches 7 to 8 o Neurological symptoms are the result of nerve demyelination: loss of position and vibratory sense, ataxia, and spasticity o Beefy red tongue, peripheral neuropathy o Liver enlargement (older adults especially): indicates right sided heart failure Anemia of Chronic Disease (ACD) • Develops secondary to a chronic disease, Ca, or long term infection and is the second most common form of anemia • ACD is also associated with low Hgb, but it is caused by destruction of RBCs by a hyperactive reticuloendothelial system, decreased production of RBCs by hypoactive bone marrow, or altered iron metabolism, with defective transfer of iron from stores to the plasma. o Appears to be produced by the activation of the cellular immune system • Specific cytokines that have been implicated in ACD include TNF alpha, interferon delta, IL 1 beta, and IL6 o Microvascular eruptions occur in the GI tract in response to the inflammatory mediators o Results in occult blood escaping into the intestines • Normocytic-normochromic anemia: develops slowly often mild or asymptomatic • Anemias that resemble ACD include anemias secondary to malignancy, chronic renal disease, HIV infection, and heart failure Sickle Cell anemia • SCA: normal amount of Hgb but their REBs contain an abnormal type of Hgb: hemoglobin S • Group of autosomal recessive genetic disorders characterized by the predominance of this Hgb. • Includes SCA (homozygous form that is most severe) • Sickle cell thalassemia syndrome and Sickle cell HbC: heterozygous forms in which the child also inherits another type of abnormal Hgb from one parent • Sickle cell trait, in which the child inherits HgbS from one parent and normal hemoglobin (HgbA) from the other parent, is a heterozygous carrier state. o It does not cause abnormalities in the blood count, and it does not produce vaso-occlusive symptoms under physiological conditions • Found in those of African, Mediterranean, Indian, and Middle Eastern heritage • Sickle cell Hgb: produced by a recessive allele of the gene encoding the beta chain of the protein hemoglobin o A single amino acid, glutamic acid, is replaced by valine at the sixth position of the pain • Two cardinal pathophysiological features: chronic hemolytic anemia and vaso-occlusion (results in ischemic tissue injury) • Low oxygen tension in the blood from ischemia or decreased partial pressures of oxygen in the air cause HgbS to crystallize: distorts the RBC into a sickle shape and makes them fragile and easily destroyed • Degree of deoxygenation required to produce sickling varies with the percentage of HgbS in the cells • Sickling is rarely permanent: regain a normal shape when reoxygenated and rehydrated o Some irreversible sickling occurs r/t damage to the plasma membrane of the RBC o Hemolytic anemia may be r/t repeated cycles of sickling and unsickling • Tissue injury is produced by hypoxia secondary to the obstruction of blood vessels by sickled erythrocytes • Sickled cells are unable to squeeze through the smaller blood vessels=tissue ischemia • Organs at greatest risk for damage are those with venous sinuses in which blood flow is low and oxygen tension and pH are low (spleen and bone marrow) • And those with limited terminal arterial blood supply (eye and head of the femur and humerus) • Kidneys are also a risk Relevant diagnostic tests hemoglobin, hematocrit, reticulocyte count, plasma iron, total iron binding capacity, ferritin, transferrin, mean corpuscular volume (MCV), serum B12, and folate Prevention & treatment for the anemias IDA: supplemental iron (ferrous salts should be used: better absorption) • Iron deficiency cannot be overcome with increased dietary intake alone • Adult dose: 150-300 mg//day in two or three divided doses • Premature breastfed neonates: 2 mg iron/kg/day until 12 months old • All breastfed infants: 1 mg/kg/day of iron beginning at 4 months and continued until the infant is consuming adequate iron containing food o Formula-fed infants do not need iron supplementation • Toddlers 1 to 3 y/o: 7 mg/day ideally from iron fortified foods o Check iron level at around 12 months • Infants and young children require 4 to 6 mg/kg/day divided in three doses for severe IDA o and 3 mg/kg/day for mild to moderate IDA • Tx continues for 3 to 4 months after hemoglobin/hematocrit return to normal to correct the anemia and replenish iron stores o Monitoring: reticulocyte count 7 to 10 days after initiation of therapy: first measurable response to therapy o Rise in RBCs 0.5 to 1.5 % o Hgb at 2 weeks: should indicate a rise of 0.1 to 0.2 g/100 mL/day o Normal Hgb: 14-18 in men and 12 to 16 in women reached in 1 to 3 months o Monitoring RBC and Hgb thereafter based on individual risks, response, and symptoms • Primary goal of management for IDA is prevention. o The key to prevention of IDA not due to a disease process is adequate nutrition. o Prevention, for infants, starts with breastfeeding o Children and adults need to eat enough iron-rich foods o Foods rich in iron that the body can readily absorb include raisins, lean meats, fish, poultry, eggs, legumes, soybeans, dark green leafy vegetables, blackstrap molasses, fortified cereals, and whole- grain rice. o The iron in many vegetables is poorly absorbed, so vegetarians must pay special attention to their intake of legumes and rice. o The recommended amount of iron for non-anemic pregnant women is 27 mg/d, with a recommendation that all pregnant women take a low dose (30 mg/d elemental iron) daily to prevent anemia o Learning energy conservation techniques, such as planning rest periods, pacing activities, keeping objects within reach when performing tasks, and sitting down when doing chores, are other important lifestyle modifications for the patient with IDA • Drug Therapy: once the decision is made to begin drug therapy choice of drug is based on age and gender variables o Iron is safe to use in preterm infants, infants, adolescents, adults, during pregnancy and lactation o doses range from 60 to 185 mg of elemental iron o Oral formulation of 325 mg (60 mg of elemental iron) TID with meals o Ferrous sulfate is the least expensive and the most easily absorbed o Slow release and EC compounds help decrease GI ADRs with only once/day dosing Folic Acid Deficiency • Eat food high ion folic acid: dark green leafy veggies, bran, yeast dried beans, fortified cereals, and nuts • For all females planning or capable of becoming pregnant: .4 to .8 mg/day • Well absorbed, doses of 1 to 2 mg/day result in correction of the deficiency in 4 to 5 weeks • Hgb levels begin to rise within the first week and complete correction in 1 to 2 months • When clinical symptoms have subsided, and labs normalized: maintenance doses 0.1 mg/day for infants to 0.8 mg/day for pregnant and lactating patients • Prevention: supplementation during pregnancy and lactation Pernicious Anemia • Eat foods high in B12: mollusks, fortified breakfast cereals, liver, trout, salmon, milk, and eggs • Prevention: 2.2 mcg/day during pregnancy • 2.6 mcg/day during lactation, 0.3 to 0.5 mcg/day for infants under age 1 • 0.7 to 1.4 mcg/day for children 1 to 10 • Pernicious Anemia: Malabsorption: therapy for life • Initial 1,000 mcg/day for 7 days: IM or deep SC • Response to injected B12 is rapid: reticulocytosis begins on the second or third day and maximal 5-10 days • If clinical improvement and appropriate reticulocyte response after 7 days: then 100 to 1,000 mcg IM weekly for a month • Hgb and Hct levels should return to normal within 1 to 2 months: may take up to 6 months for neurological symptoms to resolve • If neuro symptoms are present twice monthly dose is recommended for 6 months prior to beginning the monthly dose • B12 once monthly for life • Referral to or consultation with a hematologist and gastroenterologist should occur w/ Dx of PA Anemia of Chronic Disease • Most common form of anemia in older adults and associated with several specific chronic diseases including osteomyelitis, TB, Ra, hepatitis, carcinoma, myeloma, lymphoma, and leukemia, CRF • There is no effective therapy directed specifically at ACD o Tx of the underlying chronic disease is necessary to resolve the anemia • Can exist with IDA: Tx with iron • Energy conservation is important • When caused by CRF: decreased production of erythropoietin o Sufficient iron needed o Admin epoetin alfa (Epogen, Procrit) may increase Hct w/in 6 weeks • For CRF or zidovudine-treated HIV: epoetin alpha three times a week o Titrated to keep Hgb between 10 and 12 Sickle Cell Anemia • Prevention is focused on avoidance of precipitants of sickling crisis o Serious bacterial infections ▪ (S. pneumoniae, H. influenzae, and M. pneumoniae are major causes of sickling ▪ Teach early s/s of infections and early aggressive Tx is critical o Dehydration: adequate hydration especially during elevated temperature and physical activity o Exposure to cold: vasoconstriction, avoid • Children should receive all childhood immunizations: including pneumococcal vaccine o Infants are high risk and vaccinated with meningococcal vaccine and persons age 9 months to 55 are vaccinated o Annual flu vaccine starting at 6 months • Encourage patients to exercise: non-strenuous • Drug therapy • Daily MVI, diet rich in complex B vitamins and vitamin C, maintain hydration • 1 mg/day of folic acid during crisis • Infections: increase r/f vaso-occlusive events and subsequent ischemia o Oral PCN BK 125 mg bid until 3 y/o then 250 mg big until 5 • NIH: strong evidence for the use of hydroxyurea in the Tx of SCD in adults o No drug to cure SCD o Hydroxyurea: reduces frequency of sickle cell pain crisis in adults, reduced need for transfusion, and increase total hemoglobin and HgbF o Usually managed by a specialist in hematology o Toxicity can occur: absolute neutrophil counts lower than 2,000, absolute reticulocyte counts lower than 80,000, and platelet s lower than 80,000, or a fall in Hgb from 7 to 4.5-5 o ARF cytopenia: leukopenia and thrombocytopenia may occur w/in 10 days of starting therapy o WBC and platelet: monitor prior to and periodically during therapy o ADRs: stomatitis, anorexia, nausea, vomiting, and diarrhea may occur o Good oral hygiene and monitoring of nutritional status o Do not handle the hydroxyurea tablet w/ bare hands, wear gloves • Transfusions: With the mild to moderate anemia common to SCD, body systems except the eye and spleen adapt well to the reduced oxygen carrying capacity of the blood o Most SCD patients are relatively asymptomatic: do not require transfusions o Transfusions may be used for specific indications when anemia is severe or to prevent chronic complications Monitoring and outcome evaluation IDA • Monitoring: o Response to iron therapy is apparent within 10 days of initiation o First change noted in blood values is an increase in reticulocyte count as soon as 4 days after Tx is started, followed by a rise in Hgb of 1 to 2 g/dL/day o If the anemia is ever (Hgb less than 8): reticulocyte count obtained 5 to 10 days after starting therapy o If the anemia is not severe Hgb Hct, and ferritin checked a t 4 weeks o If mild: (Hgb 10 -12 and Hct 30-60%) follow up every 4 to 6 months o Referral is almost never required unless there is an inadequate response to therapy, there is a question of possible malabsorption that requires a GI specialist or the reason for inadequate response is unclear • Outcome evaluation: o Several weeks of therapy are required o Replenishing iron stores may take months o Speed is not the issue: unless there is rapid blood loss o In that case: consultation or referral o Hgb, Hct, and RBC at 4 weeks, 3 months, and annually o If Hgb does not return to normal w/in 6 weeks; evaluate for inadequate response o If dose is inadequate: increase dose Folic Acid Deficiency Anemia • Monitoring o Response to Tx is assessed after 2 weeks and then monthly until the condition stabilizes o A rapid rise in reticulocytes follows the initial Tx with a peak in 5 to 8 days, an improving Hgb and Hct within 1 week, and a normal Hct within 2 months o Only monitoring required: Hgb and Hct levels at regular intervals Pernicious Anemia (PA) • Monitoring: o Reticulocyte counts, Hgb, Hct, iron, folic acid, and B12 prior to Tx, between the 5th and 7th day of therapy and then frequently until Hgb and Hct are normal o Relapse of symptoms is not uncommon in the presence of continuing therapy o Blood counts at regular intervals throughout the patient’s lifetime (based on response to therapy) o Potassium levels: monitor at the same time as Hgb and Hct o LFTs should also be done (AST and ALT) ▪ If AST and ALT are elevated before the start of cobalamin therapy: evaluate every 2 to 4 weeks (monitor for liver dysfunction) ▪ If LFTs elevate after the start of therapy, more frequent testing might be required to assess for hepatotoxicity ▪ Schilling test: used for Dx but not as a monitoring tool o Consult with hematology and GI doc Sickle Cell Anemia • Hgb, Hct, reticulocyte counts, platelet counts, and WBC frequently during the first year of life to establish baseline • After the first year: Hgb and Hct only once or twice a year in stable patients • UA, BUN, crt, and liver enzyme studies to monitor for evidence of organ damage • Prior to transfusion: RBC antigens are needed, often difficult to crossmatch • Most adults with SCD should have regular medical evaluations every 3 to 6 months • Blood counts, UA, and routine chemistry done annually • With advancing age, complications such as chronic organ failure often require more frequent visits and more extensive labs • Focus on abnormalities in renal function and complications such as gallstones, aseptic necrosis, leg ulcers, and priapism • Goal of therapy: reduction in the number of sickling crises and prevention of organ damage • Hgb goal: 9 or greater but levels of at least 7 may be acceptable in asymptomatic patients • Goal for percentage of Hgb S: 30% or less Recommended amount of iron supplementation for breastfed infants • All breastfed infants 1 mg/kg/day of iron beginning at 4 months and continued until the infant is consuming adequate iron containing food • Formula-fed infants do not need iron supplementation Medication to reduce the number of acute sick cell crisis events: • Drug therapy • Infections: increase r/f vaso-occlusive events and subsequent ischemia o Oral PCN VK 125 mg bid until 3 y/o then 250 mg big until 5 y/o • NIH: strong evidence for the use of hydroxyurea in the Tx of SCD in adults o No drug to cure SCD o Hydroxyurea: reduces frequency of sickle cell pain crisis in adults, reduced need for transfusion, and increase total hemoglobin and HgbF o Usually managed by a specialist in hematology o Toxicity can occur: absolute neutrophil counts lower than 2,000, absolute reticulocyte counts lower than 80,000, and platelet s lower than 80,000, or a fall in Hgb from 7 to 4.5-5 o ARF cytopenia: leukopenia and thrombocytopenia may occur w/in 10 days of starting therapy o WBC and platelet: monitor prior to and periodically during therapy o ADRs: stomatitis, anorexia, nausea, vomiting, and diarrhea may occur o Good oral hygiene and monitoring of nutritional status o Do not handle the hydroxyurea tablet w/ bare hands, wear gloves Vitamin deficiency and risk for neural tube defects during pregnancy • Impaired thymidylate synthesis is thought to be the mechanism behind neural tube defects in the offspring of pregnant women with folic acid deficiency • Evidence suggests that maternal folic acid deficiency is associated with increased risk for neural tube defects in the fetus. • Folate deficiency is more common than vitamin B12 deficiency and often associated with alcoholics, chronic malnutrition, fad diets, and diets low in vegetables Folate requirements during pregnancy • 0.4 mg prior to conception and throughout pregnancy • Pregnant women have increased folate requirements (800 μg daily versus 50 to 100 μg per day) • Folate deficiency is more common than vitamin B12 deficiency and often associated with alcoholics, chronic malnutrition, fad diets, and diets low in vegetables Treatment for anemia of chronic disease: see above section: Tx of chronic disease/underlying cause sometimes iron is IDA and ACD happen together Chapter 37: HIV & Aids • Know the pharmacodynamics, pharmacotherapeutics clinical use, drug interactions and adverse drug reactions for: Antiretroviral medications • Antiretroviral therapy (ART) • ART can increase the number of circulating CD4 T cells: prolonged survival and diminished rate of opportunistic infections • Six classes: nucleoside reverse transcriptase inhibitors (NRTIs or nukes), nonnucleoside reverse transcriptase inhibitors (NNRTIs or non-nukes) protease inhibitors (PIs), fusion inhibitors (FI or entry inhibitors), integrase strand transfer inhibitors (INSTIs), and CCR5 antagonists o Each class inhibits replication in a different way Nucleoside reverse transcriptase inhibitors (NRTIs or nukes) • NRTI Drugs: abacavir, Didanosine, emtricitabine, lamivudine, stavudine, and zidovudine • NtRTI drugs (nucleotide): tenofovir • MOA: inhibition of replication of retroviruses by interfering w/ viral RNA-directed DNA polymerase (reverse transcriptase) o Require phosphorylation to become active in the body thus NtRTI are already chemically activated • Pharmacotherapeutics: eliminated via kidneys: require renal adjustment when CCR is less than 50 mL/min o Abacavir: eliminated by mechanism in the liver: does not require renal adjustment, half life is increased in mild hepatic impairment and must be dose adjust for Child-Pugh class A and contraindicated for classes B and C • ADRs: anemia, bone marrow suppression, flatulence, HA, myopathy, nausea, rash, renal issues, and vomiting o Symptomatic and life threatening lactic acidosis may occur: more common in patients with hepatomegaly and hepatic steatosis o Abacavir is contraindication in HLA B^5701 positive patients: genetic screening required hypersensitivity reaction can be fatal o Didanosine and stavudine: peripheral neuropathy and pancreatitis o Tenofovir: variety of ADRs, decreased bone mineral density, increased biochemical markers of bone metabolism, ARF, acute tubular necrosis, decreased urine volume, Fanconi syndrome, renal impairment, increased crt, interstitial nephritis, nephritis, nephrogenic DI, new onset or worsening renal, proximal renal tubulopathy, RF, and renal insufficiency Nonnucleoside reverse transcriptase inhibitors (NNRTIs or non-nukes) • Drugs: delavirdine, efavirenz, etravirine, nevirapine, and rilpivirine • MOA: Inhibit replication by acting as a specific, noncompetitive, reverse transcriptase inhibitor and disrupting the catalytic site of the enzyme o Attach themselves to reverse transcriptase and prevent the enzyme from converting RNA to DNA: results in HIV genetic material cannot be incorporated into host cell • Kinetics: metabolized in liver by CYP450 system o More prone to clinically significant drug interactions when combined with other ARTs o Interact with other concurrent medications and complementary/alternative medicines (inducers or inhibitors) o Follow guidelines • ADRs: difficulty sleeping, dizziness, drowsiness, fatigue, HA, liver problems (can be severe and life threatening), NVD, rash (can be severe), and vivid dreams Protease inhibitors (PIs) • Drugs: atazanavir, fosamprenavir, indinavir, lopinavir/ritonavir, nelfinavir, ritonavir, saquinavir, and tipranavir • MOA: Selective and competitive inhibitors of HIV protease o Plan an essential role in preventing cleavage of protein precursors essential for maturation, infection of new cells, and replication • Kinetics: Metabolized by CYP450 system: inducers and inhibitors o Ritonavir: very potent CYP450 inhibitor of other PIs (except nelfinavir) ▪ Commonly paired in low doses to intensify the half life of other PIs (ritonavir boosting) o Drug interactions with boosted and non-boosted PIs must be considered when devising an effective ART regimen o Do not require renal dosing adjustment • ADRs: bleeding problem, diarrhea, GI disturbance, hyperglycemia, HLD, lipodystrophy, and liver problem that can be severe Fusion inhibitors (FI or entry inhibitors) • Drugs: enfuvirtide • MOA: work by blocking an important step in the process of entry into CD4 cells known as fusion o By blocking fusion, FIs may prevent HIV from entering and infecting CD4 cells o Target gp120 and gp41 proteins • ADRs: skin itchiness, swelling, and pain at the injection site o Dizziness, fatigue, insomnia, and numbness in feet or legs Integrase strand transfer inhibitors (INSTIs) • Drugs: raltegravir and cobicistat-boosted elvitegravir • MOA: work by blocking the integrase enzyme that HIV needs to make more virus: blocks the virus ability to replicate • Therapeutics: Use with caution when administer with strong inducers of uridine diphosphate glucuronosyltransferase (RIF) o Reduce plasma concentrations of raltegravir • ADRs: raltegravir: CK elevations, diarrhea, HA, myopathy, nausea, pyrexia, and rhabdomyolysis o Elvitegravir: nausea and diarrhea, URI and bronchitis, back and joint pain, and UTI o Cobicistat: GIADRS: NVD, may exacerbate kidney impairment CCR5 antagonists • Drugs: Maraviroc • MOA: Work by blocking a molecule called CCR5 that is found on the surface of CD4 cells so HIV cannot enter • Kinetics: modulated by inhibitors and inducers of CYP3A/Pgp o Inhibitors: (ketoconazole, lopinavir/ritonavir, ritonavir, darunavir/ritonavir, saquinavir/ritonavir, and atazanavir) o Inducers: RIF, etravirine, and efavirenz o ADRs: cough, dizziness, pyrexia, rash, and URI, diarrhea, edema, esophageal candidiasis, flu, parasomnias, rhinitis, sleep d/os, and urinary abnormalities Goals of therapy • The goals of treatment with ART medication are o (Box 37-1) HIV Tx goals: Improve QOL, obtain maximal and durable suppression of HIV, prevent vertical HIV transmission, prolong survival, reduce HIV-related morbidity, reduce transmissibility of HIB, and restore and preserve immunological function • The most important goals are to (1) reduce HIV-associated morbidity and prolong the duration and quality of survival, (2) restore and preserve immunological function, (3) achieve maximal and durable suppression of plasma HIV viral load, (4) and prevent HIV transmission. • These goals are achieved by reducing HIV-related morbidity and mortality, improving quality of life, and restoring and preserving immunological function in persons infected with HIV • Complete eradication is not possible r/t latently infected CD4 T cells that are established during the earliest stages of acute HIV infection o persist with a long half-life despite prolonged suppression of HIV plasma viremia • Follow approved Tx guidelines and strategies: substantial reductions in HIV related morbidity and mortality and reduced vertical transmission • Higher viral loads are associated with more rapid disease progression o Other factors: heightened T cell activation w/ cellular turnover and expression of immune activation markers, probably contribute as well to clinical disease progression and the rate of Cd4 T cell decline • Goal of maximal viral suppression with ART may be difficult w/ pre-existing medication resistance o In 2007, About one of every six new HIV cases involved virus with ART drug resistant mutations • To successful, ART regimens need to contain at least two, and preferably three, active medicines from multiple medication classes. • If maximal initial suppression below the level of HIV detection (less than 50 copies/mL) is not achieved or is lost, it is important to change medication regimens to include at least two active medicines to achieve the maximal suppression goal • To achieve Tx goals, clinicians and patients must work together to define priorities, investigate options, and mutually determine the best Tx plan Treatment in special populations (pregnancy, children) • Dosing principles should be applied to HIV-infected children, adolescents, and adults. • The treatment of HIV-infected children involves unique pharmacological, virological, and immunological considerations • Women should receive optimal ART even if they are pregnant. • ART of the pregnant woman with zidovudine alone or with standard three-drug combinations has dramatically reduced the rates of vertical transmission from mother to child • Treatment of acute HIV infection should be considered optional at this time. Pretesting treatment • CD4 T-cell count • Pretreatment HIV genotypic resistance testing should be considering in selecting the best ART regimen Predictors of success • Predictors of ART Virological Success (Box 37-2) o High level patient adherence to ART regimen o High potency of antiretroviral medication regimen o Higher baseline CD4 T cell count o Low baseline HIB plasma viral RNA level o Rapid (i.e., equal to or greater than 1 log 10 in 1 to 4 months) reduction of viral RNA level in response to ART Phenotype assays • Phenotype assays are used to measure sensitivity to various antiretroviral agents. • The assay reports HIV sensitivity in terms of a ratio above the normal or wild-type IC50. • In other words, it is the half-maximal (50%) inhibitory concentration (IC) of a medicine (50% IC, or IC50). This information allows a provider to determine the relative extent of resistance. NR 566 Week 6 Study Guide Chapter 22: Drugs Affecting the Reproductive System • Know the pharmacodynamics, pharmacotherapeutics, clinical use, drug interactions, and ADRs for: Erectile dysfunction (ED) drugs, Estrogens and Progesterone, and Antiandrogen drugs • Androgen drugs o Testosterone is the primary male androgen o Responsible for: ▪ Growth, maturation, and maintenance of male sex organs and secondary sexual characteristics ▪ Skeletal growth spurt in adolescence and termination of linear growth by fusion of the epiphyseal growth plate ▪ Activation of sebaceous glands (acne during puberty) ▪ Enhances production of erythropoietic stimulating factor, increased RBCs production ▪ Libido o Androgen Drugs: testosterone propionate (in oil, DepoTesterone), testosterone enanthate (in oil, Delatestryl), testosterone cypionate (in oil, Depo-Testosterone), methyltestosterone (Android, Methitest, Testred, Virilon), testosterone gel (AndroGel 1%, AndroGel 1.62%, Axiron, Testium), fluoxymesterone, TD testosterone (Testoderm, Androderm), and buccal testosterone (Striant) ▪ Used to treat Indicated for the symptomatic Tx of • 1) deficiency states in males associated with hypogonadism and • 2) in both sexes for d/os such as CA and HIB, Tx libido, endometriosis, and postmenopausal symptoms in women, have been used illicitly to enhance athletic performance and increase muscle mass o Contraindicated: male breast CA, prostate CA, and Pregnancy (Category X), and lactation • Antiandrogens: several different categories o Androgen hormone inhibitors (5-alpha-reductase inhibitors) ▪ Drugs: Finasteride (Propecia, Proscar) and dutasteride (Avodart) ▪ MOA: Block conversation of testosterone to dihydrotesterone ▪ Used to Tx BPH ▪ PSA levels and digital prostate examination are required monitoring for men on these agents ▪ Finasteride (Propecia, Proscar): • Extensive hepatic metabolism • BPH: 5mg/day, 6 to 12 months of therapy until therapeutic response o Regression of prostate size increased urinary flow, and improve BPH symptoms • Approved for Male pattern baldness: 1 mg/day, three months until results • Stopping the drugs reverses the effect within 12 months • ADRs: decreased libido, impotence (can occur at both doses) ▪ Dutasteride (Avodart) • Inhibits both type 1 and 2 (5-alpha-reductase) • Peak clinical effect 6 to 12 months of therapy • Extensively metabolized in the liver (CYP3A4 and CYP3A5) • Used to Tx BPH • Absorbed through the skin, women who are pregnant or may become pregnant should not handle dutasteride capsules r/t risk of fetal anomaly to a male fetus • ADR: decreased libido and impotence o Gonadotropin-releasing hormone analogue: luteinizing hormone-releasing hormone antagonist ▪ Leuprolide acetate (Lupron) ▪ Create a reversible chemical orchiectomy state in males and an oophorectomy state in females ▪ Used to Tx: advanced prostatic CA and for management of endometriosis and uterine leiomyomata (fibroids) ▪ 1 mg SC daily or IM every 3 months (depot formulation) ▪ Increased suppression when used with flutamide (direct antiandrogen) ▪ Peds: Tx central precocious puberty ▪ Women: reducing uterine fibroids, endometriosis, and PCOS (pain relief, regain fertility) o Direct antiandrogens ▪ Flutamide (Eulexin), bicalutamide (Casodex), and nilutamide (Nilandron) ▪ Inhibit androgen uptake or nuclear binding of androgen at target tissues ▪ Uses as part of combo therapy Tx of prostatic carcinoma ▪ Flutamide: competitive antagonist at the androgen receptor site • Truly a nonsteroidal agent • ADRs: gynecomastia and reversible liver toxicity • Renal doses: less than 29 mL/min • BLACK BOX WARNING: hepatic failure, hepatic encephalopathy, and death o Usually within the first 3 months of therapy o Baseline and monthly LFTs for the first 4 months of therapy and periodically o D/c if any symptoms of hepatic injury or jaundice develop o Spironolactone (Aldactone): aldosterone antagonist and inhibitor of 5-alpha-reductase ▪ used as a K sparing diuretic ▪ Off label use for Tx of female hirsutism and acne due to antiandrogenic properties • 50 to 200 mg/day ▪ Competitive inhibitor of the dihydrotestosterone, aldosterone, and interferes with the androgen receptors in the prostate • Also reduces 17-alpha-hydroxylase activity: lowers plasma levels of testosterone and androstenedione ▪ Metabolized by the liver ▪ Short term use for primary hyperaldosteronism in patients preoperatively ▪ Long term: for those who are not good candidates for Sx with idiopathic hyperaldosteronism ▪ PMS/PMDD symptoms may be relieved by spironolactone (25 mg QID beginning on day 14 of the menstrual cycle) ▪ ADRs: dose-related and reversible when the drug is d/c GI upset, drowsiness, gynecomastia, impotence, cutaneous eruptions, and urticaria ▪ BLACK Box Warning: r/t animal chronic toxicity studies demonstrated tumorigenicity ▪ Contraindications: pregnancy • Drug Interactions o Finasteride: Nevirapine: Combo induces hepatic metabolism of finasteride: monitor for effectiveness of finasteride o Leuprolide: Pituitary, gonadotropic, and gonadal function lab test: misleading results: consider if lab test reports show unexpected values o Flutamide: Warfarin: Increased INR and risk of bleeding: monitor INR closely o Spironolactone: ▪ renal impairment and agents that affect renal function: alternation in renal function and electrolytes: monitor K in young patients with reduced renal function and patients greater than 65 y/o ▪ Digitals: may increase or decrease digitalis half-life: Monitor closely for s/s of dig toxicity and electrolyte and digitalis levels. Consider alternative Tx. ▪ Potassium: additive effect increases risk of hyperkalemia: recommend alternative ▪ Eplerenone (aldosterone receptor antagonist used for CHF): severe hyperkalemia: Combination is contraindicated, do not use together, choose different therapy Estrogens: Drugs: estrogen (conjugated A/Synthetics)(Cenestin), esterified estrogen/methyltestosterone (Covaryx, Covaryx HS, EEMT, EEMT HS), estrogen (conjugated B/Synthetic)(Enjuvia), estrogen (esterified)(Menest), estrogen (conjugated/equine)(Premarin), estrogen (conjugated/equine) and Medroxyprogesterone (Premphase, Prempro) • Endogenous hormone with multiple actions • Primary role: maturation and function of the female reproductive system • Nonreproductive effects: bone, CV system, CNS, and GI tract • Contraindication: estrogen-only products in women with an intact uterus o Combo estrogen and progesterone products in these patients Pharmacodynamics • Estrogen occurs naturally in several forms • Primary sources of estrogen in the normally cycling adult women is the ovarian follicle (70 to 500 mcg of estradiol daily) • This estradiol is converted to estrone: circulates in about equal amounts to the estradiol and too small amounts of estriol • After menopause: endogenous estrogen is generated from conversion by peripheral tissues of androstenedione, secreted by the adrenal cortex, to estrone • Effects of estrogen on the reproductive system o Maturation of reproductive organs o Development of secondary sex characteristics o Regulation of the menstrual cycle o Endometrial regeneration post menstruation • Other physical effects: o Closure of long bones after the pubertal growth spurt o Maintenance of bone density by decreasing the rate of bone resorption through antagonizing the effects of parathyroid hormone (PTH) o Maintenance of the normal structure of skin and blood vessels through its actions on the endothelial cells in the arterial wall, including the induction of NO to facilitate vasodilation and O2 uptake by cells o Reduction of the motility of the bowel through its modulation of SNS control over smooth muscle o Alters production and activity of selected proteins, results in higher levels of thyroxine-binding globulin, sex hormone-binding globulin, transferrin, and renin substrate o Enhancing the coagulability of blood by increasing the production of fibrinogen o Facilitating the loss of IVF into EC space by its action on the RAAS (retention of Na and water by the kidney), results in edema and decreased ECF volume o Maintaining the stability of the thermoregulatory center in the brain • Control of estrogen secretion is by the hypothalamus through the pituitary gland o GnRH from the hypothalamus control FSH and LH from the anterior pituitary o FSH and LH stimulate follicular development in the ovary o In the presence of adequate estrogen, LH surge causes ovulation o Negative and positive feedback loops Pharmacotherapeutics • Used for replacement after oophorectomy and natural menopause for Tx of hot flashes, vaginal atrophy, and irregular menstrual bleeding • The dose of estrogen needed for contraception is higher than the dose needed for replacement therapy o Potency ratio of replacement to contraception estrogens is 1:10 • Absolute contraindication: current or prior Hx of an estrogen-dependent CA, current pregnancy, undiagnosed dysfunctional uterine bleeding, deep vein thromboembolism, arterial thromboembolism within the prior year, clotting d/os, and severe hepatic disease • Relative contraindications: require added discussion with patient r/t risks/benefits o CVD, uncontrolled HTN, DM, gallbladder disease, obesity, endometriosis, seizure d/o, and migraine o Women who experience migraine with aura are at increased r/f stroke and should not be Rxed estrogen • Women with an intact uterus should not be Rx unopposed estrogen r/t risk of endometrial hyperplasia and endometrial CA • Estrogens have been implicated in the risk of endometrial CA • Women’s Health Initiative: clinical trials findings have had a considerable impact on the Rx patterns of clinicians and perception of hormone therapy by the public o Estrogen replacement therapy (ERT) and estrogen plus progestin therapy (HRT): increased morbidity and mortality r/t coronary heart disease CHD o WHI study: higher incidence of CHD, stroke, and thromboembolism both in women taking estrogen- only or HRT therapy • Estrogens: Pregnancy Category X r/t high rate of teratogenicity in male and female offspring • Ethinyl estradiol: contraindicated in patients who smoke and older than 35 y/o o Smokers may use postmenopausal hormone replacement therapy (ERT/HRT) r/t lower dosages of hormone o The risk of clotting and CV events remains higher than for nonsmokers o Interaction of estrogen and smoking: dose-related morbidity and mortality ADRs • Most of the ADRs are dose-related • Most concerning: cardiovascular and hematological MI, HTN, alteration in clotting factors, and thromboembolism • Unopposed estrogen: if used in women with an intact uterus: increased r/f abnormal uterine bleeding, endometrial hyperplasia, and gynecological CAs: do not Rx • Elevated systemic BP, gallbladder disease, and irregular bleeding • Women with estrogen-dependent tumors may have worsening of CA: do not Rx Drugs Interactions • Can interfere w/ lab measurements of endocrine, LFTs, and thyroid bindings globulin (in most cases, does not) • PTT and factors VII, VIII, IX, and X show increased levels while taking estrogens • Impaired glucose tolerance and increased triglycerides • Most common: anticoagulants, tricyclic antidepressants, barbiturates, anti-TB drugs, corticosteroids, seizure control meds, and drugs for spasticity • Estrogens: oral anticoagulants: estrogens increase the risk of thromboembolism, contraindicated in those at high risk for coagulopathy Clinical Use • Relief of Perimenopausal and Postmenopausal Symptoms o Relief of menopausal vasomotor symptoms can be dramatic after the initiation of hormonal therapy o Use the lowest effective dose of estrogen and for the shortest duration possible o Available in various formulations: derived from animal, plant, and synthetic sources o Lowest dose to control symptoms o ERT and HRT should be reevaluated on an annual basis: the goal of utilizing therapy for 5 years or less o Conjugated equine estrogen (Premarin) 0.3 to 2.5 mg ▪ Change dose at 6 to 8 weeks if needed o Micronized estradiol (Estrace, Gynodiol) ▪ Only bioidentical estrogen alone product available in pill form ▪ 0.5 to 2 mg doses ▪ Vasomotor suppression at 1 and 2 mg doses ▪ Typical regimen is 1 mg/day ▪ Lower dose 0.5 mg/day used for osteoporosis prevention (less useful for vasomotor relief) ▪ The bisphosphonates used for Tx of osteoporosis: only used together in extreme cases where both are needed for full bone effect o Estrone-based drugs ▪ Estrogens derived from plant sources ▪ Synthetic conjugated estrogen-A (Cenestin): 0.3 to 1.25 mg ▪ Synthetic conjugated estrogen-B (Enjuvia): 0.3 to 1.25 mg ▪ Estropipate (Ogen, Ortho-EST): 0.75 to 6 mg • Prevention and Management of Vulvovaginal Atrophy and Dryness • A decline in estrogen causes vaginal mucosa and vulvar skin to become thin and atrophic o Results in discomfort, itching, dyspareunia, and increased cases of vaginitis • Low dose oral ERT with estrogen from plant or animal sources has been shown to decrease vaginal pH: reduces vaginal infections o Thickens and revascularizes vaginal epithelium increases the number of superficial cells and reverses vaginal atrophy • Topical creams and intravaginal delivery of estrogen are equally as efficacious or PO agents in the management of vulvovaginal symptoms • An option for dryness and dyspareunia: estrogen agonist-antagonist ospemifene (Osphena) o Technically a SERM med but does not have bone health indications o ADRs: increased risk of blood clots, hot flashes, and uterine CA o Use for the shortest possible time to overcome issues of moderate to severe painful intercourse o 60 mg/day o Contraindications: prior blood clots, active thromboembolic disease, or family Hx of such, known estrogen-dependent hyperplasia or neoplasia, and abnormal genital bleeding • Reduced Risk for Colon CA o 3rd most common CA in women in the US and the third most common cause of CA death in women o Associated with aging: should be considered concurrently with menopause o ERT and HRT have been shown to reduce the risk of CRC however not approved for this use • Prevention and Treatment of Osteoporosis o Estrogen is associated with bone formation and has osteoprotective benefits o Prevent osteoporosis by reducing the bone-resorbing action of PTH o Studies have shown that there is a direct correlation between the rate of bone loss in menopausal women and estradiol levels o Bone resorption is highest in the first postmenopausal year: early therapy o Estrogen is used alone and in combination with other drugs o ERT and HRT can significantly slow the progression of bone loss o Improvement in bone mineral density (BMD) and reduction of hip and vertebral Fx o Due to CV risks FDA, and USPSTF encourages the use of alternative options for osteoporosis prevention and Tx • Contraception o Two formulations of estrogen available in combo contraceptive preparations: ethinyl estradiol (EE) and mestranol o Mestranol is weaker of the two: metabolized into EE ▪ 55 mcg of mestranol is equivalent to 35 mcg of EE o EE is the estrogen used in most hormonal contraceptive formulations ▪ (most contain 20 to 25 mcg of EE) o Suppresses FSH release and development of a dominant follicle: improves the efficacy ▪ Also adds to cycle control, decreasing the irregular bleeding patterns commonly found with progestin-only methods o Combined OCs come in three main formulations o Most common formulation: monophasic combined oral contraceptive (COC) ▪ Contains the same dose of hormone in each active pill ▪ Effectiveness of prevention: 99% or greater o Biphasic pills: altered hormone dose in the middle of the cycle o Triphasic: alter the estrogen dose, the progesterone dose, or both each week during a 28-day pack o Progesterone-only contraceptives available Progesterone • Drugs: progesterone (Prometrium, Progesterone in oil, Crinone, Prochieve), medroxyprogesterone acetate (Provera), norethindrone (Aygestin), and megestrol acetate (Megace) • Most are used in OCs and for HRT • Many formulations/generations of progesterone have been developed o Address issues of mood change, breakthrough bleeding, and sensitivity to progesterone agents Pharmacodynamics • Effects of progestin on the reproductive organs: o Thickening of the endometrium and increasing its complexity in preparation for pregnancy o Thickening of cervical mucus o Thinning of the vaginal mucosa o Relaxation of smooth muscles of the uterus and fallopian tube • During Pregnancy: progestin maintains the thickened endometrium, relaxes myometrial muscles, thickens the myometrium for labor, responsible for placental development, and prevents lactation until the fetus is born • Absence of pregnancy: reduced production of estrogen and progestin by the corpus luteum results in shedding of the endometrium to produce menstruation • Also responsible for alveolobular development of the secretory apparatus of the breast • Actions outside the reproductive system: o Stimulates lipoproteins activity: favors fat deposition o Increases basal insulin levels and insulin response to glucose o Promotes glycogen storage in the liver o Promotes ketogenesis, o Competes with aldosterone in the renal tubule to decrease NA resorption o Increases body temp o Increases ventilatory response to CO2 (only during pregnancy) Pharmacotherapeutics • Contraindications: thromboembolic disease or Hx of it • Breast cancer may become worse under hormone influence • Patients with impaired liver function would have trouble metabolizing exogenous hormones • D/c if depression recurs or occurs to a serious degree • Fluid retention may occur: those with d/os that may be negatively affected by excess fluid (epilepsy, migraine, asthma, CHF, or renal dysfunction) require careful observation • A decrease in glucose tolerance has been observed in patients on estrogen-progestin combo drugs o DM: increase glucose monitoring when on progestin • Progesterone: Pregnancy Category D • Norethindrone: Pregnancy Category X • Progesterone gel is used to support embryo implantation and maintain pregnancies as part of assisted reproductive technology (ART) treatments ADRs: • The most common ADR: irregular, breakthrough vaginal bleeding • Some may experience amenorrhea • Acne and chloasma: seen more with the more androgenic progestin products • Increased breast tenderness and galactorrhea • Nausea, depression, and weight gain • Injectable and implanted progesterone for contraception: associated with an increased incidence of weight change and irregular menstrual bleeding • Low estrogen levels associated with IM progestin (Depo-Provera): increased risk for osteoporosis o Depo-Provera: Black Box Warning FDA cautioned using this form of contraception for longer than 2 years unless other forms of contraception are not viable options o Bone density loss may extend beyond the duration of Tx o Perform a risk assessment and discuss with the patient o Delays returning to a fertile state have occurred with IM progestin use Drug Interactions • Aminoglutethimide and rifampin • Decrease the effectiveness of progestin therapy: unplanned pregnancy • Can cause erroneous lab results in LFTs, coags, thyroid, metyrapone, and other endocrine functions Clinical Use • Perimenopausal and Postmenopausal Hormone Replacement o Combinations of estrogen and progestin are used when the uterus is intact o Progestin: reduce risk of endometrial CA secondary to endometrial hyperplasia with ERT o Helps to prevent this increased incidence: reduce the buildup of endometrial tissue • Progestin-Only Contraception o Exhibit negative effect in the hypothalamic-pituitary-ovarian axis: suppresses LH surge needed for ovulation o Cause thickening of cervical mucus: sperm have difficulty penetrating o Tubal motility is slowed: delayed transport of ovum and sperm o Cause atrophy of the endometrium: prevents implantation • Off-Label Uses o Treatment of dysmenorrhea, endometriosis, hirsutism, and menstrual bleeding d/os are implemented when estrogen is contraindicated o Gel form of progesterone: used to assist with infertility o Can be used in conjunction with estrogen for postmenopausal hormone therapy Erectile Dysfunction (ED) Drugs: Phosphodiesterase Type 5 Inhibitors • Drugs: sildenafil citrate (Viagra), vardenafil (Levitra), and tadalafil (Cialis) • Ed is a common health condition associated with increased age and various comorbidities (DM, HTN) • In elderly patients start with the lowest dose Pharmacodynamics • Produces smooth muscle relaxation in the corpus cavernosum, increased blood flow, results in erection • No drug effect w/o sexual stimulation • Relaxes bladder smooth muscle: second indication of benign prostatic enlargement (if used daily) • Metabolized in the liver (CYP3A4) Pharmacotherapeutics • Absolute contraindication: concomitant use with any form of nitrates r/t risk for severe hypotension, CV collapse, and death • Patients with underlying CVD require additional pretreatment evaluation • ACC and AHA say that patients with unstable CAD, active ischemia, HF, and low BP, and patients on multiple antihypertensive agents or medication that inhibit CYP3A4 should not take PDE5 inhibitors • Not given after stroke, MI, or recent life-threatening dysrhythmias r/t heightened sensitivity to bp changes for up to 6 months • Vardenafil: QT prolongation, use with caution in patients with arrhythmias, on antiarrhythmics, or with hepatic insufficiency • Tadalafil: not indicated for persons with retinal pigmentosa • Patients who experience CP or dizziness with sex should refrain from additional use of these agents and sex until they have been evaluated. Clinical Use • Erectile Dysfunction • Sildenafil citrate (Viagra): 50 mg (25-100 mg) taken PRN approx. 1 hr before sex • Vardenafil: 1 hr before sex • Tadalafil: 15 minutes before sex o Also used to treat BPH • Intended frequency is every 3 days Drug Interactions • Antifungals, macrolide antibiotics (erythromycin) cimetidine, rifampin, alpha-blockers (additive hypotension), nonspecific beta-blockers, and diuretics • These drugs should not be used concomitantly or doses of the phosphodiesterase inhibitor should be adjusted downward ADRs • Common: HA, flushing, dyspepsia, and blue hue vision change (most common with sildenafil) • Priapism: rare but emergent • Reports of nonarterial anterior ischemic optic neuropathy (NAION) and cyanopsia (blue vision) • NAION: permanent loss of vision from ischemia of the optic nerve head • US Federal Aviation Administration: pilots should not use sildenafil or vardenafil for 6 hours before flying • Tadalafil: not approved for pilots (or workers requiring color recognition) • Sudden hearing loss Also Know: Monitoring of patients in ED drugs • While on therapy: periodic medication and health evaluations are encouraged to identify any new potential cardiovascular or health risks that may alter the ability to continue ED therapy Androgen production • Testosterone is the most important androgen in humans • Converted to dihydrotestosterone in the skin, prostate, seminal vesicles, and epididymis • In men: produced primarily within the testes and interstitial or Leydig cells (located in spaces between the seminiferous tubules) • Women produce small amounts of testosterone in the menstruating ovary and the adrenals • Both sexes produce testosterone peripherally from androstenedione, dehydroepiandrosterone (DHEA), and dehydroepiandrosterone sulfate (DHEAS) Contraindications for estrogen-containing products • Absolute contraindication: current or prior Hx of an estrogen-dependent CA, current pregnancy, undiagnosed dysfunctional uterine bleeding, deep vein thromboembolism, arterial thromboembolism within the prior year, clotting d/os, and severe hepatic disease • Relative contraindications: require added discussion with patient r/t risks/benefits o CVD, uncontrolled HTN, DM, gallbladder disease, obesity, endometriosis, seizure d/o, and migraine o Women who experience migraine with aura are at increased r/f stroke and should not be Rxed estrogen • Women with an intact uterus should not be Rx unopposed estrogen r/t risk of endometrial hyperplasia and endometrial CA • Estrogens have been implicated in the risk of endometrial CA • Women’s Health Initiative: clinical trials findings have had considerable impact on the Rx patterns of clinicians and perception of hormone therapy by the public o Estrogen replacement therapy (ERT) and estrogen plus progestin therapy (HRT): increased morbidity and mortality r/t coronary heart disease CHD o WHI study: higher incidence of CHD, stroke and thromboembolism both in women taking estrogen- only or HRT therapy • Estrogens: Pregnancy Category X r/t high rate of teratogenicity in male and female offspring • Ethinyl estradiol: contraindicated in patients who smoke and older than 35 y/o o Smokers may use postmenopausal hormone replacement therapy (ERT/HRT) r/t lower dosages of hormone o Risk of clotting and CV events remains higher than for nonsmokers o Interaction of estrogen and smoking: dose-related morbidity and mortality Progesterone antagonists (Box 22-3) • Mifepristone (Mifeprex) • Approved by the FDA for termination of intrauterine pregnancy • Long half-life of 18 hrs and may prolong the follicular phase of the subsequent cycle • Strongly protein-bound: this binding is saturable • Nonlinear pharmacokinetics r/t plasma concentration and clearance • The antiprogesterone activity results from competitive interaction with progesterone at progesterone- receptor sites • The drug inhibits the activity of both endogenous and exogenous progesterone • When there is no progesterone to maintain a pregnancy, termination results • In 85% of women mifepristone acts as an abortifacient when used in conjunction with misoprostol during the first 7 weeks of pregnancy • Patient should expect bleeding or spotting for an average of 9 to 16 days • Persistent heavy or moderate bleeding for more than 30 days: could indicate incomplete abortion • Specific requirements associated with administration of this drug: best done in clinics that can meet these requirements • Available only from the manufacturer, not through pharmacies • Also exhibits anti-glucocorticoid and weak antiandrogenic activity • Off label use: Tx of endometriosis, Cushing’s syndrome, and uterine fibroids Chapter 31: Contraception Differentiate between the different forms of contraception, MOA, and efficacy • All PO estrogen and progestins undergo first-pass metabolism in the liver • GI illness (NVD): can decrease OC effectiveness r/t decreased absorption, use a backup method for 7 days • Estrogen: positive effects on bone mass, increasing serum triglycerides, improving HDL to LDL ratios, and stimulating coagulation and fibrinolytic pathways o Most contraceptive preparations use ethinyl estradiol (EE) o Mestranol: used only in 3 brands of OCs: metabolized into EE o Estradiol valerate: synthetic prodrug of 17b-estradiol: used in quadriphasic Natizna • Progesterone: Increases body temperature, increases insulin levels, and may slightly depress the CNS • Most progesterones are derivatives of testosterone o Norethindrone, norethindrone acetate, and ethynodiol diacetate (first generation) ▪ Norethindrone acetate and ethynodiol diacetate: converted to norethindrone ▪ Increased spotting and breakthrough bleeding: lead to second gens o Norgestrel and levonorgestrel (Second generation) ▪ Increased androgenic activity: decreased breakthrough bleeding however increased androgenic activity lead to acne, hirsutism, and dyslipidemia o Desogestrel and norgestimate (third generation) ▪ Offer decreased androgenicity ▪ Reduced ADRs on CHO and lipid metabolism found in previous formulations ▪ Lessened acne and hirsutism ▪ Medroxyprogesterone acetate: injectable contraception o Drospirenone (fourth generation) ▪ Derived from spironolactone, mild diuretic effect and anti-mineralocorticoid effects ▪ May cause hyperkalemia, use with caution with ACEs or other K sparing drugs ▪ Do not use with abnormal adrenal, renal, or hepatic function o Dienogest ▪ Displays properties of 19-nortestosterone derivates and properties associated with progesterone derivatives producing an anti-androgenic action • Rational drug selection o Start: exclude methods that are absolutely or relatively contraindicated r/t existing health concerns and age ▪ Contraindications to estrogen narrows choices considerably o Then review which delivery mode appeals to the patient o Then fine-tune choices based on acceptability and likely changes in bleeding pattern and side effect profile o Timing of a subsequent pregnancy should also be considered ▪ Some methods: a return to fertility is delayed o More than four dozen formulations of monophasic COC, two dozen multiphasic COCs and several formulations of progestin-only pills (POPs), plus nonoral hormone delivery methods • A shortlist should include one preparation that does not contain estrogen, an ultra-low dose, or 20 mcg EE (to use for women older than 35 yo or those who smoke more than 15 cigs/day), a monophasic COC, a multiphasic COC, and a nondaily administration method for women who have difficulty with daily regimens Method Advantages Disadvantages Perfect Use Efficac y Typical Use Efficac y Estrogen/Progestin Combination Contraception Pills • Menstrual cycle control • Daily administration may be difficult for some >99% 91% Patch ▪ Menstrual cycle control ▪ Weekly administration ▪ Ortho Evra: topical patch 20 mcg of EE and 150 mcg of norelgestromin ▪ Patch is applied once a week for 3 weeks with 1 week being patch- free (withdrawal bleeding occurs) ▪ Patch initiated on the first day of menses ▪ Rotate patch location: ABD, upper torso, outer arm, or buttock ▪ Avoids first-pass metabolism (decreases risk of dyslipidemia) ▪ May cause some local irritation ▪ Skin irritation ▪ May fall off partially or completely ▪ Decreased efficacy in women with high BMI: still uncertain >99% 91% Ring • Menstrual cycle control • Once per cycle administration • NuvaRing: soft flexible plastic ring: releases 15 mcg of EE and 120 mcg of etonogestrel daily • Placed in vagina left in place for 3 weeks, removed for a week when withdrawal bleeding occurs • Does not require fitting, placed by the user • Releases steady, low doses of hormones (better cycle control in the form of decreased breakthrough bleeding when compared to OCs • Lower systemic exposure to EE • User must be comfortable with vaginal administration • Contraindicated in women who have significant pelvic prolapse >99% 91% Progestin Only Oral contraceptio n • No estrogen; may be useful for users whom estrogen contraindicated (breastfeeding) • Micronor and Nor-QD • Three generics (.35 mg norethindrone) • Daily administration may be difficult for some • Unpredictable bleeding pattern • Breast tenderness • MOA: thickening of cervical mucus and prevention of sperm penetration • If a pill is taken even a few hours late a backup method is recommended for 48 hrs >99% 92% Progestin injectable contraceptio n • Given once every 12 weeks • Reliable efficacy • Use of method is discreet • Depot medroxyprogesterone acetate (DMPA, Depo-Provera) • 150 mg IM is effective at suppressing ovulation for 12 to 13 weeks • 104 mg SC dose of DMPA: lower hormone dose with no change in efficacy even in patients with high BMI • Can be done by the user w/o an office visit • MOA: thickens cervical mucus and atrophies the endometrium • Administration requires office visit • User errors can occur if the patient does not return for doses within the prescribed time • A repeat dose up to 1 week late w/o clinical assessment to exclude pregnancy • Unpredictable bleeding patters • Weight gain • Delay in return of fertility, 9 to 10 months • Black Box Warning: Depo-Provera: decrease in bone density with longer-term use • Should not be used for more than 2 years consecutively if other alternatives are acceptable • Particular concern in use in adolescents: bone accretion is still underway >99% 94% Progestin implant • Offers contraception for 3 years • Implanon: one rod contains 68 mg of etonogestrel (the active metabolite of Desogestrel) • Insertion and removed requires an office visit • Unpredictable bleeding patters >99% >99% Intrauterine Device (IUD) Copper • Hormone-free • Offers contraception for 10 years • Insertion and remove requires an office visit • Dysmenorrhea and menstrual flow may be increased in the first few months after insertion >99% >99% Progestin- releasing • Decreased menstrual flow • Offers contraception for 3 or 5 years • Mirena: Releases 20 mcg of levonorgestrel daily • Skyla: levonorgestrel 14 mcg/d and then declines to 5 mcg/d after 3 years • Only small levels of systemic circulating hormone: small incidence of systemic ADRs • MOA: thickening of cervical mucus and endometrial atrophy but only a minimal effect on ovulation suppression • Insertion and removal requires an office visit • Many women experience a notable difference in flow or amenorrhea • Normal endometrial function: returns in 1 to 3 months • Changes in menstrual bleeding are common • >99% >99% Barrier methods Male condom • Protection from most STI’s • Available without prescription • Use linked to coitus • Partner-dependent 98% 82% Female condom • Protection from most STI’s • Available without prescription • Use linked to coitus 95% 79% Spermicide • Available without prescription • Use linked to coitus 82% 71% Diaphragm • May be inserted several hours before intercourse • Available only with proper provider fitting and prescription • Must be used with spermicide • User must be comfortable with vaginal insertion technique 94% 88% Cervical cap • May be inserted several hours before intercourse • Available only with proper provider fitting and prescription • Must be used with spermicide • User must be comfortable with vaginal insertion technique • Lower efficacy for parous women 74-91% 68-84% Vaginal sponge • Available without prescription • Contains spermicide • May be inserted several hours before intercourse • Lower efficacy for parous women 80-91% 68-84% Combined contraceptive medication • Mechanism of Pregnancy Prevention o Progestins: primarily responsible for the contraceptive effect in hormonal preparations ▪ Exhibit a negative effect in the hypothalamic-pituitary-ovarian axis: suppresses LH surge needed for ovulation ▪ Thickening of cervical mucus, making penetration by sperm difficult ▪ Tubal motility is slowed, delaying transport of the ovum and sperm ▪ Cause atrophy of the endometrium, preventing implantation o Estrogen component of hormonal contraception improves efficacy by suppressing FSH release ▪ Development of a dominant follicle ▪ Adds to cycle control, decreasing irregular bleeding patterns commonly found with progestin-only methods o More than four dozen formulations of monophasic COC, two dozen multiphasic COCs and several formulations of progestin-only pills (POPs), plus nonoral hormone delivery methods • OCs: traditionally dispensed in 28 pill packages or 21 pills (for 7 days either pill or placebo while bleeding) • Primary difference between brand names is the type and dose of progestin used • Monophasic preparations: same dose of estrogen and progestin for each active pill • Biphasic: vary the dose of progestin with an increase in the amount of progestin in the latter half of the active pills (rarely used today) • Triphasic: vary the dose of estrogen, progestin, or both • Extended menstrual cycling: Seasonale: active pills for 84 days, with 7 days off, produces withdrawal bleeding once every 3 months o Seasonique and LoSeasonique: classified as monophasic but contain 10 mcg of EE in the last week of once cycle of dosing o Lybrel: uses a continuous cycle without any hormone-free weeks for 1 year o Monophasic pills may be used similarly to space or manipulate the timing of menstrual periods, using the active pills continuously with less frequent intervals of inert pills • COCs are Started in several different ways o First-day start: first pill is taken on the first day of the menstrual cycle o Sunday Start: First pill on the Sunday following the start of menses, backup method for 7 days o Quick start: first pill is taken on the day of the office visit, a backup method for 7 days • Do not smoke with combined therapy at any age and the increased risk in women over age 35 • Stop combined therapy 4 weeks before and 2 weeks after r/t prevention of thrombus formation Use of contraceptives in obese women • Possible decreased efficacy of topical patch contraceptivesstill uncertain • BMI greater than 30: increase the risk of VTE 3 to 10 times Black Box Warnings • Black Box Warning: Depo-Provera: o decrease in bone density with longer-term use o Should not be used for more than 2 years consecutively if other alternatives are acceptable o Particular concern in use in adolescents: bone accretion is still underway Missed medication instructions Estrogen dosing recommendations (Table 31-3) WHO Contraindications to Initiation of Combined Contraception with 35 mcg EE or Less Risk Contraindication WHO Category 2 Contraceptive benefits usually outweigh risks; may require more frequent monitoring Age >40 yr Smoker <35 yr BMI >30 due to increased VTE risk Hx HTN during pregnancy, BP now normal Known hyperlipidemia First-degree relative with DVT/PE Major surgery without prolonged immobilization Superficial thrombophlebitis SLE on immunosuppressive therapy or with severe thrombocytopenia Rheumatoid arthritis Sickle cell disease Valvular heart disease, uncomplicated Migraine without neurological aura, age <35 yr Unexplained vaginal bleeding, suspicious for a serious underlying condition Cervical intraepithelial neoplasia Cervical cancer, awaiting treatment Undiagnosed breast mass Risk Contraindication Diabetes, insulin-dependent or non-insulin-dependent, without vascular disease Asymptomatic gallbladder disease or post cholecystectomy Benign focal nodular hyperplasia of the liver Hx of cholestasis in pregnancy Ulcerative colitis or Crohn's disease Postpartum & breastfeeding, 4–6 wks PP with a normal risk of DVT and >6 wks PP Postpartum & not breastfeeding, 3–6 wks PP with a normal risk of DVT WHO Category 3 Risks usually outweigh contraceptive benefits Postpartum & breastfeeding, 3–4 wks and 4–6 wks with increased risk of DVT Postpartum & not breastfeeding, 3–4 wks with increased risk of DVT Age >35 yr and smoker <15 cigarettes/d Several drug interactions: protease inhibitors; anticonvulsants; lamotrigine; rifampin Multiple risk factors for coronary artery disease (based on age, smoking, diabetes, hypertension) Hypertension, adequately controlled and monitored • Systolic 140–159 or diastolic 90–99 Migraine without neurological aura, age >35 yr Hx of breast cancer, no disease >5 yr Diabetic with nephropathy, neuropathy, retinopathy Diabetic with vascular disease Diabetic >20 yr Symptomatic gallbladder disease Past COC-related cholestasis WHO Category 4 Unacceptable risk WHO Category 4 Unacceptable risk (continued) Postpartum <3wk Age >35 yr and smoker >15 cigarettes/d HTN, not controlled or with vascular disease • Systolic ≥160 or diastolic ≥100 Current or hx of DVT/PE Major surgery with prolonged immobilization Known thrombogenic mutations Current or hx of ischemic heart disease Current or hx of stroke Valvular heart disease, complicated Migraine with neurological aura SLE with positive or unknown antiphospholipid antibodies Current breast cancer Active viral hepatitis Cirrhosis, severe/decompensated Benign hepatocellular adenoma or malignant liver tumor Chapter 38: Hormone Replacement Therapy and Osteoporosis Etiology, Risk Factors, Diagnosis, and Clinical Findings • Perimenopause is the transition period between reproductive and nonreproductive years o Can last up to 10 years in some women • Extreme variability in frequency and quality of menstrual flow • After menopause (no menses for 12 consecutive months): ovary continues to produce androgens o Converted in fatty tissues into less potent estrogens: estrone and estriol • The changes in estrogen/progestin cause o Vasomotor symptoms (hot flashes), urogenital symptoms (usually occurs later: topical estrogen), bone mineral density is reduced, CVD disorders increase, and the risk for various CAs increases • Use caution when Rx for those with risk factors: CVD • Osteoporosis prevention (HRT is not used solely for the prevention or Tx of osteoporosis, use other medications) • HRT or ERT does not provide protection against cognitive performance, sleep disturbance, or skin changes • Osteoporosis is a worldwide phenomenon • Etiology: • Remodeling: three phases: o Activation: hormone, drug, vitamin, or physical stressor cause formation of osteoclasts o Resorption: Osteoclasts excavate (resorb) bone o Formation: Osteoblasts formation of new bone (fill the cavity left behind by osteoclasts) • For the remodeling process to work osteoclasts and osteoblasts must communicate o A protein called osteoprotegrin (OPG) is key: OPR and OPG-ligand (control bone resorption and growth) • During childhood and adolescence: modeling occurs: bones are becoming larger in size/mass • Later in life: remodeling, existing bone is replaced w/o increase in the total amount of bone o Bones reach peak mass in the early 20’s • If Ca or phosphorus levels are low: PTH stimulates osteoclasts to resorb bone, releases Ca and phosphorus o PTH: stimulates intestines to absorb more CA and the kidneys to active more Vit D to facilitate absorption and reabsorb Ca ▪ Disease processes in any of these organ systems can produce osteoporosis • Estrogens role in bone remodeling: reduce the bone-resorbing action of PTH • Bone loss occurs when the balance between osteoclastic activity and osteoblastic activity is altered • Osteoporosis is a generalized metabolic disease characterized by decreased bone mass r/t the imbalance • Risk factors: increased age, female gender, think small-boned frame, estrogen deficiency greater than 45 y/o, White and Asian ancestry, cigarette smoking, ETOH intake, reduction of estrogen, genetics (Family Hx), diet (low in Ca Vit D with minimal sun exposure), endocrine status, limited physical activity, drugs (Warfarin, TH, glucocorticoids, PPIs, and SSRIs), certain medical conditions (AIDS/HIV, Cushing’s syndrome, DM type 1, MS, hyperparathyroidism, parathyroid tumor, PA, RA, severe liver disease, stroke, thalassemia, thyrotoxicosis) • Risk factors in women: age greater than 62, bodyweight less than 57.7 kg or BMI less than 21, Hx of hip Fx in a parent or personal Hx of Fx after menopause • Risk factors in males: older than 70, glucocorticoid use of 5 mg or greater for 3 months or longer, anticonvulsants (phenobarbital, phenytoin, and carbamazepine), Long term PPI use, bodyweight less than 70 kg, or weight loss more than 10% compared with usual young adult or adult weight in recent years, heavy tobacco or more than 14 ETOH drinks/week, sedentary lifestyle, deficits in vit D or Ca intake, prostate CA therapy, and undiagnosed low testosterone • Diagnosis: • DEXA scan: to determine osteoporosis o Widespread screening is not cost-effective o Screening should be limited to men over 70, women over 65, and younger women if their 10-year fracture risk are equal to or greater than a 65 yo w/o risk o WHO: bone density 2.5 SDs below the average adult peak bone mass • Men typically not Dx until Fx occurs r/t no set screening triggers as there are for women • Clinical findings: • Bone is histologically and biochemically normal but not enough of it to maintain skeletal integrity and mechanical support • Can be generalized (axial skeleton) or regional (appendicular skeleton) • Spongy and compact bone is lost: spongy loss exceeds compact bone lass • End result is Fx: vertebral, hip, and wrist Fx are the most common • Ethnic differences: • African American women: tend to have higher bone mineral density than white women but they are still at significant risk for osteoporosis o Misconception that it does not occur in this population can delay prevention and Tx o R/f hip Fx doubles approx. every 7 years o More likely to die from the hip Fx than white women o Sickle cell and lupus increase the risk o Population: high rates of lactose intolerance (avoidance of dairy) leads to insufficient Ca intake • Asian women: share many of the risk factors that apply to white women o High-risk group o Tend to consume less Ca than whites: lactose intolerance is common in this population o Higher incidence of vertebral Fx • Hispanic women: prevalence is similar to that of white women o Increasing risk of hip Fx o Population consumes less Ca o Twice as likely to develop DM: increases risk for bone loss Optimal time frame for hormone replacement therapy • ERT/HRT: Optimal time frame: up to 5 years o Only start HRT at the time of menopause onset: ADRs increased in those who were postmenopausal for several years • Osteoporosis prevention: directed at the first years of menopause when bone loss peaks Hormone replacement therapy in women with a uterus versus without • No uterus, estrogen only • Intact uterus, estrogen plus progesterone Treatment goals and outcome evaluation • The aim of ERT and HRT is to provide relief from symptoms associated with menopause o Prevent or reduce vasomotor symptoms o Prevent or reduced vaginal atrophy associated with estrogen decline o Reduce the risk for osteoporosis o Ensure that the benefits of ERT/HRT outweigh the risk associated with Tx by providing comprehensive pretreatment evaluation of contraindications and risk, and assisting women in making informed decisions about therapy • For women who elect to use ERT/HRT annual risk, benefits assessment, and QOL (ADRs), and need for continued use should be considered o Annual complete Hx, physical, and mammogram: evaluate for changes in risk profile o LFTs, lipid profile, and repeated annually is abnormal o All post-menopausal bleeding require uterine biopsy r/o CA o Monitor system’s endpoints not levels of hormones ▪ Testing for ERT/HRT hormone levels is unreliable (saliva test) or just not available • For women who present complaining of heavy menses: bleeding causes a drop of 2 g of Hgb o Specialty care: may need a D&C • Any bleeding after menopause: evaluation Hx, physical, pelvic exam, mammogram, pelvic US, and endometrial biopsy, lab tests • In those with urinary incontinence or chronic bladder infections: if blood in the urine and culture is negative, patient will need a urological evaluation o Topical therapy is within the primary care setting • Goals of Treatment: Osteoporosis o Prevention is key o Pharmacological therapy that is inexpensive, safe, and effective o Nonpharmacological therapy and prevention: appropriate exercise program ▪ The best Tx for osteoporosis is prevention ▪ Develop healthy lifestyle while building bone mass is the most cost-effective strategy ▪ Brisk walking, resistance training ▪ Decrease belly fat: this is a risk factor Osteoporosis medication MOA • Estrogen MOA: prevents osteoporosis by reducing the bone-resorbing action of PTH o Estrogen receptors found in bone (validates the hypothesis that estrogen has direct effects on bone remodeling) • Raloxifene (Evista): selective estrogen receptor modulator (SERM) o Approved for preventing and Tx postmenopausal osteoporosis and prevention of ER-positive breast CA o MOA: Selectively actives certain estrogen pathways in bone and has antiestrogen effects on the uterus and breast o Reduces the resorption of bone with less risk for CV effects o Reduces vertebral Fx by 34%: other site Fx reduction is insignificant o Decreases LDL • Bisphosphonates: alendronate (Fosamax)-1st line drug, risedronate (Actonel), ibandronate (Boniva)-2nd line drug, zoledronic acid (Zometa) o MOA: reduce bone resorption by adhering tightly to bone and inhibiting osteoclastic activity o First-line therapy o GI ADRS: empty stomach, glass of water, sit up for 30 min to 1 hr ▪ ADRs are not as significant and life-threatening as estrogens ▪ Very low risk of osteonecrosis of the jaw and subtrochanteric femoral Fx o Risk of AFib (more common in the IV formulation) o In extreme cases can be given with a SERM • Calcitonin: balances PTH by shutting down osteoclastic activity and increasing osteoblastic activity in the presence of hypercalcemia o Low serum Ca levels increase the secretion of endogenous calcitonin o Results in small decrease in serum Ca o All previously marketed calcitonin drugs were pulled off the market due to CA associated risks • Teriparatide (Forteo): synthetic PTH derived from recombinant DNA technology o MOA: bone actions are identical to that of human PTH ▪ Acts to stimulate bone formation ▪ Preferential stimulation of osteoblastic activity over osteoclastic activity o Only taken for 2 years o ADRs: orthostatic hypotension within 4 hours and hypercalcemia o Increased r/f osteosarcoma: must be closely monitored o Contraindicated: Paget’s disease, skeletal radiation, or Hx of bone mets • Denusomab (Prolia): human antibody against the TNF RANKL o Inhibits osteoclast survival and function o Reserved for those who do not respond to bisphosphonate therapy or for CA with bone mets o Injected twice yearly: significant reduction of new vertebral Fx and reduction of hip Fx o Newer and very expensive: reports of osteonecrosis of the jaw, increases in sepsis, and deaths • Calcium and vitamin D o Critical to bone formation and used as complementary agents in prevention and treatment o Hormonal regulation of Ca greatly affects Ca metabolism o Inadequate Vitamin D can decrease absorption of Ca from the intestine and increase PTH: leads to bone loss and Fx o Calcium supplementation without Vitamin D does not reduce Fx rates Contraindications for bisphosphonate therapy o GI bleeding, PUD, GERD, Barrett’s esophagitis Risk and benefits of hormone replacement therapy • ERT o Risks ▪ CV events, stroke, MI, CHD, breast CA, thromboembolic events, endometrial CA (adding progestin helps to reduce this risk), dyslipidemia ▪ Oral estrogens increase CRP o Benefits ▪ Prevention of bone loss (even in low doses): reduced osteoporosis and hip Fx ▪ Reduced symptoms of menopause (hot flashes, vasomotor) ▪ Women may feel more in control of their bodies, improve emotional stability, and reestablish sleep patterns ▪ Improved dyspareunia and vaginal irritation due to atrophy ▪ Relief of urinary urgency ▪ Reduced risk for colon CA T-scores and recommended osteoporosis medication Chapter 44: Sexually Transmitted Infections and Vaginitis Etiology, diagnosis, risk factors, and clinical findings o Etiology: o In women: STIs are a common cause of vaginitis: discharge and vaginal irritation o Not all vaginitis is infectious and that those infected with an STI are often asymptomatic o Genital contact between people is required for transmission of most STIs o Fomite transmission (vibrators, toilet seats, and bath towels) can occur with hardier organisms o Women tend to experience more morbidity than men r/t secretions deposited into the vagina vault during intercourse o Transmission can be facilitated or impeded by vaginal pH, the presence of inflammation caused by spermicides, and the mucosal integrity of either partner o Bacteria and viruses can invade the mucosal lining of the oral, genital, or anal tract o All bodily secretions, especially blood, have the potential to transmit infection o Risk Factors: o Unprotected sex, multiple partners, age, vaginal pH o Identify populations most at risk: adolescent, young adults, gay and bisexual male populations o Race and ethnic disparities: prevalence of chlamydia, gonorrhea, and syphilis all higher in the black population o Some people choose not to get treated: fear of homophobia and social stigma o STI as a Precursor to CA: o HPV (types 16, 18, 31, 33, and 35), Pap tests at regular intervals, 1st at age 21 o Hep C: can be sexually transmitted, increased r/f liver CA o Diagnosing vaginal discharge and vulvar conditions requires examination of the area affected and microscopic examination of vaginal secretions. o May or may not be sexually transmitted o Bacteria and viruses can invade the mucosal lining of the oral, genital, or anal tract o All bodily secretions, especially blood have the potential for transmission. o Diagnosis: Chlamydia and gonorrhea via NAAT and wet mount for BV, trich, and candida Disease/Syndrome Discharge appearance Symptoms pH Diagnostic Test Microscopic Findings Moniliasis Candidiasis White, cruddy + burn, itch <4.5 Culture/KOH Budding yeast hyphae GC/Chlamydiasis Mucopurulent, thick + irritating Normal DNA/culture WBCs >10/hpf Bacterial Vaginosis (BV) Thin, white, odor +itch, odor a big issue >4.5 +amine/culture- change in vaginal flora “clue cells” (coccoid bacteria that obscure epithelial cell borders) Trichomoniasis Blood tinged, purulent +itch, dysuria, foul odor <4.5 Wet mount = + trichomonads Trichomonads >10 WBCs/ hpf cytological Nonspecific, white + pruritus, burn 3.5-4.5 Culture reports change in normal flora 4+ lactobacillus atrophic Scanty, may be white or yellow +burn, sore, cracks <5-7 Culture is negative Several epithelial cells normal White None 3.8-4.2 Not necessary 1-2 lactobacillus Syphilis o Systemic disease caused by Treponema pallidum o Spread by direct contact of the mucosal tissue with infected lesions. o Diagnostic symptoms may present as early as 5 days and as late as 90 days after exposure o Primary syphilis infection presents with ulcer or chancre at the site of infection. o Secondary infection manifestations include rash, mucocutaneous lesions, adenopathy, and neurologic complications o Anogenital condylomata lata are a common symptom of secondary syphilis o Generalized Papulosquamous eruption o Tertiary infections with syphilis present with cardiac, neurological, ophthalmic, auditory, or gummatous lesions. o Neurosyphilis may occur at any stage but most commonly presents in the latent stage. o Treatment of latent syphilis is intended to prevent occurrence or progression of late complications. o Latent infections lack clinical symptoms and are detected by serological testing. o All patients with syphilis should be tested for HIV o All women should be screened for syphilis early in pregnancy: many states mandating screening with the first prenatal visit  if untreated infection of the fetus or perinatal death o Parenteral PCN G (rather than oral penicillin) preferred drug for all stages of syphilis o 2.4 million units IM in a single dose (primary, secondary, and early latent (tertiary)) o 3 weekly doses of benzathine PCN G 2.4 million units (late latent or unknown) o Patients who are allergic to PCN o doxycycline 100 mg BID x 14 days (Less GI upset) o or tetracycline 500 mg QID x 14 days (GI upset) o Parenteral PCN G is the only therapy with documented efficacy for syphilis during pregnancy o women who report penicillin allergy should be desensitized and treated with penicillin Gonorrhea • Gram-negative intracellular diplococcus Neisseria gonorrhoeae transmitted through the urethra, rectum, pharynx, vagina, or eye. • Dx: UA, urethral (men), endocervical or vaginal (women) specimens using nucleic acid amplification test (NAAT) or gonorrhea culture • Men tend to be symptomatic when infected • Many infected women have no symptoms until complications (like PID) have occurred • Incubation period can be 2 days to 2 weeks. • Patients infected with gonorrhea are often co-infected with chlamydia patients being treated for gonorrhea also need treatment for chlamydia (reverse statement is true as well) • Complications: PID, tubal scarring, infertility, ectopic pregnancy, salpingitis, or disseminated gonococcal (GC) infection • Disseminated GC: characterized by pustular dermatitis, asymmetrical arthralgia, tenosynovitis, or septic arthritis • Infected pregnant women ARF endometritis after procedures • 1st choice: combination treatment with ceftriaxone IM or oral cefixime, plus either oral azithromycin(preferred) or doxycycline. o Ceftriaxone 250 mg IM injection x 1 PLUS azithromycin 1 g PO x 1 o OR doxycycline 100 mg oral BID x 7day • Alternative: cefixime 400 mg PO x 1 PLUS azithromycin 1g oral single dose • OR doxycycline 100mg oral BID x 7day (Test of cure in 1 week) • For cephalosporin allergyincrease azithromycin to 2gm oral single dose (monitor for NV) • Persons with severe cephalosporin allergies should be treated only after consultation with an infectious disease specialist • Sexual partners require treatment, even without symptoms, if they had sexual contact with the patient during the 60 days preceding the onset of symptoms or diagnosis of the infection • To prevent reinfection: patients and their partners should abstain from intercourse until therapy is completed. • Women have a high rate of reinfection in the 6 months after treatment. • Recommendations include retesting men and women 3 months after treatment regardless of whether the sex partner has been treated Chlamydia • Chlamydia trachomatis most common STI in US • Often asymptomatic: causes serious sequelae (PID, ectopic pregnancy, and infertility) • Rate of infection in women double than of men • Dual therapy for gonorrhea and chlamydial infections is the treatment standard for both men and women • First Choice: o Azithromycin 1g orally in single dose o OR doxycycline 100mg BID x 7 days o 1st choice in pregnancy: azithromycin 1g orally in single dose o OR amoxicillin 500mg orally TID x 7days • Alternative therapy: • Erythromycin base 500 mg PO QID for 7 days • Or erythromycin ethylsuccinate 800 mg QUID x 7 days • Or Ofloxacin 300 mg BID x 7 days • Or levofloxacin 500 mg daily x 7 days • Except in pregnant women or non-compliance: test of cure is not recommended • Instruct to abstain from sex for 7 days after single-dose therapy or until completion of a 7-day regimen. • All sexual partners in the past 60 days should be tested and treated. • The most recent partner, even if more than 60 days has elapsed, should be evaluated. • Expedited partner therapy is recommended to decrease reinfection Chancroid o Haemophilus ducreyi: endemic in some areas of the US o Co-infection with HIV, syphilis, and HSV can occur o Dx is difficult r/t lack of sensitive testing media o Treatment is initiated when 1) one or more painful ulcers, 2) negative for syphilis and HSV and 3) the appearance of ulcers with suppurative inguinal adenopathy o Usually starts as a small papule that rapidly becomes pustular and ulcerates o The ulcer enlarges, begins to develop ragged and uneven borders and a erythematous rim o The lesions are tender o First line treatment: Azithromycin 1 g PO x 1 o Or ceftriaxone 250 mg x 1 IM o Or Ciprofloxacin 500 mg PO BID x 3 days (not for use in patients under 18, pregnant, or lactating) o Or erythromycin base 500 mg PO TID x 7 days o Patients should be re-examined 3 to 7 days after therapy is started: usually improve after 3 days-2 weeks o There may be significant scarring o Poor response to Tx may indicate co-infection with another STI, HIV, or the pathogen is resistant o Include HIV testing in evaluation at start of therapy and at 3 month intervals o Sexual partners should be examined and treated if they had sexual contact within 10 days before Dx Granuloma Inguinale (Donovanosis) o Rare, genital ulcerative disease caused by intracellular gram negative Klebsiella granulomatis o Painless, slow progressing ulcers w/o lymphadenopathy: bleed easily o Usually affects the skin and mucous membranes in the genital region: results in nodular lesions that progress to large, beefy lesions (difficult to heal) o Progressively expand and locally destructive o Can be spread by sex, fecal route, or passage through the birth canal o First line: Doxycycline 100 mg PO BID x 3 weeks or until ulcers have completely healed o Alternative: Cipro 750 mg PO BID x 3 weeks o Or erythromycin base 500 mg PO QUID x 3 weeks o Or azithromycin 1 g PO weekly for 3 weeks o Or trimethoprim/sulfamethoxazole one double strength (160/800mg) tablet PO BID x 3 weeks o Relapse is common w/n 6 to 18 months despite best therapy Lymphogranuloma Venereum o LGV: caused by C. trachomatis serovars L1, L2 or L3 o Unilateral tender lymphadenopathy and self-limited genital ulcers o Present with: Proctocolitis in men and perianal inflammation in women o Complication: strictures or fistulas o Can lead to an invasive systemic infection o If not treated early: chronic, colorectal fistulas and strictures o Dx is made serologically o First line: Doxycycline 100 mg PO BID for 21 days (do not use in pregnant or lactating women) o Alternative Tx: Erythromycin base 500 mg PO QID x 21 days (use in pregnant women) o Partners from the last 60 days should be treated Bacterial vaginosis o BV is caused by a replacement of the normal hydrogen-peroxide-producing lactobacilli present in the vaginal flora by an overgrowth of organisms such as Prevotella spp., Mobiluncus spp., Gardnerella vaginalis, or Mycoplasma hominis. o The normally acidic vaginal environment is important in preventing the overgrowth of other organisms, which tends to occur when the pH is raised. o associated with having multiple sex partners, douching, and lack of vaginal lactobacilli. o Diagnosed by the use of clinical criteria or Gram's stain criteria and must include three of the four following signs and symptoms o a homogeneous, white, noninflammatory discharge that smoothly coats the vaginal walls o a vaginal pH greater than 4.5; o a positive whiff test (fishy odor) with 10% potassium hydroxide (KOH); or o the presence of “clue cells” (vaginal epithelial cell peppered with coccoid bacteria) under high- power microscopy. o Asymptomatic women tend to spontaneously improve, treatment should be reserved for symptomatic women o Treatment of asymptomatic, pregnant women preparing to undergo termination of pregnancy is recommended. o First line: Metronidazole 500 mg oral BID x 7days o OR clindamycin cream 2% 5 g intravaginally at bedtime x 7 days (may be less effective than metronidazole) o OR metronidazole gel 0.75% 5g intravaginally at bedtime x 5 days o alternative treatment tinidazole 2g orally x 2 days o or tinidazole 1 gm orally for 5 days o or clindamycin 300 mg PO BID x 7 days o or clindamycin ovules 100 mg intravaginally HS x 3 days o Clindamycin cream can weaken condoms, especially those made with latex o Routine Tx of sexual partners not recommended: not shown to reduce recurring infections o BV during pregnancy ARF premature rupture of membranes, preterm labor, preterm birth, and postpartum endometriosis: should be screened at Dx of pregnancy o Don’t use topical agents in pregnancy: ADRs from clindamycin cream Vulvovaginal candidiasis (VVC) o May be caused by several yeast species, although Candida albicans is the most common o Recent history of antibiotic use is most often the cause. o Patient with chronic recurrent infections should be screened for diabetes o Presents w/ vulvar itching with a Hx of cottage cheese like discharge o Dx made with a wet preparation, Gram’s stain, or culture o Low vaginal pH (4-4.5) can help to distinguish candida from other vaginal infections o The azoles as a drug class are among the most effective treatments o oral and topical treatments equally effective o Oral azole agents stimulate the cytochrome P450 (CYP450) enzyme system in the liver and have potential drug interactions with calcium channel antagonists, cisapride, warfarin, oral hypoglycemic agents, phenytoin, protease inhibitors, theophylline, and rifampin. o Recommended creams and suppositories for the treatment of VVC are oil-based and may weaken latex condoms and diaphragms. o Self-medication with over-the-counter (OTC) preparations is advised only for women who have been previously diagnosed with VVC and who have a recurrence of the same symptoms. o VVC often occurs during pregnancy. o Only topical azole therapies, applied for 7 days, are recommended for use by pregnant, symptomatic women. o Oral azoles are contraindicated in pregnancy. o In severe cases, 7 to 14 days of topical therapy, or fluconazole 100 mg, 150 mg, or 200 mg orally every third day for a total of 3 doses o Oral fluconazole (100 mg, 150 mg, or 200 mg) weekly for 6 months can also be used for suppression of the infection Herpes Simplex Virus Type 1 (HSV1) and Herpes Simplex Virus Type 2 (HSV2) o Genital herpes is incurable and recurrent: exacerbation and remission are highly variable o Two serotypes of HSV: HSV-1 and HSV-2. o Most recurrences are a result of HSV-2. o Most persons infected with HSV-2 have not been diagnosed and shed the virus in the genital tract without obvious symptoms o HSV-2: can cause recurrent BV o Extremely painful vesicles during outbreaks: restoring QOL is important o Suppressive for patients experiencing six or more outbreaks each year. o Most patients experience fewer episodes after 1 year of suppressive therapy o Suppressive therapy reduces, but does not eliminate, subclinical viral shedding. o Systemic treatment with oral acyclovir, famciclovir, and valacyclovir are the mainstays of treatment for genital herpes. o First Episode: Acyclovir 400 mg orally 3 x daily 7-10 days • OR acyclovir 200 mg orally 5 x daily x 7-10 days • OR famciclovir 250 mg orally 3 x daily 7-10 days • OR valacyclovir 1gm BID 7-10 days • Note: Tx may be extended if complete healing does not occur after 10 days of therapy o Episodic Therapy: Acyclovir 400 mg PO TID x 5 days • Or acyclovir 800 mg PO BID x 5 days • Or Acyclovir 800 mg PO TID x 2 days • Or famciclovir 125 mg BID x 5 days • Or famciclovir 1 g BID x 1 day • Or valacyclovir 500 mg PO BID x 3 days • Or valacyclovir 1 g PO daily x 5 days o Antiviral therapy for recurrent genital herpes can be administered episodically or continuously as suppressive therapy. o Episodic therapy is effective in shortening the duration of outbreaks if started within the first 24 hours of lesion outbreaks or during the prodromal phase (burning, itching, and tingling) that often precedes outbreaks. o Provide an Rx for patient to self-treat when appropriate o Consider reassessing the need for suppressive therapy annually by temporarily discontinuing the drug to see if outbreaks occur o For those with HIV: drug choices and dosing are the same as for those with a first episode or initial outbreak o Episodic or suppressive therapy with PO antiviral agents is beneficial o Perinatal transmission of HSV is low but there is concern for transmission to the neonate at birth o Safety of acyclovir, valacyclovir, and famciclovir in pregnancy has not been well established o However, acyclovir can be given PO to pregnant women with first episode or severe recurrent o HSV can be life threatening to the newborn: if infected near delivery date (use protection) o C-section does not ensure protection from HSV2 infection o Pregnant women with HSV require careful management by an OB Human Papillomavirus (HPV) o By age 50 more than 80% of American women will have contracted at least one strain of genital HPV o Men and women can be carriers o Most infections are asymptomatic but can lead to anogenital Cas o Aggressive screening and Tx is warranted: especially high risk individuals o More than 40 viral types found in men and women o HPV types 6 and 11 are low risk and cause 90% of all genital warts o Patients with visible warts may be infected with several types simultaneously o HPV types 16, 18, 31, 33, and 35: associated with cervical neoplasia and detected on Pap smear o Types 16 and 18: cause approx. 70% of cervical CA o Lesions may be penile, scrotal, cervical, vaginal, urethral, oral, or perianal o HPV Vaginal Tx: TCA or BCA 80-90% in small amounts should be applied directly to wart area o Use talc, sodium bicarb, or liquid soap to remove unreacted acid o Repeat weekly if needed o Alternative Vaginal Tx: cervical lesion: biopsy lesions and refer for Tx o HPV External Genital: Patient-applied Tx (can be expensive) o Podofilox 0.5% solution or gel: apply with cotton swab to visible warts BID x 3 days, 4 days no Tx, Repeat for up to four cycles o Imiquimod 5% cream: apply once daily HS three times a week for up to 16 weeks: wash Tx area with soap/water 6 to 10 hours after application o Sinecatechin 15% ointment: Apply 0.5 cm of ointment to each wart TID. Do not use for more than 16 weeks o HPV External Genital: Provider-applied Tx o Cryotherapy (only NP who is specially trained if not refer) o Pedophyllin resin 10-25% applied to each wart. Wash preparation off 1-4 hours after application, can repeat weekly if needed (do not use if the patient is pregnant) o Trichloroacetic acid (TCA) 10-25% or bichloroacetic acid (BCA) 80-90%, apply weekly if needed, neutralize acid with soap or sodium bicarb o Surgical removal o Urethral meatus warts: Cryotherapy with liquid nitrogen o or podophyllin 10-25% in compound tincture of benzoin: repeat weekly if needed (do not use in pregnancy) o Anal warts: cryotherapy with liquid nitrogen Genital warts o Usually flat, papular, or pedunculated growths on the genital mucosa. o Dx: made by visual inspection and may be confirmed by biopsy o Biopsy is recommended in certain circumstances o (for example, if the diagnosis is uncertain; the lesions do not respond to standard therapy; the disease worsens during therapy; the patient is immunocompromised; or warts are pigmented, indurated, fixed, bleeding, or ulcerated). o In addition to the external genitalia (penis, vulva, scrotum, perineum, and perianal skin), genital warts can occur on the uterine cervix and in the vagina, urethra, anus, and mouth o HPV types 16, 18, 31, 33, and 35 are found occasionally in visible genital warts patients who have visible genital warts are frequently infected simultaneously with multiple HPV types o Management of HPV involves removing warts if the patient is symptomatic due to location, size, and number of warts and monitoring for the development of cancer cells especially in patients with abnormal Pap test results. o Goal of Tx: removal of the warts o If left untreated, visible genital warts might resolve spontaneously, remain unchanged, or increase in size or number. o Treatment possibly reduces, but does not eliminate, HPV infection o Choice of therapy: symptoms, number of warts, location of the warts, and size o Cost, patient preference, experience of the provider, and access to wart morphology o Generally warts on moist surfaces or areas such as the armpits, under pendulous breasts, groin, creases of the neck, or skin folds respond better to topical Tx o The majority of patients will need more than one treatment and most respond within 3 months of therapy. o Treatment regimens are classified as either patient-applied or provider-applied. o Surgical referral should be considered by the nurse practitioner when the size, number of warts, ineffective response to topical treatments, and location of warts make it difficult to treat effectively. o Imiquimod, podophyllin, and podofilox should not be used during pregnancy. o Because genital warts can proliferate and become friable during pregnancy, many specialists recommend their removal during pregnancy. o There is no evidence that on Tx is superior to another o Tx regimen should be changed if warts do not resolve in three to six treatments o Tx of lesions may cause chronic pain syndromes of the vulva: extremely rare o Two vaccines to prevent HPV infection are currently on the market: Gardasil and Cervarix o Cervarix (a bivalent vaccine) provides protection against HPV types 16 and 18 and is licensed for use in females 11 to 25 years of age. o Gardasil (a quadrivalent vaccine) provides protection from HPV types 6, 11, 16, and 18 and is licensed for both males and females ages 9 to 26 years. o Can be expensive: financial assistance through Vaccines for Children program o A nine-valent vaccine is currently undergoing FDA review: against 6, 11, 16, 18, 31, 33, 45, 52, 58 o Cervical lesions should be biopsied Trichomonas • Trichomonas spp.  protozoa • Men are rarely symptomatic but may harbor Trichomonas in the prostate gland for years if left untreated. • Many women have a malodorous, yellow-green vaginal discharge with vulvar irritation. o However, some women have minimal discharge and may be asymptomatic. o Increased vaginal pH: increased r/f Trichomonas infection in men • Rapid testing: increased sensitivity and specificity compared to microscopy • First Line: oral metronidazole (avoid ETOH: disulfiram reaction) or tinidazole o Metronidazole 2 gm orally as single dose o OR tinidazole 2gm orally as a single dose • Alternative Treatment: metronidazole 500mg orally BID x 7 days • Rescreening in 3 months after treatment may be considered in sexually active women, due to a high (17%) reinfection rate • Pregnant women who are infected with vaginal trichomoniasis are at risk of preterm labor, premature rupture of membranes, and low birth weight o Treatment for symptomatic women or asymptomatic women in any stage of pregnancy is oral metronidazole o Pregnant women should be followed up in 1 month. • Sexual partners should be treated for trichomoniasis and patients should be instructed to abstain from intercourse until both partners are treated and asymptomatic Pediculosis Pubis (Genital Lice) o Pediculosis pubis: commonly called crabs o Ectoparasitic infection: Tx based on where the lice are found (scalp, body, pubic hair) o Generally sexually transmitted: teenagers and young adults most commonly affected o Pubic itching is the primary complaint o Organism can be resistant to treatment o Retreatment in one week of symptomatic individuals is recommended o First Choice: permethrin 1% cream rinse applied to affected areas, wash off in 10 mins o Or pyrethrins with piperonyl butoxide applied to affected areas, wash off in 10 mins o Advise the patient to thoroughly saturate hair with the medication. o Leave medication on for 10 minutes, then thoroughly rinse off with water. o Dry off with a clean towel o Reapply in 7 days if needed o Alternative regimens: Malathion 0.5 % lotion applied for 8-12 hours, then wash off o Or ivermectin 250 mcg/kg PO repeated in 2 weeks o Ivermectin contraindicated in pregnancy, lactation, and children, pyrethrin lotion, or shampoo. o Malathion 0.5% lotion or oral ivermectin are reasonable options and may be used when treatment failure is believed to have occurred or if resistance is suspected o Sexual partners should be treated at the same time to avoid reinfection. o Treatment of nonsexual contacts is not needed. o Decontamination of household and personal items with hot washing or dry-cleaning is usually adequate. o Pubic lice cannot live away from the body for more than 72 hours Scabies o Sarcoptes scabiei is the parasite involved in scabies o Predominant symptom: severe, nocturnal pruritis o After a person is infested with scabies for the first time, sensitization can take up to several weeks to develop. o Pruritus can then occur within 24 hours following a subsequent infestation. o Direct contact from person to person is needed to contract the parasite o Can be passed sexually in adults, but adults and particularly children may become infected by sleeping on infected sheets in motels and hotels o If via sexual contact, the skin adjacent to the nipples, periumbilical area, waist, genitalia, buttocks, and thighs is commonly affected. o Diagnosis: skin scraping can be obtained however Dx can be made by H&P alone o First Choice: Permethrin 5% cream (Elimite, Acticin) is the drug of choice for the treatment of scabies, especially in pregnant women or young children. ▪ The cream is massaged into the skin from the neck to the soles of the feet. ▪ Left on for 8 to 14 hours and then washed off in the shower. ▪ A repeat application in 1 to 2 weeks is recommended o Or Oral ivermectin is another alternative: 200 mcg/kg PO, repeated in 2 weeks ▪ The use of ivermectin is not recommended in pregnant or lactating women, and children. ▪ Ivermectin is considered the drug of choice for nursing home outbreaks or when topical treatment is not practical. ▪ Ivermectin may need to be repeated in 2 weeks. ▪ Ivermectin is not active against nits. o Alternative Choice: Lindane 1% 1 oz lotion for 30 mg of cream applied in a think layer to affected areas o Wash off after 8 hours o ADRs: r/f toxicity and r/f seizures o Do not use immediately after a bath or shower or in perrons with dermatitis, women who are pregnant/lactating and children younger than 2 y/o o In addition to eradicating the mite, treatment with antihistamines and topical low- or medium-dose corticosteroids to control the itching can be helpful o Bedding and clothing should be decontaminated or removed from body contact for at least 72 hours. o Education: The rash and pruritus of scabies can persist for up to 2 weeks after treatment o Sexual contacts and family members may be in the incubation period (4 weeks). o For this reason, all family members and close contacts need simultaneous treatment to prevent recurrence Diagnosis testing, outcome evaluation and recommendations for rescreening Most of these answers are listed in the above sections gonorrhea o Dx: UA, urethral (men), endocervical or vaginal (women) specimens using nucleic acid amplification test (NAAT) or gonorrhea culture o Sexual partners require treatment, even without symptoms, if they had sexual contact with the patient during the 60 days preceding the onset of symptoms or diagnosis of the infection o To prevent reinfection: patients and their partners should abstain from intercourse until therapy is completed. o Women have a high rate of reinfection in the 6 months after treatment. o Retesting men and women 3 months after treatment regardless of whether the sex partner has been treated o Rescreen in 1 year is recommended o A test-of-cure in 1 week if oral cefixime is used to treat infections of the cervix, urethra, and rectum chlamydia o Except in pregnant women or in those who have been noncompliant, a test-of-cure is not recommended for persons treated with the recommended or alternative regimens o High-risk, pregnant individuals should be retested during the third trimester to decrease the possibility of the infection being passed during delivery. o Pregnant women should have follow-up testing after treatment and should be retested 3 months Chancroid: o Patients should be re-examined 3 to 7 days after therapy is started: usually improve after 3 days-2 weeks o Follow up is recommended because lack of response may indicate HIV (cofactor for HIB transmission) o Sexual partners should be examined and Tx 10 days prior to Dx Granuloma Inguinale (Donovanosis) o Sex partners should have clinical s/s prior to initiation of therapy o May take many months for the lesions to heal: follow up Lymphogranuloma Venereum o Patients may need further testing to r/o coexisting STIs o Sexual partners with contact in the last 60 days should be examined and treated Trichomonas: o Rescreening in 3 months after treatment may be considered in sexually active women, due to a high (17%) reinfection rate o Sexual partners should be treated at the same time to avoid reinfection. o Dx: rapid testing or microscopy (must be skilled in the procedure, done very quickly after collecting sample to look for trichomonas movement on a wet plate prep) Treatment guidelines o Treatment of STIs is based on national guidelines recommended by the CDC o CDC’s Sexually Transmitted Diseases Treatment Guidelines o See page 1220 Table 44-2 pathogens, first choice meds, and alternative choice meds o The guidelines are presented earlier in this study guide Metronidazole education o When mixed with ETOH, metronidazole has the potential to cause disulfiram-like reactions o ETOH should not be consumed during or for at least 1 day following completion of therapy Empirical Tx of PID (not on the study guide but I thought this was important) o Should be imitated if the following minimum criteria are met and no other cause for the symptoms (appendicitis) can be found o Uterine/adnexal tenderness o Cervical motion tenderness o Oral temp higher than 38.3 o Abnormal cervical or vaginal mucopurulent discharge o Presence of WBCs upon saline microscopy examination o Elevated ESR o Elevated CRP o Lab documented cervical infection with gonorrhea or chlamydia o Multidrug regimen that provides empiric, broad-spectrum coverage of the most likely pathogens o Begin Tx immediately to avoid life altering compactions o Tx as if patient has all types of infection o Obtain a preg test o All pregnant women with PID should be hospitalized and Tx with IV ABX o May need IV ABX and/or hospitalization if temp is high or PO drugs cannot be tolerated (NV) o OP Tx for mild to moderate PID: o Cefoxitin 2 g x 1 dose plus probenecid 1 g PO x 1 dose plus doxycycline 100 mg PO BID x 14 days o Metronidazole PO 500 mg BID x 14 may also be added to the regimen o Expert consultation when Tx PCN allergic patient with PID o Hospitalize and refer all patients with severe PID as well as all pregnant women with PID regardless of the severity of infection o Transition from IV to PO w/in 24 hours of clinical improvement o Consider changing drugs from PO to IV in those who fail to respond to PO therapy w/in 72 hours o The PID patient should show significant improvement w/in 3 days of starting therapy o If patient does not improve usually require hospitalization, additional Dx testing, and surgical intervention NR 566 Week 7 Study Guide Chapter 48: Women as Patients Pharmacokinetic gender differences between men and women • These three medications have shown differences that are clearly significant between men and women • Propranolol (Inderal): early drug that showed a clear gender difference in metabolism (clearance) o PO doses: significantly higher (63%) rate of clearance in men than women o IV doses do not do this: indicates hepatic first pass metabolism o Women: show a significantly greater clinical response to PO doses • Verapamil (Isoptin, Calan): PO clears more quickly in men than women based on CYP3A4 o Higher absolute bioavailability in women causes greater pharmacodynamic effects on BP and HR in women • Erythromycin: more effective in women o More rapidly cleared in women than men r/t CYP 3A4 Pharmacokinetic Parameter Sex-Based Difference Absorption and bioavailability • Gastric emptying time is slower in females, mainly related to the effects of estrogen. Drugs absorbed in the stomach will have longer exposure to absorption sites. • Gastric levels of alcohol dehydrogenase are lower in females. Plasma concentrations are greater in females than males after ingestion of similar amounts of alcohol. • Gastric acid secretion, pH, osmolality, electrolyte concentrations, and levels Pharmacokinetic Parameter Sex-Based Difference of bile acids and proteins do not vary significantly between sexes. Distribution • Females have lower body weights and BMI than males. • Females have a higher proportion of body fat. Lipophilic drugs are more readily absorbed and have relatively greater volumes of distribution than hydrophilic drugs. • Plasma volume is lower in females. Drugs with high volumes of distribution will be more concentrated in the plasma of females. • Organ blood flow is lower in females. • Estrogen is distributed attached to a serum-binding globulin. Exogenous estrogens increase levels of many serum-binding globulins such as corticosteroid-binding globulin and thyroxine-binding globulin resulting in less free drug. Metabolism • Studies have been inconsistent in showing differences in CYP450 substrates; the general trend is toward high rates of metabolism for CYP450 3A4 substrates and lower rates for 1A2 and 2D6 substrates. • Females have lower levels of p-glycoprotein and higher rates of drug clearance for drugs that are substrates of p-glycoprotein. Excretion • Gender differences in rates of renal excretion of most drugs are probably more related to simple weight differences. Renal clearance of drugs that are not actively secreted or reabsorbed is dependent on the glomerular filtration rate, which is directly proportional to weight and consequently higher (on average) in men. • Drugs that are actively secreted by the kidney may show gender differences, but further study is required to demonstrate this difference. Recommended Calcium and Vitamin D Intake for Adolescents and Adults • Adolescents o 1,300 mg of calcium daily is recommended for females aged 9 through 18 years, ▪ drinking three cups of low-fat or skim milk & consuming 8 ounces of low-fat yogurt. ▪ If unable to meet the recommended daily amount of calcium through diet alone, calcium carbonate (Tums, Caltrate, or Viactiv) with food to maximize absorption. o vitamin D is required for optimal calcium absorption ▪ a daily multivitamin that includes at least 400 IU vitamin D should also be taken • Adults o 19-50 years old: 1,000 mg/day o 51-70 y/o: 1,000 mg/day o 71+: 1,200 mg/day Non-Pharmaceutical Treatment of Symptoms Associated with Menopause • Many options for managing menopausal symptoms are available exercise, relaxation techniques, massage therapy, acupuncture, herbs, or pharmaceuticals (such as HRT) • Some women unable or choose not to take HRT and have turned to phytoestrogenssubstances with estrogen-like properties found in o certain herbs: ginseng, black cohosh, dong quai, fenugreek, and licorice o certain foods—especially carrots, yams, and soy products—varying efficacy • Nutritional supplements for a diet rich in calcium and vitamins E, D, and B complex. • Menopausal women should avoid foods such as caffeine, alcohol, and spicy foods that can trigger vasomotor symptoms • Table 48-3: Alternative Therapies for Menopause Symptoms o Hot flashes: pharmaceuticals o Reduced libido: talk with the patient, sexual counseling/therapy, investigate sleep issues, screen for depression, weight loss, suggest using alternative therapies for vaginal dryness or soreness o Mood changes: Meditation, yoga, prayer, adequate sleep, stress management techniques o Sleep disturbances: pharms, exercise, decreased intake of or avoid stimulants, avoid alcohol, meditate, pray o Stress incontinence: decrease intake of caffeine beverages and diuretics, Kegel exercises, treat/reduce constipation, bladder training program o Vaginal dryness or soreness: reduce or avoid use of medication such as antihistamines, decongestants, anticholinergics, and diuretics, alternative therapies for hot flashes (increases epithelial lining of vaginal tissues), water soluble lubricants o Weight fluctuations: Exercise 30 to 60 minutes daily • Emphasis in the care of women in their 50s should be on health promotion and prevention of the diseases of older age o After menopause the lipid profile of women changes. The decline in endogenous estrogen removes a protective physiological mechanism that supports higher levels of HDL and lowers LDL • Loss of estrogen places a woman at increased risk for CAD, HTN, and stroke o Primary prevention studies demonstrate that diets high in complex carbohydrates, fiber, and protein and low in animal fat are best • thirty minutes of moderate physical activity for at least 6 days per week is recommended for best health • several years of gradual decline or erratic levels of endogenous estrogen precede cessation of ovarian function. • All women experience changes in their secondary sexual characteristics. Some women barely notice vasomotor instability, whereas hot flashes and insomnia incapacitate others Treatment for Primary Dysmenorrhea • Pain shortly before or during menstruation, one of the most common gynecological complaints • more common in women under 20 • Primary dysmenorrhea  due to increased myometrial activity, with contractions induced by prostaglandins in the second half of the menstrual cycle. • NSAIDS are the first line of drug treatment for women not desiring contraception (OTC doses may be suboptimal) o particularly effective if begun 2 to 3 days before menses or at the first sign of bleeding o Preparations containing acetaminophen, which is not an NSAID, are ineffective because of the absence of anti-prostaglandin properties. • For women who want contraception: PO contraceptives are a good therapeutic choice • Decreased prostaglandin synthesis results from an atrophic endometrium • Complementary and alternative medicines shown to improve symptoms of dysmenorrhea  thiamine (vitamin B1), magnesium, vitamin E, and omega-3 fatty acids • Comfort measures,  heat (poultices or heating pads), massage/effleurage, guided imagery, progressive relaxation, yoga, exercise, and meditation • Decreasing dietary intake of salt, sugar, and red meat in the luteal phase and increasing water intake may reduce edema • If dysmenorrheal discomfort is NOT relieved by one of the NSAIDS, further investigation into the cause of symptoms is required. • Secondary dysmenorrhea usually develops later in a woman’s life (after age 25) and often due to pelvic pathology o such as adenomyosis, endometriosis, pelvic inflammatory disease (PID), endometrial polyps, and myomas (fibroids) o Many of the relief measures for primary dysmenorrhea are also helpful for women with secondary dysmenorrhea, but treatment is aimed at removal of the underlying pathology. Patient Education for NSAID Administration to Treat Dysmenorrhea • Particularly effective if begun 2 to 3 days before menses or at the first sign of bleeding • Preparations containing acetaminophen, which is not an NSAID, are ineffective because of the absence of anti-prostaglandin properties. • Take with at least 8 oz of water • GI ADRs: Risk for GI bleeding, gastric ulcers • Do not take more than prescribed Rationale Drug Selection for PMDD • To meet diagnostic criteria, patients must exhibit five or more symptoms, including at least one “core” symptom. o core symptoms  markedly depressed moods, heightened anxiety/tension/edginess/nervousness, affective lability, persistent and marked anger and irritability o other symptoms, such as decreased interest in usual activities, marked lack of energy (fatigue, lethargy), hypersomnia or insomnia, difficulty concentrating, appetite changes or cravings, and a subjective sense of being overwhelmed or “out of control • Symptoms are cyclical, occurring during the luteal phase of the menstrual cycle and are significantly reduced or disappear completely during menstruation. • Nutritional, herbal, and drug therapies play a role in this complex disorder • First-line therapy for PMDD  SSRIs. o fluoxetine, paroxetine, and sertraline o Both luteal phase and continuous use are beneficial, continuous use may be better when cycles are not predictable, such as during peri-menopause • Second-line drug therapy includes tricyclic clomipramine and benzodiazepine. o tricyclic clomipramine has anticholinergic side effects o benzodiazepine is associated with tolerance if used long term. • Gonadotropin releasing hormone agonist and danazol (a weak androgen): have been used to Tx PMDD o Inhibit FSH and LH, suppressing ovarian steroid hormone production and preventing ovulation o Equivalent to a medically induced oophorectomy: often requires addback estrogen and/or progesterone o Specialty care Therapy Dosing Symptom Improvement Nutritional Supplements Calcium carbonate 1200-1600mg/d Core symptoms Magnesium up to 500mg/d Bloating Tryptophan Up to 6 gm/d Insomnia; affective symptoms Vitamin B6 Up to 100mg/d Core symptoms; depression Vitamin E 200-400 mg/d Core symptoms; depression Herbals Evening primrose oil 500mg daily to 1000mg TID Anti-inflammatory; breast tenderness Chaste tree berry 30-40 mg/d Breast engorgement Drugs (SSRIs) Citalopram (off-label) 10-30mg/d All symptoms; fewer side effects than other SSRIs Fluoxetine 20 mg/d All symptoms; sexual side effects Paroxetine 10-30 mg/d All symptoms; GI and sexual side effects Sertraline 50-150 mg/d All symptoms; GI and sexual side effects Drugs (other) Alprazolam 0.375-1.5 mg/d Anxiety and other affective symptoms Bromocriptine Up to 2.5 mg TID Breast engorgement Clomipramine 25-75 mg/d All symptoms; anticholinergic effects Ibuprofen 500-1000 mg/d Pain; breast engorgement Spironolactone 100 mg/d Water retention LGBTQ Population • Social stress of being different (fear of homophobia): leads to smoking, ETOH use, depression, HIB/AIDS, physical violence, and attempted suicide • Ignorance of healthcare providers: some think that lesbians cannot get STIs or do not need Pap Smears • Lesbians and bisexual women: less likely to have health insurance to access to health services • Providers have a profession and moral responsibility to treat all people with respect and dignity regardless of sexual orientation or preference • Strategies to promote a positive environment o Be aware of your own biases and be open, knowledgeable, and comfortable with sexual differences o Create a safe atmosphere for disclosure of information for appropriate Dx, Tx, and information o Have health information about lesbian and bisexual health readily available o Avoid using euphemisms such as “special friend” when asking about a client’s partner Chapter 49: Men as Patients Pharmacokinetic Gender Differences Between Men and Women • See the table above for Ch 48 Causes of Primary and Secondary Hypogonadism • Hypogonadism refers to the failure of the testes to produce androgen , sperm, or both • One of the main causes of male infertility, due to lack of testosterone, leading to low sex drive and ED • Patho – Given that testosterone is largely protein bound, any disturbance in that binding or the amount of protein available for binding affects the testosterone that is available for use by sex organs. • Cause can be congenital: results in the birth of a child whose sex is not entirely clear • Primary hypogonadism (testicular failure) – characterized by low testosterone and elevated gonadotropins o Common causes – CA Tx, hemochromatosis, Klinefelter syndrome, Mumps orchitis, normal aging, testicular trauma, undescended testicles • Secondary hypogonadism (hypothalamic – pituitary failure) – characterized by low testosterone and low or normal gonadotropins o Common causes – HIV/AIDS, inflammatory disease, medications, obesity, opioid uses, pituitary disorders, Kallmann syndrome • Subclinical (compensated) – seen in older men, typically have high gonadotropins and normal testosterone levels Testosterone Therapy and Monitoring • Restoring testosterone level WNL with TRT can improve many of the effects of hypogonadism • Table 49.1 Post-pubertal Hypogonadal-Related Problems and Effectiveness of TRT o Decreased BMD: increases BMD o Decreased cognitive function: Increases cognitive function o Decreases energy, mood, and QOL: Increased energy, mood, and QOL o ED: Increased sexual function o Decreased Hct and Hgb concentrations: Increased Hct and Hgb concentrations o Decreases muscle mass: Increased muscle mass o Decreased prostate glands: Increased prostate gland size-prostate symptom score, urine flow rate, and post voiding residual urine in the bladder after voided did not change significantly o Decreased sexual desire: Increased libido o Dyslipidemia impaired glucose regulation, HTN, insulin resistance, and obesity: Improvement of insulin sensitivity o CVD: Increases coronary blood flow in patients with CAD Dosages: • Buccal (Striant): 30 mg to gums BID • IM Depot esters-testosterone cypionate (Depo-testosterone), enanthate (Delatestryl), propionate, phenylpropionate, isocaproate, decanoate, acetate: 50-400mg every 2-4 wks • SQ pellets (Testopel): 10-15 pellets every 3-6 months • Topical skin patch (Androderm): 5mg/d applied once daily to dry, intact skin of shoulders, upper arm, or abdomen • Topical Gel (Androgel 1% or 1.62%): 5-10mg (1-2 packets/tubes or 1-8 pumps depending on concentration) once daily to clean, dry, intact skin of shoulders, upper arm, or abdomen • Topical gel (Fortesta 2%): 40mg (4 pumps) applied once daily to inner and anterior thighs • Topical genital skin patch (Testoderm): 6mg/d patch applied to scrotal area and worn for 22-24 hrs • Topical liquid (Axiron): 30-90 mg (2-4 pumps) once daily to axillae • Topical non-genital skin patch (Androderm): 5 mg/day applied to clean, dry, intact skin of shoulders, upper arm, or ABD • Typically, the depot esters are administered parenterally by IM or SQ route • Transdermal and buccal routes are also available • If IM route is chosen, patients should be taught to self-administer • If injected pellets SQ, pellets can fall out and absorption may differ • Patches can be applied daily, rotating sites, and watching for skin disruptions • Liquid, topical gel can be applied o Care should be used with secondary contact of women and children. o Clothing should cover area of application. o Careful and thorough hand washing after application o 3-4 hours after application: all med is absorbed then wash the area before direct skin contact with others • Topical gels are expensive and may not be covered by insurance. • A testosterone level in the midrange (400 to 600 ng/dL) is the goal • Mood, sense of well-being, and libido may vary among patients, dosages can be adjusted on these effects • Monitoring: o Evaluate the patient 3-6 months after Tx is started to determine efficacy in treating symptoms and ADRs o Evaluate the patient’s testosterone level 3 to 6 months after staring therapy (Goal is mid-normal range) ▪ Injectable: measure levels midway between injections ▪ TD: measure levels 3 to 12 hours after applying patch ▪ Buccal: measure immediately after applying a new dose ▪ Pellets: measure at the end of therapy ▪ PO: measure 3 to 5 hours after ingestions o Evaluate Hct at baseline, 3-6 months, then annually o Evaluate BMD after 1-2 years of therapy o Evaluate PSA levels and perform a digital rectal exam prior to beginning therapy, at 3 and 6 months, then per age related guidelines for prostate CA screening ▪ (Uro consult if concerns about therapy, increase in PSA levels greater than 1.4 ng/mL in 12 months, or prostatic abnormality is palpated) Contraindications for Testosterone Replacement Therapy Contraindications: • Abnormal digital rectal exam • Breast carcinoma (history of or presence) • Elevated PSA • Hypercoagulable states • Polycythemia (Hct > 51%) • Prostate carcinoma (history of or presence) • Psychopathology • Severe BPH • Severe CHF (Stage III or IV) • Untreated sleep apnea Risks: • Erythrocytosis is a risk of TRT, so checking Hgb and Hct periodically is recommended • Gynecomastia • Liver toxicity and liver tumor • Prostate Cancer – periodic check of PSA and digital rectal exam is recommended • Decreased sperm and infertility: Testicular atrophy and infertility • Adolescents may develop acne and gynecomastia, and more seriously, aggressive behavior and premature closure of growth plates, leading to small stature • Exacerbation of sleep apnea • Worsening s/s of BPH • Skin diseases (esp for topical and dermal routes) Erythrocytosis from Overuse or Abuse of Testosterone • Erythrocytosis is a risk of TRT, so checking Hgb and Hct periodically is recommended • Evaluate Hct at baseline, 3-6 months, then annually Rational Drug Selection for ED • Common sexual problem, has increased incidence with aging • Caused by a disruption of the normal interactions of the hormonal, vascular, neurological, and psychological systems. • Causes negative impact on intimate relationships, quality of life, and self-esteem • Treatment includes the use of phosphodiesterase type 5 (PDE-5) inhibitors or TRT, if appropriate. • Other treatments may include penile self-injection, vacuum-assisted erection devices, and/or cognitive behavior therapy. • Risk factors include: chronic illnesses such as DM, HTN, obesity, CVD, or dyslipidemia. Can also be caused by smoking or some medications. • First-line therapy for ED: lifestyle changes, modifying drug therapy that may cause ED, and pharmacotherapy with PDE-5 inhibitors • PDE-5 Inhibitors: sildenafil (Viagra), vardenafil (Levitra), & tadalafil (Cialis) o Part of the first-line therapy for ED, along with lifestyle management, modifying drug therapy o Most effective oral meds for ED o PDE-5 meds vary in onset and duration of action • Tadalafil: absorption is not affected by food, may assist w/ achieving erection from sexual stimulation for up to 36 hrs. • Sildenafil: long safety record, has a 4 hr duration of effectiveness, should be taken on empty stomach as high fat meals or alcohol may slow absorption • Vardenafil: has a 4 hr duration of effectiveness, should be taken on empty stomach as high fat meals or alcohol may slow absorption Contraindications for PDE-5 Inhibitors o Contraindications: o Sensitive to any of the components of the medication o CVD: AMI, stroke, or life threatening arrhythmia w/in the last 6 months o Use caution: unstable angina, severe HF or prolonged QT, hypotension (SBP <90), or HTN (BP >170/110) o Concurrent use of nitrates (fatal hypotension) Adverse Effects of PDE-5 Inhibitors o Drug/Drug interactions: o Potentially fatal hypotension when used w/ nitrates or nitroprusside o Hypotension when used with alpha blockers o Antihypertensives: additive hypotension o Visual disturbance: blue hue o Rare: sudden hearing loss o Priapism is a potential adverse effect, educate patient on this condition and need for seeking emergency care if erection lasts longer than 4 hrs. Chapter 50: Pediatric Patients Use of Topical Steroids in Young Children and Infants • Infants and young children have a thin stratum and larger body surface area in relation to size and this affects topical absorption of medication • Children absorb topical medications more readily than do adults which leads to systemic toxicity • Lidocaine or diphenhydramine can lead to systemic toxicity • Most providers are familiar with the concern for systemic absorption of topical corticosteroids in children, with systemic Cushingoid symptoms or hypothalamic pituitary adrenal axis suppression developing with topical use • Due to the Best Pharmaceuticals for Children Act and pharmacokinetic studies that indicated HPA axis suppression or adrenal suppression in children several topical corticosteroids have been relabeled o Diprolene (diprosone) cream, ointment, and lotion are not recommended for use in children younger than 12 y/o r/t HPA axis suppression o Diprolene AF and Elocon (mometasone) lotion are not recommended o Lotrisone (clotrimazole and betamethasone dipropionate) cream used for Tx of tinea pedia 39.5% of patients demonstrated adrenal suppression as determined by cosyntopin testing o Lotrisone cream used for Tx of tinea cruris 47% showed adrenal suppression by cosyntropin testing o Lotrisone has been relabeled and is not recommended for children younger than 17 y/o and not recommended for diaper dermatitis o Fluticasone (Cutivate) ointment: similarly relabeled to be used only in adults Influences on Medication Adherence o With adults, the provider can assume that, if reasonably clear instructions are given, the patient will take the medication as prescribed. o With children, many added variables affect administration of the medication and compliance with the medication regimen. o The first consideration is the developmental level of the child and the amount of parental control at each developmental level. o This section addresses these differences and suggests strategies for improving compliance at each age level. Breastfed Infants o Breastfeeding an infant, the first year of life is beneficial both physically and emotionally to the infant. o Therefore, when prescribing medications to a lactating woman, the practitioner needs to be aware of which medications can be used safely and which are contraindicated (Box 50-1). o The most up-to-date, peer-reviewed, and easily accessible resource is the LactMed database at the National Library of Medicine TOXNET Web site (http://toxnet.nlm.nih.gov/index.html). o LactMed is searchable by drug name and gives recommendations for prescribers with updated safety information. Infants o Totally dependent on their parents to give medications o Intervention at this age is aimed at teaching parents or caregivers how to properly administer the medication o Many parents are nervous the first time they give their child medication o Thorough education helps to ensure compliance and decrease risk of medication errors o ARF medication dosing errors by parents (common 39.4%) o Written or printed instructions are important Toddlers and Preschoolers o More independent and giving meds to this group can be a challenge o Key to success: discuss medication administration w/ parents and choose a med that poses the fewest problems with administration o Doses/day, palatability, and dosage forms should be taken into consideration o If the toddler is resistant: once daily or BID Rx is possible, chewable formulations (can self-administer), using higher concentrations if possible, to decrease volume administered o Listen to parents, they know their child and can help anticipate problems School-Age Children o Often easier r/t development (industrious and eager to learn) o Include child in the decision making process (if possible) o Let the child choose the formulation o Aim teaching at both the parent and child o Need to know the rationale for RX o Clarification about helpful medication versus illicit drugs o Avoid school hour dosing to simplify meds: regulations at school regarding administration Adolescents o Often administer their own medications however compliance rates vary with this age group o Some are excellent at self admin and others are poorly compliant o Developmentally entering the period of formal operational thinking, characterized by propositional thinking and abstract reasoning o Younger adolescents may still be in the concrete thinking stage and interactions may reflect this stage rather than abstract though process of older teenagers o Form a relationship with adolescents and ask their perspective regarding their medications o Teenagers appreciate having their opinions taken into consideration as Tx is planned o Be aware of laws of the state in which you practice: confidentiality if necessary o Teenagers too must understand the confidentiality laws of their state and what age they are able to receive confidential Tx o Parents often struggle with letting teens self admin meds: must be skilled at assisting family members as they move from parent controlled to child or teen controlled medication admin o Compliance or adherence with the medication regimen is an issue for all patients. o Pediatric patients pose a unique dilemma because the practitioner must address both the child’s compliance and the parent’s, plus possibly that of other caregivers. o The many factors that influence adherence include length of medication regimen; number of medications prescribed; the medication interval, palatability, cost, and family issues. o The practitioner needs to consider all these issues when prescribing to ensure successful treatment Specific Factors That Influence Compliance Long-Term Medication Regimens o Chronic illness, often including a daily medication regimen, presents several problems for the family o Asthma, ADHD o Even patients for whom noncompliance can be life-threatening are not taking their medications as prescribed o Children with sickle cell disease were found to have a refill rate for their daily medications of 58.4%. o Of 104 adolescents prescribed antiretroviral treatment for HIV, 65.4% reported full adherence, with nonadherence associated with higher viral load. o Risk factors for medication nonadherence after pediatric heart transplantation (median age 6 years) includes adolescent age at transplant, black race, Medicaid insurance, and ventilator or ventricular assist device support at transplant. Number of Medications Prescribed o The number of medications prescribed can have an impact on compliance with the regimen. o The more medications that are prescribed, the lower the compliance rate is. o Keeping medication schedules simple increases, the likelihood of success for the treatment. Medication Interval o Medication interval has a significant impact on the success of the treatment, especially given the number of families with both parents working and more children in day care. o Children who are in school or have parents who are both working may not receive their medications as often as recommended if they are taking any medication that needs to be administered more than twice a day. Palatability o Palatability is often overlooked as a reason for noncompliance, yet in children it is a critical factor in medication compliance. o Some antibiotics were ranked better tasting than others, with the cephalosporins (cefdinir, cefixime, cephalexin, and cefaclor) ranked as the best tasting overall. o Dicloxacillin ranked the worst for taste. o Although no published reports studied taste differences between brand-name and generic preparations, anecdotal reports from parents and patients suggest that brand-name preparations taste better. o Of medications with the same efficacy profile, the best tasting is the easiest to administer to young children. Cost o The cost of the medication needs to be addressed for patients who are not adequately insured o The cost of common antibiotics prescribed for otitis media range from $4 to more than $100 to treat a 15-kg child for 10 days. o Prescribing an expensive antibiotic for a family who cannot afford to fill the prescription places the family in an uncomfortable position. o Simply asking the family if they have insurance to cover the medication and then problem solving with them if they do not will increase the likelihood that the family will fill the prescription. o For example, if an antibiotic is indicated, check the retail $4 list first to determine if the medication is available. o If possible, give the family a few days of medication samples to defray the cost of the treatment if a less expensive medication is not available. o Knowing which pharmacies in the local area are the least expensive or calling ahead for a price check before sending the family to the pharmacy is helpful. o A family who knows the approximate amount that the medication will cost will not be surprised when the prescription is filled. Family Issues ▪ Family issues affect the family’s ability to comply with the prescribed treatment regimen. ▪ Families in which both parents are working and therefore have limited time with their children have more problems with complex treatment regimens. ▪ Lack of social support can leave a parent isolated and make parenting more stressful. ▪ Parental fatigue is often overlooked as a factor in treatment outcomes. ▪ Parents who are fatigued can easily miss medication doses; even those who are usually well organized can miss medication doses when they are tired. ▪ Disruptive and dysfunctional families may have difficulty in following the plan of treatment because of the chaos present in the home. ▪ Another family situation that needs to be addressed when clinical improvement is less than satisfactory is parental use of the child’s medications. o For example, a child may be prescribed stimulants for attention deficit-hyperactivity disorder, and a parent or other family member may be abusing the child’s medication. o This is a situation no practitioner wants to encounter, yet there should always be some index of suspicion when the family history is not clear. ▪ All these issues need to be accounted for when the practitioner is prescribing a medication and during follow-up on the patient’s progress. They often present in an unclear fashion and ferreting out the reason for noncompliance with the treatment regimen may take some time. ▪ Improving Compliance in the Pediatric Patient ▪ When poor compliance is identified, it is essential to address this issue and determine strategies with the patient and parent to improve the success of the treatment regimen. Improving Compliance in the Pediatric Patient Medication Concentration Medication concentration can be adjusted in some of the liquid preparations. The practitioner can choose to prescribe a more concentrated form when a parent has difficulty administering medications to a patient. Many of the antibiotics come in different strengths, and giving one-half teaspoon is easier than administering a full teaspoon. Prednisone comes in two different strengths, as well as in tablets that can be crushed. By involving the parents in the decision to use a more concentrated form of a medication, you are allowing them some control over the treatment regimen, and they may therefore be more likely to administer the medication that is prescribed. Written Versus Oral Instructions Most practitioners should address the issue of written versus oral instructions in their own practice. Giving written instructions along with the oral directions will improve compliance. This is especially important for over-the-counter medications such as acetaminophen and ibuprofen. Parents need to understand the different formulations and dosing by weight, and every parent should have a weight-based dosing chart. Self-Monitoring Calendars Self-monitoring calendars should be a standard in the treatment of preschool and school-age children. Children can apply a sticker or color in a box as each dose is taken. Parents should be involved in the process and set a reward for completion of the medication regimen. In acute illness such as otitis media, in which the patient will be returning to the clinic, the practitioner may offer a reward for a full calendar. Children with chronic illness, who are often on long-term medication, need to have a set reward for a certain number of days of successfully taking their medication. Parents need to take an active role in medication calendar usage, and they, too, should be praised for their participation in the medication regimen. Telephone/E-mail Reminders Telephone or e-mail reminders are helpful in increasing compliance, especially if the parent is leaving the clinic with multiple prescriptions. A quick telephone call or e-mail allows the parent to clarify the treatment regimen and reinforces teaching that took place in the clinic. Mobile Phone Medication Adherence Apps There are several mobile phone apps available for reminding patients to take medications. A medication regimen does not have to be complex to use a reminder app. For example, an app can be used to help remind adolescent females to take their daily oral contraceptive pill. Contracts and Reinforcement Programs Contracts or reinforcement programs may be necessary if compliance continues to be a problem. The practitioner, the patient, and the family need to agree about the goals of the treatment contract and the consequences of noncompliance. A case conference may be necessary to involve other disciplines in the treatment. A home visit may provide information that leads to an altered treatment program that will be better tolerated by the family. The role of the practitioner is to attempt to simplify the medication treatment and still have an adequate therapeutic outcome. This goal should be shared with the patient and family. Pain Relief in Infants and Children o Box 50-2 o Pain in children can range from teething pain to pain associated with OM o Parents often ask about using acetaminophen or ibuprofen for the Tx of pain in children o For the safety of their children and the efficacy of the medication, parents should be taught how to administer OTC pain medications properly o The two most used analgesics in peds are acetaminophen and ibuprofen o ASA should never be given to children for acute pain r/t risk of Reye syndrome (teach parents this) o Ibuprofen is effective for pain control and has an additive anti-inflammatory effect which appears to provide better pain control in acute OM than acetaminophen o Do not give to children younger than 6 months old r/t immature kidneys are unable to excrete it o Leads to increased risk of toxicity and renal damage o All children receiving ibuprofen should be well hydrated to prevent renal damage o Avoid ibuprofen if fluid intake is decreased o Although acetaminophen and ibuprofen provide good pain relief for mild to moderate pain and both have antipyretic effects, ibuprofen is the drug of choice for night pain associated with OM r/t its longer duration o Both drugs are equally easy to administer (ibuprofen is not available in suppository form) o Combining acetaminophen and ibuprofen in an alternating schedule for fever or pain is not recommended in the OP setting o One or the other, properly doses, should be used o The goal to antipyretic therapy is not to reduce temperature to normal but to decrease discomfort associated with fever o Give parents a dosing chart with the child’s dose based on weight o The different strengths of acetaminophen must be dosed correctly Dosing of Acetaminophen and Ibuprofen in Infants and Children o Acetaminophen o PO or rectally, peaks in 30 to 60 minutes o Dosage: 10-15 mg/kg/dose q 4-6 hours o Ibuprofen (over 6 months old) o 5-10 mg/kg/dose q 6-8 hours Education on Medication Administration for Infants • Parents are often unsure how to administer medication to an infant. • While the parents are in the clinic with the child, the practitioner should address this issue and demonstrate how to do it. • For ease of administration, use a medication syringe and insert the syringe into the mouth along the inner cheek. • To decrease choking, advise parents to squirt small amounts (1 mL) of medication at a time into the inner buccal space. • Wait until the infant swallows, and then administer another small amount until all the medication is administered. • Direct parents not to administer the medication directly over the tongue, which increases choking and allows the infant more easily to spit out the medication. • Advise parents to check with the pharmacist before mixing any medication with formula or breast milk; some medications are bound with the calcium or other ingredients in the formula, causing them to be less effective. • Breastfed infants often choke and sputter when medications are first administered because these infants are used to only the feel of the breast in their mouths. o Warning parents of this response and teaching them proper technique will help them gain confidence in medication administration. • Giving acetaminophen in suppository form is an option if administering oral medications to the infant is difficult. o The practitioner can demonstrate this procedure, which works well in breastfed infants especially. Chapter 51: Geriatric Patients Physiologic Changes in Geriatric Patients (Effects of Aging) • From Ch. 48 (Women as Patients) o Thinning and graying of the hair, weight gain, drying skin, vision changes associated with presbyopia, and increased healing time for musculoskeletal injuries o Female GU tact changes: less of the ureterovesicular angle and detrusor muscle instability result in frequency and leakage with increased residual bladder volume o Breast composition changes from dense glandular tissue to fatty replacement • From Ch. 49 (Men as Patients) o Testosterone decreases with aging • From Ch 51 (Geriatric Patients) • Mental changes o Mental changes in the healthy older adult are minimal o White brain matter does deteriorate at about 60 y/o o Disease processes inhibit the older adults of the ability to think clearly and maintain independence o Both acute (hypoglycemia, infection, electrolyte imbalances) and chronic (Alzheimer’s and CV accidents) o Cerebral blood flow, brain mass, and neurotransmitter concentrations are reduced as we age: more susceptible to neuro ADRs (Can cause delirium) o Older adults have a reduced immunologic function and reduced ability to generate a fever: often present with confusion • Sensory Changes • Sight: o Almost 1/3 of adults over 65 have some visual impairment o Presbyopia: results from loss of elasticity in and thickening of the lens (impairs near vision) o Conditions that become increasingly prevalent with age: cataracts, glaucoma, and macular degeneration o Create difficulties for the patient in self-management of medication and can restrict the pharmacological choices available to Tx the pt.’s medical condition to prevent worsening of the eye disease o Visual loss also makes it difficult for patients to manage their own medication or provide their own transportation: Causes them to be increasingly dependent upon others • Hearing: about 1/3 of adults over 65 have some level of hearing loss, especially in the high frequency range, and the incidence of hearing impairment increases with each decade o Cerumen impaction, seen in over 1/3 of older adults further impairs hearing ▪ Drier cerumen and are more likely to use hearing aids which contribute to impaction o Hearing loss may be gradual and be unaware that they are not fully hearing what is spoken o Missing one or two words in the instructions for an Rx could have devastating consequences • Smell and Taste: o Smell and taste decline with aging and are further impaired with the use of medications o These sensory losses may impair appetite and nutritional status: lack of dietary iron or B vitamins can lead to atrophic changes of the oral mucosa, results in dry mouth and changes in taste o Some older adults may c/o a burning mouth and tongue o Local trauma, poor nutrition, DM, or anemia can cause these symptoms o Older adults also respond to taste and texture in taking oral suspensions • Peripheral Sensation: o Peripheral neuropathy, ARF falls o Problems with circulation, vitamin deficiencies, and DM can aggravate age related loss of peripheral nerve function o Many classes of meds further contribute to fall risk, especially in the frail older adults o Prescribing requires a careful assessment of risk versus benefit of these agents • Musculoskeletal Changes o Changes range from impaired manual dexterity to mobility o Can cause difficulty with drug administration and adherence o Mobility problems (gait, limb function, manual dexterity, and driving): threaten independence and functioning specifically their adherence to drug therapy o Neuro disease (CVD, Parkinson’s, and motor neuron disease): can lead to increased difficulties in the practical application of drug treatments o Such problems may range from the inability to visitor the doctor’s office to the inability to open the childproof containers o Immobility from joint deformity, pain, and impaired manual dexterity may make activating inhalers or nebulizers or applying eye drops difficult to perform o Some medications may dramatically improve patient’s mobility (Meds for Parkinson’s), but others may dramatically exacerbate mobility problems (antihypertensives, tricyclic antidepressants) Pharmacodynamic and Pharmacokinetic Implications of These Changes Pharmacokinetic Changes Absorption and Distribution • Older adults undergo normal age-related changes in the body that affect the action of bioactive substances • General absorption of drugs is not dramatically different in the older adult compared to the younger population for most prescribed and OTC medications. • Oral drugs are absorbed by the gastrointestinal (GI) tract. o With aging, the GI tract produces less acid as fewer parietal cells are generated, but the change is too small to be clinically relevant. o However, this small change does become clinically significant in the presence of drugs that compound this problem, such as proton pump inhibitors. o There is decreased active transport of some drugs, causing decreased bioavailability. • Total body fat increases with age, whereas subcutaneous fat decreases o Frail, older adults can have little subcutaneous fat, can reduce the absorption of TD drugs. o This can be clinically important when managing pain with transdermal systems. • Distribution of drugs can be dramatically changed with age. o Aging results in a reduction of lean muscle mass by about 20% and total body water by 10% to 15%, causing changes in the distribution of many drugs. o Pharmaceutical agents primarily distributed in lean body mass or body water may reach higher serum concentrations in older adults, resulting in magnified effects compared to similar doses in younger adults. o Water-soluble drugs, such as lithium and digoxin, can have a smaller volume of distribution, resulting in a higher peak plasma concentration at normal dosages due to the lower ratio of water to fat in the body of the older adult. ▪ This is especially important for drugs with narrow therapeutic ranges. Increased bioavailability is also apparent with drugs administered IM, which are often highly water soluble. o Lipid-soluble drugs, such as benzodiazepines, have a higher tendency to accumulate in adipose tissue, resulting in lowered serum concentrations. ▪ This accumulation in the adipose tissue can result in a prolonged duration of action due to an increased half-life. ▪ The result is a less intense immediate medication effect as well as a prolonged or unpredictable effect in the older adult. • Normal aging also results in a decrease, by as much as 20%, in serum albumin levels. o Albumin is the primary drug-binding protein in the plasma. o The consequence of these lowered serum albumin levels is fewer protein molecules available for binding to the drug and higher levels of free, unbound drug. o It is free drug that exerts the physiological effect, so the result of decreased albumin is an increased potential for drug toxicity. o This problem can be aggravated by poor nutrition, age-related liver changes, or the competition of multiple medications for protein-binding sites. o With decreased binding ability, drug doses need to be reduced to prevent deleterious effects. Metabolism and Excretion ▪ Aging results in a decrease in both liver size and blood flow ▪ Drug clearance by the liver depends on hepatic blood flow and enzyme activity. ▪ Acetylation and conjugation do not change appreciably with age. ▪ Oxidative metabolism through the cytochrome P450 (CYP450) system does decrease with aging, resulting in a decreased clearance of some drugs. o The oxidative reactions of phase I metabolism decline secondary to these changes in the liver, resulting in decreased drug clearance and an increase in half-life for agents such as amitriptyline, morphine, and propranolol that rely on high rates of blood flow through the liver for extraction. o In contrast, phase II metabolism is relatively unaffected by age, so that benzodiazepines such as temazepam and oxazepam, which are metabolized through phase II conjugation reactions, are better tolerated than diazepam, metabolized through phase I reactions. o Reduction in metabolism is more pronounced in malnourished or frail elders. ▪ First-pass metabolism is also decreased so that drugs like propranolol and verapamil, which undergo first- pass metabolism, have increased bioavailability in older adults. o On the other hand, pro-drugs which rely on first-pass metabolism for activation, will act more slowly in older adults. ▪ Most drugs are eliminated through the kidney. o The human kidney loses about 1% of its function every year, beginning at age 30, so that by age 80 there is generally a 50% decline in renal function. o Dosing guidelines are available to help providers make appropriate adjustments for renal impairment. o Examples of drug classes for which there is evidence of age-related reduction in clearance include antihypertensives, fibrates, sedative/hypnotic, and anxiolytic medications. o Disease processes may be more of a factor in the reduction of renal function than normal aging. o Diabetes and hypertension, for example, have been found to be powerful determinants of renal dysfunction in the very old. ▪ Determination of normal renal function is not easily established in the older adult. o Although serum creatinine is the most reliable and easiest test to determine renal function in younger persons, it is an unreliable marker in older adults. o Renal function tests are affected by poor nutrition and reduced lean muscle mass. o Adequate protein ingestion is a requirement for evaluating accurate BUN, and adequate muscle mass is a requirement for determining accurate serum creatinine levels. o Creatinine clearance evaluated with the Cockcroft-Gault equation is the best method for this evaluation. o Although it is the most accurate method of evaluation, this equation still has limitations, especially in frail elders for whom it may overestimate creatinine clearance. Table 51-1 Common Pharmacokinetic and Pharmacodynamic Changes in Older Adults Process Changes in Older Adults Implications Absorption Change not clinically significant Oral drugs: Decreased acid production by parietal cells; delayed gastric emptying; reduced blood flow to GI tract. IM drugs: Decreased lean muscle mass. Use of drugs that have same action as aging change may increase problem to level of clinical significance. Distribution Increased fat stores Lipid-soluble drugs have greater Vd and longer half-lives. Vd = volume distribution Decreased body water Water-soluble drugs have smaller Vd and higher peak plasma levels. Decreased serum albumin levels Decreased drug-protein binding, increased levels of free drug. Especially a problem for drugs with high protein- binding percentages. Changes onset and duration of action of highly tissue-bound drugs as well. Metabolism Decreased hepatic blood flow Less first-pass effect so increased amount of drugs that have high first-pass breakdown and decreased amount of pro- drugs that require first-pass activation. Decreased CYP450 system function Decreased metabolic clearance of drugs. Decreased metabolism of some drugs. Altered drug-drug interactions. Excretion Decreased renal mass and GFR Decreased tubular secretion Decreased renal clearance of drugs. May require dosage adjustments. Treat older adults as if they had renal impairment. Pharmacodynamic changes Reduced thermoregulatory ability Increased hypothermia risk. Direct effect on phenothiazines, BDZs, TCAs, and narcotics. Impaired baroreceptor function and altered fluid status Postural hypotension with antihypertensives, TCAs, MAOIs, antihistamines. Pharmacodynamic Changes ▪ One of the characteristics of old age is a progressive decline in counterregulatory (homeostatic) mechanisms and altered receptor sensitivity. o Pharmacodynamic changes of aging involve increased sensitivity to drugs such as anticoagulants, and cardiovascular and psychotropic drugs due to common physiological changes in the cardiac and neurological systems. o Decreased elasticity in the blood vessels and reduced heart rate responsiveness to the demands of exercise increase the risk for orthostatic hypotension in older adults. o Reductions in cerebral blood flow and neurotransmitter concentrations enhance the neurological effects of pharmacological agents. o The results of this decline include less mitigation of drug effects, more intense drug reactions than those experienced by younger patients, and a higher rate and intensity of ADRs ▪ Reflex tachycardia, commonly seen with vasodilator therapy, is often blunted, possibly due to dampened baroreceptor response. o Caution must be exercised with the use of anticholinergic medications because older adults are sensitive to ADRs related to this class of medications. o ADRs include cognitive impairment, hallucinations, dry mouth, blurred vision, glaucoma, constipation, nausea, urinary retention, and tachycardia ▪ Narcotic and psychoactive drugs: over sedation, confusion, and respiratory depression and distort the patient's sense of balance. ▪ Drugs that lower BP may cause orthostatic hypotension. ▪ Some drugs used to treat diabetes are more likely to produce hypoglycemia. ▪ Diuretics carry an increased risk for fluid and electrolyte imbalances. ▪ NSAIDs are more likely to produce GI bleeding, cognitive impairment, fluid overload, and hypertension. ▪ Coumarin anticoagulants have a greater effect on clotting factor synthesis: warfarin doses are generally lower in older adults ▪ Some OTC cold remedies with anticholinergic side effects may cause the patient with glaucoma to experience vision loss secondary to the increase of intra-optic pressure and commonly cause urinary retention and constipation. ▪ Paradoxically, some older adults are less sensitive to certain drugs. o decreased response to a beta blocker (propranolol), resulting in smaller reduction in heart rate or only a mild increase in heart rate in response to a beta agonist (epinephrine). Steps to Avoid Polypharmacy ▪ Several steps can be taken to reduce polypharmacy: 1. Obtain a complete drug history and review current drugs (prescription and others) every 6 months. Look for drugs without indications and discontinue. (Many drugs must be tapered rather than abruptly discontinued to avoid serious withdrawal symptoms). 2. Avoid prescribing when the benefit is questionable. Consider the age of the patient and the stage of the disease when evaluating the benefit of prescription drug use. 3. Evaluate for duplications in drug therapy. Simplify the regimen by using drug combinations or by prescribing single drugs that will provide multiple therapeutic effects whenever possible. The use of combination drugs improves adherence when compared to dual therapy. 4. Review the medication list for drugs prescribed for an ADR. If these exist, explore whether the original drug can be withdrawn or changed to avoid this reaction. Avoid treating adverse reactions/side effects with more drugs. Treat the underlying condition, rather than the symptoms, if possible. 5. Prescribe lifestyle changes and other nondrug therapies whenever possible. These may require support from family and other health-care providers. 6. Clearly understand the difference between manifestations of the aging process and the disease state that must be treated. Common Herbal Medications Used by Elderly Patients o Fish oil, garlic, ginseng, gingko, ginger, Vitamin E o Gingko biloba (prevent or Tx of dementia) o Saw palmetto (relieve symptoms of BPH) o Chondroitin sulfate and glucosamine sulfate (osteoarthritis) o St. John’s Wort and SAMe (S-Adenosyl-L-Methionine) (depression) Hypoglycemia in Geriatric Patients o Some drugs used to treat diabetes are more likely to produce hypoglycemia o “Low concern” drugs: those that should not be used in older persons known to have specific medical conditions o These medications include long acting oral hypoglycemics [Show More]

Last updated: 1 year ago

Preview 1 out of 126 pages

Add to cart

Instant download

document-preview

Buy this document to get the full access instantly

Instant Download Access after purchase

Add to cart

Instant download

Reviews( 0 )

$15.50

Add to cart

Instant download

Can't find what you want? Try our AI powered Search

OR

REQUEST DOCUMENT
96
0

Document information


Connected school, study & course


About the document


Uploaded On

Mar 22, 2023

Number of pages

126

Written in

Seller


seller-icon
securegrades

Member since 4 years

117 Documents Sold


Additional information

This document has been written for:

Uploaded

Mar 22, 2023

Downloads

 0

Views

 96

Document Keyword Tags

Recommended For You

Get more on STUDY GUIDE »

$15.50
What is Browsegrades

In Browsegrades, a student can earn by offering help to other student. Students can help other students with materials by upploading their notes and earn money.

We are here to help

We're available through e-mail, Twitter, Facebook, and live chat.
 FAQ
 Questions? Leave a message!

Follow us on
 Twitter

Copyright © Browsegrades · High quality services·